Sei sulla pagina 1di 120

Universidad La Salle. Facultad Mexicana de Medicina.

Curso de Extensin Universitaria para la Preparacin del Examen Nacional para Aspirantes a Residencias Mdicas. Examen de Gineco.obstetricia.

Nombre: Examen del Mdulo I. Nmero de intentos: 3. Vigencia: 27 de Febrero del 2013. Horario: 7:00 p.m. a 9:00 p.m. Programar aleatorio. (4 bloques con 20 preguntas cada uno).

1.- Femenino de 51 aos, con mioma uterino de tamao equivalente a una gestacin de 12 semanas, que presenta hipermenorreas y hemoglobinemia de 9 gr%. No se demuestra patologa asociada. Se encuentra en espera para la prctica de una histerectoma programada a realizar en 4 meses. En esta paciente est indicado el tratamiento preoperatorio con: a) b) c) d) Estrgenos. Inhibidores de la fibrinlisis. Derivados del cornezuelo del centeno. Anlogos de la GnRH.

Anlogos de la GnRH: Son derivados de la hormona GnRH en donde se ha realizado una sustitucin peptdica en posicin 6 y en algunos casos en la 10, obteniendo compuestos hasta unas 200 veces ms potentes debido a mayor afinidad por los receptores y a su resistencia a la degradacin por peptidasas. Aunque su accin inicial produce un incremento en la produccin de FSH-LH (efecto flure-up o llamarada) tras 5-6 das de exposicin contnua, los receptores son internalizados producindose un estado de hipogonadismo hipogonadotropo y niveles de estradiol similares a los de la postmenopausia. Los anlogos de GnRH estn disponibles en distintas frmulas: administracin nasal (varias aplicaciones al da), subcutnea (aplicacin diaria) o intramuscular (preparados depot mensuales o trimestrales) (Shaw RW 1999). Marco Filicori y sus colaboradores de la Universidad de Bolonia fueron los primeros en utilizar en 1983 los aGnRH en un estudio que confirm su eficacia para reducir el tamao de los miomas uterinos y secundariamente sntomas como alteraciones menstruales, dolor plvico y sntomas de presin local. Otros autores como Minaguchi H y colaboradores continan comprobando la efectividad de los anlogos de la GnRH en el tratamiento del mioma uterino tras evaluar en el ao 2000 seis estudios con un total de 602 pacientes tratadas con nafarelina. La disminucin del tamao se calcula entre un 30-70%, y se ha

observado como el mayor porcentaje de reduccin ocurre tras el primer mes de tratamiento, no existiendo reducciones o siendo stas mnimas despus del tercer mes (Healy et al 1986; Friedman et al 1989; Matta et al 1989; Williams y Shaw 1990). En miomas pediculados o con gran proporcin de calcio o colgeno (hialinizacin) la repuesta es tambin menor. Debe tenerse en cuenta que si despus de dos meses de tratamiento no se ha producido un significativo descenso del tamao del mioma, ste ya no debe ser esperado y debe pensarse en la posibilidad de la existencia de un tumor muscular maligno no diagnosticado (Messia AF et al 1998). Tras finalizar el tratamiento y recuperarse el estado de hipogonadismo, el mioma retorna rpidamente a su tamao inicial (Friedman AJ et al 1987; Matta WH et al 1989). En casos prximos a la menopausia, la reduccin del tamao del mioma y su sintomatologa, permitira hablar de una solucin mdica del problema, pero en todo caso la utilizacin de aGnRH facilitara la intervencin quirrgica al acortar el tiempo de intervencin, la hemorragia y el acceso a localizaciones complicadas como el caso de miomas interligamentarios o situados en istmo o crvix. En el caso de la ciruga histeroscpica la reduccin del tiempo de ciruga permitira reducir el volumen de fluidos aportado a cavidad uterina y los riegos de absorcin e hiponatremia. Los mecanismos de accin por los que los aGnRH actan son: o Hipoestrogenemia: es necesario mantener la hipoestrogenemia, pues la elevacin de sus niveles lleva a un rpido incremento del tamao del mioma. El crecimiento del mioma es dependiente de los niveles de estrgenos (aumentan de tamao con el embarazo y se reducen durante la menopausia o el tratamiento con aGnRH, pudiendo volver a crecer durante la THS), pero aunque los estrgenos parecen ser importantes en el crecimiento del mioma, su relacin debe ser algo ms compleja pues no se han descrito incrementos significativos del tamao de miomas durante el tratamiento con gonadotrofinas en RA (situaciones con elevados niveles de E2), algunos de ellos no se modifican durante el embarazo o incluso decrecen y se han encontrado crecimientos despus del tratamiento con citrato de clomifeno (antiestrgeno). En relacin con la hipoestrogenemia podran estar los cambios inducidos en el flujo vascular uterino (incrementos en el ndice de resistencia de las arterias uterinas) que suponen una reduccin de la vascularizacin o las modificaciones de distintos factores de crecimiento. o Cambios histolgicos: el tratamiento con aGnRH puede producir degeneracin roja, infiltracin linfocitaria, y necrosis, as como reduccin de la proliferacin celular e incremento de la apoptosis. Pero en otras circunstancias no es posible encontrar 7 diferencias. No se ha encontrado una relacin entre los cambios histolgicos y el porcentaje de reduccin del tamao del tero, y existe una gran variabilidad entre distintas pacientes o entre distintos miomas de una misma paciente, no existiendo pues un patrn histolgico caracterstico de respuesta ante el tratamiento con aGnRH.

BIBLIOGRAFA: 1. Abad L, Abad de Velasco L, Parilla JJ. Etiopatogenia. Papel de las hormonas esteroideas, factores de crecimiento y otras sustancias. Cuad Med Reprod 1999;5(1):15-29. 2. Albano C, Platteau P, Devroey P. Gonadotropin-releasing hormone antagonist: how good is the new hope? Curr Opin Obstet Gynecol 2001;13(3):257-62. 3. Coutinho EM.Treatment of large fibroids with high doses of gestrinone. Gynecol Obstet Invest 1990;30(1):44-47. 4. Chavez NF, Stewart EA. Medical treatment of uterine fibroids. Clin Obstet Gynecol 2001;44(2):327-84. 5. De Leo V, la Marca A, Morgante G. Shortterm treatment of uterine fibromyomas with danazol. Gynecol Obstet Invest 1999;47(4):258-262. 6. Eldar-Geva T, Healy DL. Other medical management of uterine fibroids. Baillieres Clin Obstet Gynaecol 1998;12(2):269-88. 7. Felberbaum RE, Germer U, Ludwig M, Riethmuller-Winzen H, Heise S, Buttge I, Bauer O, Reissmann T, Engel J, Diedrich K. Treatment of uterine fibroids with a slow-release formulation of the gonadotrophin releasing hormone antagonist Cetrorelix.HumReprod 1998;13(6):1660-8.

2.- Femenino de 25 aos G-3, P-1, A-1 con 39 SDG por FUR. Reporta contracciones uterinas que han sido regulares las ltimas tres horas. Al examen encuentras que las contracciones son cada tres minutos y duran 50 segundos y son firmes a la palpacin. Tuvo ruptura de membranas hace una hora y lo demuestras con papel de nitrazina. El examen digital cervical demuestra una dilatacin de 5 cm, con borramiento del 100% y presentacin en vrtex en estacin 0. Cual de los siguientes criterios es el ms preciso para decir que se encuentra en la fase activa del trabajo de parto?

a) b) c) d)

Dilatacin cervical mayor de tres centmetros Borramiento cervical ms de 90% Duracin de las contracciones de ms de 30 seg Ruptura de membranas

FASES DEL TRABAJO DE PARTO El trabajo de parto se divide en tres fases: Fase 1 latente Es llamado as al periodo que sirve para la preparacin uterina del parto, ocurre al final del embarazo y va hasta el inicio de las contracciones del trabajo de parto. Los aspectos a destacar en este lapso es el reblandecimiento cervical, el aumento importante en el nmero de receptores para oxitocina a nivel de las clulas endometriales, un aumento sustancial en los puentes de unin y el nmero de conexinas a nivel miometrial y por consiguiente una mayor sensibilidad a los agentes uterotnicos. Fase 2 activa Es el lapso que representa el trabajo de parto activo, y se acepta que se inicie cuando existen 3 cm de dilatacin y las contracciones uterinas son aptas para producir un avance en el trabajo de parto; se divide en tres periodos: Primer periodo. Se inicia cuando las contracciones uterinas alcanzan la frecuencia, intensidad y duracin suficientes para causar borramiento y dilatacin del cuello uterino, y finaliza cuando ste se encuentra en completa dilatacin. El lapso de tiempo que dura es variable, pero se acepta como normal hasta diez horas en primigrvidas y ocho horas en multigrvidas; pero independientemente de esto, se debe considerar como adecuado si el borramiento y la dilatacin cervical son progresivos e ininterrumpidos. Segundo periodo. Se inicia con una dilatacin cervical completa y termina con la expulsin del feto; tiene una duracin variable, pero se acepta como normal una hora en pacientes primparas y 30 minutos en multparas; y tiene como caracterstica que debe de ser progresivo e ininterrumpido.

Tercer periodo. Este comienza inmediatamente finalizada la expulsin fetal y termina con la expulsin total de la placenta y las membranas corioamniticas; a este periodo se le conoce tambin como de alumbramiento y es el ms corto de los periodos del parto; como norma general se acepta que no debe de extenderse ms all de 10 minutos. Existen algunos autores que incluyen un cuarto periodo dentro del trabajo de parto, el cual abarca aproximadamente la hora posterior al alumbramiento, y comprende el lapso de tiempo cuando ocurre la contraccin y retraccin de las fibras miometriales, as como la trombosis de los vasos adyacentes, lo cual es un efectivo control de la hemorragia del sitio de implantacin de la placenta. Fase 3
Este periodo es el que representa el regreso de la mujer a su estado previo al embarazo, y se caracteriza por la involucin uterina, la eyeccin lctea y por ltimo la restauracin de la fertilidad; existen estudios que involucran en esta fase a la endotelina-1 y a la oxitocina como substancias responsables de estos cambios postparto.

PROGRAMA DE ACTUALIZACION CONTINUA PARA GINECOLOGA Y OBSTETRICIA PAC GO-1 Libro 3 Obstetricia 2005

3.- Femenino de 64 aos de edad acude a consulta externa con la siguiente sintomatologa: plenitud, estreimiento, distensin abdominal se acompaa de USG. Con resultado de imagen qustica en ovario derecho de 15 por 15 cms. El diagnstico ms probable es: a) b) c) d) Teratoma qustico. Disgerminoma. Endometrioma. Cistadenoma seroso.

Los Tumores de Ovario son una patologa frecuente dentro del contexto de la patologa femenina. Por esta causa consultan un grupo elevado de mujeres, tanto las consultas de ginecologa como las de Ciruga propiamente dicha. Las edades oscilan desde las tempranas hasta las ya avanzadas, siendo el riesgo de degeneracin maligna muy variable y relacionado con le edad. La experiencia de la clnica revela la alta incidencia de tumores de ovario en la etapa del climaterio, comprendida entre los 35 y 65 aos de edad 1. El cistoadenoma seroso de ovario (CSO) es un tipo de tumor derivado del epitelio superficial (celmico), formado por reas qusticas. El cistoadenoma seroso de ovario es el tumor ms frecuente de aquellos que provienen del epitelio celmico superficial. Hay tumores pequeos macroscpicamente y tumores masivos que ocupan toda la pelvis e incluso la cavidad abdominal. Estas frecuentes neoplasias qusticas uniloculares estn tapizadas por clulas epiteliales altas, cilndricas y ciliadas, llenas de un lquido seroso claro y de superficie lisa con abundantes vasos. Las variedades benigna, limtrofe y maligna representan, en conjunto, 30% aproximadamente de todos los tumores del ovario. El riesgo de presentar tumores epiteliales se incrementa con el paso de la edad, ya que pese a que la declinacin de la funcin ovrica marca el envejecimiento gonadal progresivo, el ovario humano nunca pierde su capacidad para generar tumores. Por lo general, cuando es detectado, su tamao es grande, en donde la imagenologa puede ayudarnos a considerar su diagnstico.

1. Captulo 22 Tumores Benignos de Ovario. En: Novak ER, Jones G., Jokes HW. Tratado de Ginecologa. 9 ed. Ciudad de la Habana. Editorial Cientfico Tcnica; 1977.p.432 66.

2. MedlinePlus Enciclopedia Mdica en Espaol: Quistes Ovricos. Disponible en: http://vsearch.nlm.nih.gov/vivisimo/cgibin/querymeta?v%3Aproject=medlineplusspanish&s pell=spell&query=Quistes+Ov%C3%A1ricos Acceso: Actualizado 20/6/06. 3. Captulo XL Tumores Ovricos En: Llusi Botella J, Nez Clavero JA. Tratado de Ginecologa. Ciudad de la Habana. Editorial Cientfico Tcnica. 1983; T 3.1; p. 751 803.

4.- Femenino de 23 aos acude al servicio de ginecologa, por referir ciclos opsomenorreicos, desde el inicio de su menarquia, en los ltimos 7 das ha incrementado 15 Kg. de lo que pesaba habitualmente. Exploracin Fsica: acn facial importante, as como bigote. El diagnostico ms probable en esta paciente es: a) b) c) d) Sx. De Asherman Sx. Stein Leventhall Sx. Amenorrea Galactorrea Sx. Karman

Sndrome de Ovario Poliqustico (SOP) es uno de los ms comunes trastornos endocrinos que afectan a las mujeres alrededor del 5% al 10% de las mujeres en edad reproductiva (12-45 aos) y se piensa que es una de las principales causas de la infertilidad femenina. Las caractersticas principales son la obesidad, anovulacin (dando lugar a la menstruacin irregular) o amenorrea, acn, y las cantidades excesivas o los efectos de andrognicos (masculinizantes) hormonas. Los sntomas y la severidad del sndrome varan mucho entre las mujeres. Si bien las causas son desconocidas, resistencia a la insulina, la diabetes y la obesidad estn fuertemente correlacionadas con el SOP. Bulun SE, Adashi EY. The physiology and pathology of the female reporductive axis. In: Kronenberg HM, Melmed S, Polonsky KS, Larsen PR, eds. Williams Textbook of Endocrinology. 11th ed. Philadelphia, Pa: Saunders Elsevier; 2008:chap 16.

5. - Femenino de 19 aos, atendida en sala de urgencias ginecoobsttricas, Antecedente: cursa embarazo de 38 SDG. Exploracin Fsica: en trabajo de parto. Repentinamente presenta sangrado profuso transvaginal y dolor abdominal. Si la paciente presenta abruptio placentae EL factor de riesgo ms frecuente es? a) Edad materna b) Elevada paridad c) Trauma abdominal d) Hipertensin materna

Se han sugerido numerosos factores que desempean un papel causal en el abruptio placentae, pero no existe una explicacin etiolgica satisfactoria para cada uno de ellos: Traumatismo. Malformacin o tumoracin uterina. Brevedad de cordn umbilical (menor de 20 cm) Descompresin brusca del tero (RPM, expulsin de un 1 gemelo) Compresin de la vena cava inferior (sumamente raro y no demostrado)

Hipertensin materna: ms del 50% de los casos de desprendimientos estn asociados a HTA. Deficiencia de cido flico. No demostrado. Tabaquismo. Asociado por la necrosis de vasos deciduales (anomalas deciduales). Paridad y edad materna. En general la asociacin de mayor edad y paridad, es no demostrable, No se puede descartar, que la paciente, por lgica, que sufre un desprendimiento, generalmente Es mayor de 20 aos (no excluyente) y multpara. Sin embargo, si hay relacin en que la paciente que sufri un DPPNI, tiene 5 veces ms probabilidades de sufrir otro DPPNI, independientemente de la edad y paridad. Iatrogenia. Versin externa y PTC induccin. Obstetricia. Scwarcz, Sala, Duverges. 7 edic. Edit. El Ateneo. (Biblioteca Fac. Med. UNNE).

6.- Mujer de 21 aos, que presenta una tumoracin de 2 cm de dimetro en el cuadrante nfero-externo de la mama izquierda, indolora, de consistencia firme, superficie lisa, forma ovoidea, mvil y bien delimitada del parnquima vecino, sin antecedentes de derrame por el pezn, sin piel de naranja ni retraccin del pezn, El diagnstico ms probable es:

a) b) c) d)

Fibroadenoma. Carcinoma. Ectasia de los conductos mamarios. Quiste solitario.

FIBROADENOMA MAMARIO Tumor benigno ms frecuente en las mujeres entre los 20 y 35 aos. ETIOLOGIA Existen mltiples teoras siendo la ms aceptada la hormonal, generalmente son nicos, solo el 20% son mltiples o bilaterales. De tamao variable hasta de 10 cm. Ocupa el 13.6% de la patologa mamaria benigna.

CUADRO CLNICO Lesin nodular de consistencia dura, de larga evolucin y no dolorosa. Normalmente llegan a los 3 cm. De dimetro. Durante la fase tarda del ciclo menstrual el tumor suele presentar un leve aumento de tamao. Durante la menopausia presentan regresin hasta la calcificacin (signo de palomitas de maz). DIAGNOSTICO Es clnico, se presenta como un tumor bien delimitado, desplazable, no adherido a piel ni a planos profundos, liso o multilobulado en ocasiones. Se localiza frecuentemente en cuadrantes externos. EXMENES DIAGNSTICOS ULTRASONIDO MAMARIO.- Identifica un ndulo slido, bien delimitado de bordes regulares. TRATAMIENTO.Conservador con vigilancia estrecha dependiendo del tamao y en caso de ser necesario exresis del ndulo para estudio histopatolgico. hospitalgeneral.salud.gob.mx/

BIBLIOGRAFIA: 1. Snchez BC. Tratado de Enfermedades de la glndula mamaria. Ed. Manual Moderno. Cap. 13- 15. 2.- De Vita V. Cancer of the Breast. In Cancer: Principles and Practice of Oncology: Fifth Ed. Philadelphia: Lippincott-Raven, Chapter 36; pp: 1521-1616. 3.-Consenso Nacional Acerca del Tratamiento de Cncer de Mama. En Tumores de mama: Diagnstico y Tratamiento. 2 Ed. McGraw-Hill Interamericana; pp: 119-126. 4.-Eberlein T. Current management of carcinoma of the breast. Ann Surgery 1994; 220: 121-136. 5. Encyclopedie Medico. Chirurgicale Praxis Medica, Editions Techiques de Mexico, tomo 5, ao 2005.

7.- Femenino de 42 aos, gesta- 5, partos-3, abortos-1, con diagnstico de anemia ferropnica, de 9.5 g/dl, refiere ciclos menstruales de 31,32 x 8,9 das de duracin, acompaados de cogulos, los cuales aparecieron despus del nacimiento de su segundo hijo hace 13 aos. E.F.: Buen estado general, TA 130/80, genitales con evidencia de sangrado activo, al tacto vaginal se detecta tero de consistencia firme voluminoso, irregular, aproximadamente de 12 cm. anexos libres. En esta paciente el diagnstico ms probable es:

a) Adenomiosis uterina. b) Cncer cervicouterino. c) Miomatosis uterina. d) Hiperpalsia adenomatosa de endometrio.

MIOMATOSIS UTERINA Definicin: Tumor benigno que se origina en el miometrio, por lo que su componente histolgico predominante es el tejido muscular y, en menor medida, el conectivo y fibroso. El nico tratamiento efectivo es el quirrgico; sin embargo, slo requieren ser tratados aquellos que producen sntomas. Evaluacin y Diagnstico: Historia: 1. 2. 3. 4. El sntoma ms frecuente suele ser la hemorragia uterina. Los sntomas principales estn relacionados con el crecimiento del tumor. La paciente puede notar una masa en hipogastrio o abdomen inferior. La masa se puede asociar a dolor plvico, o manifestaciones por compresin de rganos o estructuras vecinas. 5. Puede haber alteracin de la fertilidad.

Examen Fsico: 1. Se debe realizar con la vejiga y el recto vacuo. 2. El hallazgo primordial es el aumento de volumen y consistencia del tero, el cual puede ser simtrico (ndulos submucosos) o irregular (ndulos intramurales o subserosos). Exmenes Auxiliares: 1. 2. 3. 4. Papanicolau crvicovaginal: Indicado siempre; permite descartar neoplasia epitelial cervical o cncer infiltrante de crvix. Ultrasonido transabdominal y transvaginal: Indicado siempre; permite evaluar la localizacin, tamao y nmero aproximado de miomas. Hemoglobina, hematocrito: Indicado cuando hay historia de sangrado; orienta en la severidad del sangrado y anemia. Hemograma y VSG: Indicado cuando hay historia de fiebre; si es anormal sugiere infeccin o necrosis del mioma (puede ser apropiado descartar infeccin de otro rgano o sistema). Grupo sanguneo y factor Rh: Si hay anemia severa o en el preoperatorio. Perfil de coagulacin (tiempo de protrombina y de tromboplastina parcial, recuento de plaquetas): Si hay historia de sangrado exagerado. Gonadotrofina corinica (subunidad srica): Permite descartar posible embarazo en casos de ciclos irregulares, retraso menstrual o tero de consistencia blanda.

5. 6. 7.

Diagnstico Diferencial: 1. 2. 3. 4. 5. 6. Embarazo. Tumor de ovario. Enfermedad inflamatoria plvica, complejo inflamatorio anexial plvico. Endometriosis. Adenomiosis. Tumor extragenital: colon, retroperitoneo.

Referencias Bibliogrficas: 1. Hillard PA. Benign Diseases of the Female Reproductive Tract: Symptoms and Signs. En: Berek JS, Adashi EY, Hillard PA, eds. Novak's Gynecology. Baltimore: Williams and Wilkins, 1996:331-97. Hutchins FL, Greenber MD. Miomas Uterinos: Diagnstico e Indicaciones de Tratamiento. Clinicas de Ginecologa y Obstetricia. Temas Actuales. 1995;5:609-14. Davis KM, Sclass WD. Tratamiento Mdico para Miomatosis Uterina. Clinicas de Ginecologa y Obstetricia. Temas Actuales. 1995;5:671-81. Selwyn P, Oskowitz MB. Leiomyomata Uteri. En: Friedman EA, ed. Gynecological Decision Making. St. Louis: Mosby, 1983:148-9. Diaz Huamn V. Tumores Benignos del Aparato Reproductor Femenino. En: Ludmir A, Cervantes R, Castellano C, eds. Ginecologa y Obstetricia, Prevencin Diagnstico - Tratamiento. Lima: Concytec, 1996:907-25.

2. 3. 4. 5.

8.- Femenino de 34 aos con antecedentes patolgicos de hipertensin arterial sistmica de 4 aos de evolucin, bien controlada tratada con IECAS , actualmente cursa con 7 semanas de gestacin , signos vitales dentro del parmetro normal y exmenes de laboratorio sin alteraciones, se refiere asintomtica , la conducta ms adecueda a seguir es:

a) Mantener el tratamiento y asociar alfametildopa para disminuir los riesgos fetales de los IECAs. b) Mantener el tratamiento y asociar hidralacina para disminuir los riesgos maternos de los IECAs. c) Mantener el tratamiento dado el buen control tensional. d) Suspender los IECAs dado el riesgo que presentan para el feto.

El uso de IECA y ARAII durante el segundo y tercer trimestre de embarazo est contraindicado, debido a que estos medicamentos inducen toxicidad fetal (descenso de la funcin renal, oligohidramnios, retraso en la osificacin del crneo) y toxicidad neonatal (insufi ciencia renal, hipotensin, hiperpotasemia). En cuanto a su uso durante el primer trimestre de embarazo, un estudio publicado en el ao 2006, 1 mostraba un incremento de la incidencia de malformaciones congnitas, en particular malformaciones cardiacas, en nios nacidos de madres expuestas a IECA durante el primer trimestre de embarazo en comparacin con las mujeres que no recibieron tratamiento antihipertensivo o que recibieron tratamiento con otros medicamentos antihipertensivos. Estudios posteriores realizados no han confirmado a da de hoy los resultados de este estudio. En lo referente a los ARAII, no se dispone de estudios epidemiolgicos analticos apropiados, por lo que no se puede descartar que exista el mismo riesgo que para los IECA. A pesar de estas incertidumbres, el Comit de Medicamentos de Uso Humano (CHMP) de la Agencia Europea de Medicamentos (EMEA) ha recomendado prudencialmente evitar el uso

de IECA y ARAII durante el primer trimestre del embarazo.

Cooper WO et al. Major congenital malformations after fi rst-trimester exposure to ACE inhibitors. N Engl J Med 2006; 354 (23): 243- 51. ref.: 2008/10, junio.

9.- Femenino de 23 aos, G1 en trabajo de parto prematuro con embarazo de 30 semanas de gestacin. A pesar del uso de agentes tocolticos, estos no han dado resultado. La induccin para la maduracin pulmonar del producto se debe realizar por medio de: a) Betametasona b) Sulfato de magnesio c) Hidroxiprogesterona d) Clorprocana

La utilizacin de betametasona como inductor de madurez pulmonar fetal (IMPF) disminuye la morbilidad neonatal relacionada con prematurez pero su efecto diabetgeno materno ha sido poco estudiado. La revisin Cochrane de un ciclo nico de corticosteroides se actualiz en 2006. En esta actualizacin se incluyeron 21 estudios con un total de 3885 mujeres y 4269 lactantes.

En la revisin se descubri que la administracin de determinados corticosteroides a mujeres con riesgo de tener un parto prematuro reduce considerable los riesgos de complicaciones relacionadas con la prematurez como muerte fetal y neonatal combinada, sndrome de dificultad respiratoria, hemorragia cerebroventricular, enterocolitis necrotizante, infecciones sistmicas y retraso en el desarrollo durante la niez. Los beneficios estaban presentes cuando el tratamiento se iniciaba entre las 26 y las 35 semanas de gestacin y en los nios que nacan entre 1 y 7 das despus de haber comenzado el tratamiento; tambin se observaron beneficios en los subgrupos de mujeres con rotura prematura de membranas y trastornos hipertensivos. La muerte fetal y neonatal combinada se redujo incluso en neonatos que nacieron a menos de las 24 horas de haber administrado la primera dosis. No se demostraron beneficios cuando el tratamiento comenz antes de las 26 semanas de gestacin, tampoco se observaron beneficios en los recin nacidos antes de las 26 semanas de gestacin ni en los que nacieron despus de 7 das o ms de la administracin del tratamiento. En el caso de los neonatos que nacieron despus de las 36 semanas hubo una tendencia a aumentar la muerte fetal y neonatal combinada. Se observ una reduccin en el peso al nacer en los neonatos que nacieron entre los das 1 y 7, al igual que en los que nacieron ms de 7 das despus del primer tratamiento. Un estudio que reclut mujeres con preeclampsia severa sugiri que las mujeres tratadas tenan un mayor riesgo de sufrir diabetes gestacional. La evidencia epidemiolgica y en animales sugiere que pueden haber efectos adversos a largo plazo por la exposicin prenatal a los corticosteroides, entre ellos la alteracin de la tolerancia a la glucosa y la hipertensin. Los estudios en animales tambin han sugerido que afecta el crecimiento del cerebro. 1. 2. National Institute of Health (NIH). Consensus Conference; Effect of corticosteroide for fetal maturation on perinatal outcomes. JAMA 1994;(12):1-19. White A, Marcucci G, Andrews E, Edwards K. Antenatal steroids and neonatal outcomes in controlled clinical trials of surfactant replacement. Am J Obstet Gynecol 1995; (173):286-90. Klauss MH, Fanaroff AA, Martin RJ. Problemas respiratorios. En: Asistencia del recin nacido de alto riesgo. 2 ed. La Habana: Editorial Cientfico-Tcnica, 1981:194. Avery M, Frank N, Gribetz I. The inflationary force produced by pulmonary vascular distention in excised lungs. The possible relation of this force to that needed to inflatc the lungs at birth. J Clin Invest 1959;38:456. Chu J, Clements J, Cotton E. Neonatal pulmonary ischemia. Pediatrics 1965;40:733.

3.

4.

5.

6. Liggins GC, Howle RN. A controlled trial of antepartum glucocorticoid treatment for prevention of respiratory distress syndrome in premature infants. Pediatrics 1972;50: 515-25. 7. Wright LL, Verter J, Younes N. Antenatal corticosteroids administration and neonatal outcome in infants 501 to 1500 g. Am J Obstet Gynecol 1995; (173):263.

10.- Femenino de 31 aos se enva de alta con diagnstico de enfermedad inflamatoria plvica, regresa a los 15 das con temperatura de 38.5 c, mal estado general y datos de irritacin peritoneal, El diagnstico ms probable es?

a) b) c) d)

Endometritis Absceso tubo-ovrico Hidrosalpinx Ooforitis aguda

La enfermedad inflamatoria plvica (EIPA) es un sndrome clnico caracterizado por la infeccin del tracto genital superior que se produce casi siempre por va ascendente desde el cuello uterino. El impacto que la infeccin plvica ejerce sobre la condicin fsica de la mujer va desde la infeccin asintomtica o silente a una mayor morbilidad que en algunos casos puede llegar hasta la muerte. Incluye una variedad de condiciones inflamatorias que afectan el tracto genital superior. Los Centros de Control de Enfermedades (C .D. E.) la definen como un sndrome agudo debido al ascenso de microorganismos de la vagina o el cuello uterino al endometrio, trompas uterinas y en ocasiones a las estructuras vecinas (ovarios, peritoneo y cavidad pelvianas). En el momento actual se incluyen como principales agentes etiolgicos de la E.I.P.A la Neisseria gonorrhedae, las clamydias y los anaerobios. Otros microorganismos como los microplasmas y los actinomices se estn observando con frecuencia.

La presencia de anaerobios as como de bacterias aerobias puede deberse a un fenmeno de sobre infeccin secundaria. Hay autores que sealan que excepto para el gonococo y la Clamydia trachormatis, no existen datos suficientes que permitan afirmar que otras bacterias tengan un papel primario en la infeccin de unas trompas sanas. Una vez alterada la integridad anatmica de la trompa, se producira la infeccin mixta o poli microbiana.

El absceso tubo ovrico es una formacin inflamatoria que compromete el ovario y la trompa y puede ser uni o bilateral. En este absceso las estructuras comprometidas estn infectadas y contienen pus. Este proceso inflamatorio es secundario a un proceso infeccioso de la pelvis, habitualmente producido por grmenes muy patgenos, que llegan al tracto genital a travs de una relacin sexual, es decir corresponde a una complicacin severa de una enfermedad de transmisin sexual. Se caracteriza por aumento de volumen del ovario y trompa, los que se encuentran adheridos entre s producto de esta infeccin, adems el proceso infeccioso se extiende habitualmente a otras estructuras y rganos pelvianos, los que estn muy inflamados y adheridos formando lo que se denomina plastrn. El tratamiento se inicia mdicamente con antibiticos de amplio espectro para cubrir tanto grmenes aerbicos como anaerbicos, generalmente requiere de hospitalizacin para iniciar una terapia agresiva endovenosa con los antibiticos y para monitorizar adecuadamente a la paciente, pues la infeccin produce compromiso del estado general pudiendo llegar hasta la sepsis generalizada.

Diagnstico El cuadro clnico se sospecha cunado una paciente consulta por dolor abdominal intenso, progresivo, fiebre y compromiso de su estado general, habitualmente en el examen se encuentra un distensin abdominal y a la palpacin del abdomen hay dolor, y signo de blumberg positivo o irritacin peritoneal. El Tacto vaginal demuestra fondos de saco vaginales abombados y dolorosos y habitualmente el cuello del tero lateralizado y doloroso a la movilizacin si el compromiso es unilateral, adems de palpar una masa para uterina irregular y sensible.

BIBLIOGRAFA:

1. 2. 3. 4. 5. 6. 7. 8.

Botella Llusi,J.Clavero Nez,J.A:Tratado de Ginecologa.14 edicin. Ed.Diaz de Santos.pg 833-844.Madrid,1993. Brunham,R,C:Infectionin woman and ectopic pregnancy. Am J Obstet Gynecol.67:722,1999. Cates,W,Wasserheit,J,N:Genital Infection Epidemiology and sequeale.Am J Obstet Gynecol 164-1771,1998. Keit,L,G; Berger,G,S:On the causation of pelvic inflammatory disease. Am J Obstet Gynecol 149-215,2002. Muller,B,R;Allen,J,et al.Pelvic Inflamatory disease after histerosalpingography.Brit J Obstet Gynecol,91-1181,1999. Toth,A,O Leary,W,M: Evidence of microbial transfer by espermatozoo.Am J O bstet Gynecol 59-556,2003. Varela,R,et col:Abceso Tuboovrico,Acta mdica Portuguesa ,p:537-542,Vol. 8,2001. Sopper,D,E:Pelvic Inflamatory disease.Infections disease.Clin of North America .831-840,vol 8;n 4.Dec 2003.

11.- Paciente femenino de 28 aos de edad con deseo de un embarazo, antecedentes de G3 A2 - P1 se le realiza una histerosalpingografa, se constata que existe un sndrome de Asherman. Ello significa que se trata de:

a) b) c) d)

tero bicorne Endometriosis en la trompa Sinequias uterinas Insuficiencia istmico cervical

El sndrome de Asherman es una enfermedad ginecolgica rara que se caracteriza por la presencia de sinequias (adherencias) intrauterinas que pueden ocasionar amenorrea (ausencia de perodos menstruales regulares) e infertilidad.

En 1894 Heinrich Fritsch describe por primera vez la presencia de sinequias intrauterinas de tipo postraumtico, en una paciente que desarroll una amenorrea secundaria a un curetaje. Posteriormente en 1927 Bass inform de veinte casos de atresia (oclusin de una abertura natural) cervical tras abortos inducidos, pero no fue hasta 1948, cuando Joseph

G. Asherman recopil la informacin hasta entonces existente y acu el nombre con el que se conoce actualmente a la enfermedad.

Asherman describi originalmente dos tipos diferentes de amenorrea secundaria, en funcin de su etiologa (estudio de las causas de las enfermedades): la amenorrea traumtica atrtica, debida a estenosis del orificio cervical interno y la amenorrea debida a adherencias intrauterinas. Posteriormente ambas entidades se agruparon en una nica entidad bajo el nombre de sndrome de Asherman.

Suele presentarse en mayor proporcin tras dilataciones y curetajes uterinos de repeticin y sobre todo si se realizan durante el embarazo o si existe infeccin uterina en el momento en el que se realizan estas intervenciones.

Las adherencias intrauterinas pueden producirse debido a cualquier factor que lleve a una destruccin de las paredes del miometrio (capa muscular de la pared del tero). Sin embargo, hay que distinguir entre factores predisponentes, siendo el principal de ellos el embarazo y factores causales, entre los que se encuentran: traumatismos uterinos, intervenciones quirrgicas que afecten al tero, agentes fsicos o qumicos e infecciones uterinas por tuberculosis o esquistosomiasis. En cualquier caso, el factor ms importante es el trauma uterino en el momento del parto o el puerperio.

El cuadro clnico es muy variable y las manifestaciones clnicas varan con el grado de oclusin de la cavidad uterina y la severidad de las adherencias, pudiendo presentarse: esterilidad cuando la oclusin de la cavidad uterina incluye porciones proximales (ms cerca de un centro, tronco o lnea media) de las trompas de Falopio o cuando las adherencias impiden la nidacin del huevo; las pacientes presentan con frecuencia amenorrea, oligomenorrea (disminucin de la frecuencia de las menstruaciones), dismenorrea (menstruacin dolorosa) y abortos repetidos.

Hysteroscopic treatment of severe Asherman's syndrome and subsequent fertility. Capella-Allouc S; Hum Reprod, 1999 May.

12.- Femenino de 37 aos, es atendida en consulta externa con reporte de papanicolaou que reporta un NIC I, la especuloscopa se observa crvix con ectropin periorificiario. El mtodo ms sensible para corroborar el diagnstico en esta paciente es: a) b) c) d) Papanicolaou. Exudado vaginal. Prueba de koh. Colposcopa.

9.5.2 Las pacientes a quienes se les realiz citologa cervical, cuyo resultado es LEIBG (infeccin por VPH, displasia leve o NIC 1); LEIAG (displasia moderada y grave o NIC 2 y 3) o cncer deben enviarse a una clnica de colposcopa, para realizar estudio colposcpico. 9.5.3 Si el resultado de la citologa es LEIBG, la colposcopa es satisfactoria y sin evidencia de LEIBG, se realizar control citolgico en un ao (Apndice Normativo A) 9.5.4 Si la citologa es de LEIBG, la colposcopa es satisfactoria y existe evidencia de lesin, se debe tomar una biopsia dirigida. 9.5.4.1 Si la biopsia dirigida es negativa, se realizar nueva colposcopa para verificar el diagnstico y en caso necesario, tomar nueva biopsia dirigida y revalorar. 9.5.4.2 Si la biopsia dirigida es reportada como LEIBG se podr dar tratamiento conservador: criociruga, electrociruga o laserterapia (slo si cumple con las condiciones referidas en el Apndice 1) o se podr mantener a la paciente en vigilancia en la clnica de colposcopa, con colposcopa y estudio citolgico cada seis meses, durante 24 meses. Jueves 31 de mayo de 2007 DIARIO OFICIAL (Primera Seccin) 9.5.4.3 Si la biopsia dirigida es reportada como LEIAG (Lesin Intraepitelial Escamosa de Alto Grado) se realizar tratamiento conservador (electrociruga o laserterapia). En las mujeres posmenopusicas, dependiendo de las condiciones anatmicas del crvix, se realizar tratamiento conservador en la clnica de colposcopa o tratamiento quirrgico (histerectoma extrafascial) en el servicio que corresponda. 9.5.4.4 Si la biopsia dirigida reporta cncer microinvasor o invasor, la paciente se transferir a un Servicio o Centro Oncolgico para su tratamiento correspondiente. 9.5.4.5 Si la citologa reporta LEIBG y la colposcopa es no satisfactoria, se tomar cepillado endocervical (Apndice Normativo A) 9.6 En caso de colposcopa no satisfactoria, negativa a LEIBG y con cepillado endocervical negativo, se continuar su control en la clnica de colposcopa en seis meses, con colposcopa y citologa. 9.6.1.1 Si el cepillado endocervical reporta LEIBG se tratar a la paciente como LEIAG, con mtodos conservadores escisionales.

Jueves 31 de mayo de 2007 DIARIO OFICIAL (Primera Seccin) Modificacin a la Norma Oficial Mexicana NOM-014-SSA2-1994, Para la prevencin, deteccin, diagnstico, tratamiento, control y vigilancia epidemiolgica del cncer crvico uterino. Al margen un sello con el Escudo Nacional, que dice: Estados Unidos Mexicanos.- Secretara de Salud. MODIFICACION A LA NORMA OFICIAL MEXICANA NOM-014-SSA2-1994, PARA LA PREVENCION, DETECCION, DIAGNOSTICO, TRATAMIENTO, CONTROL Y VIGILANCIA EPIDEMIOLOGICA DEL CANCER CERVICO UTERINO.

13.- Es el caso de paciente femenino de 17 aos la cual presenta amenorrea acompaada de profundas alteraciones del olfato. Cul de los siguientes diagnsticos es el ms probable?

a) Sndrome de amenorrea-galactorrea. b) Amenorrea de causa uterina. c) Sndrome de ovario poliqustico. d) Amenorrea por alteracin hipotalmica.

AMENORREA HIPOTALAMICA. (Hipogonadotrpica) Psicgena. (Stress emocional) Anorexia nerviosa. (Deficiencia nutricional) Ejercicio excesivo. (Carrera de fondo, natacin, gimnasia, ballet) Frmacos. (Fenotiazina, reserpina, bloqueadores ganglionares, anticonceptivos) Pseudocisis. Sndrome de Kallman (Deficiencia selectiva de gonadotropinas y anosmia). El ejemplo clsico de la alteracin hipotalmica que lleva a desrdenes del ciclo menstrual es el Sndrome de Kallman.

Lectura Recomendada: Etiopatogenia de la amenorrea hipotalmica funcional Interaccin de las respuestas hormonales del Sistema Nervioso Central y Neuropptidos Perifricos. Revista Argentina de Endocrinologa y Metabolismo Copyright 2008 por la Sociedad Argentina de Endocrinologa y Metabolismo Vol 45 No. 2.

14.- Femenino de 40 aos de edad G.3 P.1 C 2, la cual es diagnosticada por miomatosis uterina de pequeos y medianos elementos sintomticos, sus antecedentes refieren cirugas plvicas previas, el tratamiento de eleccin es: a) b) c) d) Progesterona. Histerectoma total abdominal. Observacin Anlogos de GnRH.

CUADRO CLINICO La miomatosis uterina muestra manifestaciones clnicas en menos del 50%, de estas las ms frecuentes son: 1. 2. 3. 4. 5. 6. Hemorragia uterina anormal. Dolor. Distensin abdominal. Compresin genitouterina. Compresin gastrointestinal. Compresin plvica.

10. DIAGNOSTICO El diagnstico se realiza a travs de imagenologa: 1-Ecografa. 2-TAC 3-Rayos X 4-Histeroscopia.

11. TRATAMIENTO La miomatosis uterina debe ser tratada cuando produzca cualquiera de las manifestaciones clnicas anotadas, toda paciente que se programe para histerectoma debe tener previamente legrado biopsia Lo podemos dividir en Conservador o Radical.

1.

CONSERVADOR:

Este tratamiento se puede instaurar en pacientes con deseo de preservar el tero. Igualmente se puede subdividir: -Expectante -Quirrgico: miomectoma -Medico Tratamiento expectante: Esta indicado en pacientes, cuyos sntomas son leves y no deseen o tengan alguna contraindicacin medica para tratamiento quirrgico. En ellas se recomienda controles clnicos y ecogrficos cada 6 meses a 1 ao. Tratamiento mdico. AINES Anlogos GnRH: Progestgenos: Andrgenos. Antiandrgenos

Tratamiento quirrgico: MIOMECTOMIA: 1. Criterios del ACOG en pacientes infecundas. Procedimientos: Va endoscpica: Laparoscopia: Miomas subserosos sesiles o pediculados < 5cm. Histeroscopia: miomas submucosos Va laparotoma Aquellos miomas que se salgan de las caractersticas anteriores. Indicaciones: Hemorragia anormal. Perdida reproductiva. Infecundidad. Dolor.

El tratamiento previo con anlogos esta indicado cuando se desee disminuir el tamao del mioma para prevenir sangrado quirrgico.

Contraindicaciones: Embarazo. Cncer endometrial. Infecciones. Dificultad tcnica.

2. Criterios del ACOG para Miomectoma en pacientes que desean conservar el tero. Procedimiento: Va endoscpica. Va abdominal. Va vaginal.

Indicacin:

a. b.

Presencia de uno o dos Leiomiomas asintomticos de tamao tal que se pueden palpar por va abdominal y constituyen una preocupacin para la paciente. Pacientes ovulatorias con miomas como posible causa de hemorragia uterina excesiva, demostrada por cualquiera de las siguientes circunstancias: Hemorragia profusa: de duracin mayor de 8 das. Anemia por prdida sangunea aguda o crnica.

2. RADICAL: HISTERECTOMIA Para pacientes post menopusicas, con paridad satisfecha o sin deseo de preservar el tero. Criterios del ACOG para Histerectoma por miomas. 1. Presencia de 1, 2, o 3 Miomas asintomticos de tamao tal que son palpables por va abdominal y preocupan a la paciente.

2. Hemorragia uterina excesiva. Duracin mayor de 8 das. Anemia por prdida sangunea aguda o crnica.

1.

Molestias plvicas producidas por los miomas: signos compresivos.

Contraindicaciones: 1. Deseo de conservar la fecundidad. 2. Miomas asintomticos. 3. Contraindicacin mdica o dificultades tcnicas para la ciruga.

Guarnaccia M. and Rein M. Traditional Surgical Approaches to Uterine Fibroids Abdominal. Myomectomy and Hysterectomy. Clinical Obstetrics and Gynecology 2001. 44.2. 385-400. - Milad. M and Sankpal R. Laparoscopic Approaches to Uterine Leiomyomas. Clinical Obstetrics and Gynecology. 2001. 44-2. 401-411. - Carlson K. et al. Indications for Hysterectomy. N. Engl. J. Med. 1993. 328(12) 56-

15.- En la sala de urgencias recibe a una paciente que inicia con convulsiones por preclampsia usted decide administrar el siguiente frmaco ya que es el de eleccin en sta patologa:

a) Sulfato de magnesio. b) Diacepam. c) Fenitona. d) Donadores de xido ntrico.

Manejo de la Preeclampsia 1. Manejo ambulatorio: HTA sin proteinuria significativa, se recomienda el reposo en cama. Monitoreo de TA, peso, presencia de protenas en orina. Ecografas peridicas para ver el feto y evaluar posibles retardo de crecimiento. 2. Manejo hospitalario: para mujeres con HTA inducida por el embarazo y 2+ o ms o proteinuria significativa y en quienes fall el manejo ambulatorio. 3. Laboratorio y evaluacin del peso: debe realizarse diariamente. Evaluacin de la dinmica fetal. Monitoreo de sntomas como cefalea, alteraciones visuales y dolor epigstrico. 4. El parto es el tratamiento de eleccin: el cual debe realizarse cuando el feto est maduro pero puede realizarse en forma temprana si la salud de la madre est en peligro o si hay evidencia de distress fetal. El parto est indicado cuando la paciente cumple con los criterios de preeclampsia severa. Betametasona 12.5 mg IM dos veces por da puede estimular la maduracin de los pulmones fetales.

5. Terapia antihipertensiva: est indicada slo si la TA es persistentemente > 160/110 , es importante disminuir la TA hasta una diastlica de 90 a 100 mmHg porque la presin normal podra resultar en hipoperfusin de la placenta. Los diurticos nunca estn indicados, estas pacientes ya son hipovolmicas. Los IECA no deben ser usados durante el embarazo. Las medicaciones de largo plazo, incluyen alfa metildopa, atenolol y labetalol. 6. Terapia anticonvulsivante: A- Profilaxis de las convulsiones: est indicada en todas las pacientes pre-eclmpticas durante el trabajo de parto y el parto y por un mnimo de 24 hs luego del mismo. Algunos mantienen la terapia con magnesio hasta que comienza la diuresis. El Sulfato de Magnesio es la droga de eleccin. La dosis profilctica es de 4 a 6 g de sulfato de magnesio IV y contina con 2 g c/ hora. B- Tratamiento de las convulsiones: Sulfato de Magnesio 1 g/min IV hasta controlar las convulsiones hasta un mximo de 4 a 6 g. El nivel teraputico es de 4 meq/l. Toxicidad del magnesio: ausencia de reflejo patelar, debilidad muscular, parlisis respiratoria y depresin cardaca, 10 ml al 10 % de gluconato de calcio puede ser administrada IV. La terapia con sulfato de magnesio contina por lo menos 24 horas en el post parto, la terapia puede detenerse si la excrecin urinaria es > 200 ml/h por cuatro horas consecutivas. C- Prevencin: 81 mg de aspirina diarios pueden ser administrados luego del primer trimestre en mujeres con hipertensin crnica o historia previa de preeclampsia, sin embargo la eficacia de esta indicacin ha sido cuestionada.

Bibliografa:
Myers JE, Baker PN. Hupertensive diseases and eclampsia. Curr Opin Obstet Gynecol 2002; 14: 119-125. 2. Tierney, McPhee, Papadakis. Diagnstico clnico y tratamiento 2003. 38 ed, Mxico, Manual Moderno, 2003: 770-773. 3. Wilson MI, Goodwin TM, Pan VI, Ingles SA. Molecular epidemiology of preeclampsia. Obstet and Gynecol Survey 2003; 58(1):39-66. 4. Burrow GM. Complicaciones mdicas durante el embarazo. 4 ed, Mxico, McGraw-Hill panamericana: 1996: 1-25. 5. Guyton AC, Hall JE. Embarazo y lactancia en: Tratado de fisiologa mdica, 10 ed, Mxico, McGraw-Hill Interamericana 2001: 1135-45. 6. Vaticon D. Fisiologa de la fecundacin, embarazo, parto y lactancia, en: Tresguerres JAF. Fisiologa Humana. Mxico, Interamericana McGraw-Hill, 1992: 1086-1109. 7. Pridjian G, Puschett JB. Preeclampisa. Part 1: Clinical and Considerations. Obstet and Gynecol Survey 2002; 57 (9): 598-618. Pathophysiologic

8. Pridjian G, Puschett JB. Preeclampisa. Part I1: Experimental and Genetic Considerations. Obstet and Gynecol Survey 2002; 57 (9): 619-40. 9. IMSS. Embarazo de alto riesgo. Gua diagnstica teraputica. Rev Med IMSS 1998; 36(1):45-60.

16.- An 18-year-old woman complains of myalgias, a sore throat, and painful mouth sores for 3 daysduration. Her temperature is 38.2 C (100.8 F), blood pressure is 110/80 mm Hg, pulse is 84/min, respirations are 15/min. Her gingival are edematous and erythematous, and there are vesicles on her right upper and lower lips. Her pharynx is mildly erythematous but without exudates, and there is tender mobile cervical lymphadenopathy. Her breath is not fetid, and the dentition is normal. Which of the following is the most likely causal agent?

a) Actinomyces israelii b) Herpes simplex virus 1 c) Nocardia asteroids d) Streptococcus pyogenes

Las infecciones por herpes simplex son comunes en la prctica diaria, y con frecuencia el paciente acude a los servicios de urgencia. Estas infecciones son ocasionalmente recurrentes, generalmente dolorosas y asociadas con sntomas sistmicos, por lo cual el mdico de urgencias debe estar familiarizado con el cuadro clnico y su manejo. Existen dos variedades de virus del Herpes simplex (VHS) capaces de causar infeccin en el hombre: el tipo 1 (VHS-1) y el tipo 2 (VHS-2) que se distinguen entre s por varias caractersticas, incluyendo sus comportamientos clnico y epidemiolgico, antigenicidad, composicin del ADN y la sensibilidad a diferentes agentes fsicos y qumicos (Cuadro No.1) Cuadro No. 1 DIFERENCIAS ENTRE LOS VIRUS HERPES SIMPLEX TIPOS 1 Y 2 Caractersticas clnicas VHS-1 VHS-2 Genital Genita PerianaL Neonata Paroniquia l VHS-2 especfico

Va de transmisin Oral Sndromes Oral-facial caractersticos Ocular Encefalitis Paroniquia Reactividad a antgenos VHS-1 especfico monoclonales especficos

El VHS-1 es de localizacin primordialmente extragenital, con predileccin por los tejidos de origen ectodrmico, mientras que el VHS-2 corresponde al "Herpes progenitalis" descrito por separado, dentro de las infecciones de transmisin sexual. La regin oral es la localizacin habitual del Herpes simplex 1, el cual es causa frecuente de lesiones orofaciales recurrentes y de otro tipo de enfermedades (encefalitis). EPIDEMIOLOGIA El ser humano es el nico reservorio natural conocido del virus herpes simplex, aunque algunos animales de experimentacin pueden infectarse con facilidad. La infeccin primaria del VHS-1 ocurre sobre todo durante la infancia, mientras que el tipo 2 se presenta en la adolescencia y adultos jvenes activos sexualmente. Las tasas de infeccin son inversamente proporcionales al estrato socioeconmico. El principal mecanismo de transmisin es el contacto directo con las secreciones infectadas. El VHS-1 se trasmite por saliva y el VHS-2 por va genital. Aunque los ttulos virales son ms altos cuando existen lesiones activas, tambin es frecuente la liberacin viral en infectados asintomticos. Por lo tanto, la transmisin viral puede efectuarse an en ausencia de lesiones activas. La persistencia de la infeccin y la recurrencia de las lesiones son un fenmeno frecuente tanto para el VHS-1 como para el VHS-2 y por lo comn se producen por reactivacin endgena. Los factores que la precipitan van desde la luz solar, el viento, traumatismos locales, fiebre, menstruaciones y hasta estrs emocional.

DIAGNOSTICO Cuadro Clnico. Los cuadros clnicos causados por este virus se suelen dividir en dos grupos: el debido a la infeccin primaria y el correspondiente a la infeccin recurrente. En el primer grupo se incluyen la gingivoestomatitis aguda, la vulvovaginitis aguda y la infeccin herptica del ojo, que puede llegar a queratitis. Las recurrentes se circunscriben al "Herpes labialis", queratitis, blefaritis y queratoconjuntivitis. Todos los cuadros son autolimitados, pero tanto las formas primarias como las recurrentes, se pueden complicar. Una de estas complicaciones es la Encefalitis herptica y el Eczema herpeticum. Infeccin primaria. El primer contacto clnico de infeccin por virus del herpes simple suele ser el ms grave. Los enfermos aquejan fiebre, malestar general, artralgias y por ltimo la presencia de un grupo de vesculas sobre una base eritematosa, dolorosa, inflamada y sensible. La gingivoestomatitis es la manifestacin ms comn, cuya gravedad vara desde la erosin de pequeas reas a la ulceracin extensa de la boca, lengua y encas. La infeccin puede ser bastante grave como para dificultar la ingesta de alimentos y

lquidos (odinofagia). La curacin tiene lugar en 7 a 14 das, a menos que las lesiones se sobreinfecten con estafilocos o estreptococos. Infeccin recurrente. Generalmente existe prurito, dolor o molestias focales que preceden la aparicin de las vesculas. Las vesculas se rompen espontneamente despus de unos cuantos das y sanan en una semana sin dejar secuelas. LECTURAS RECOMENDADAS 1. Callen JP, Cooper Ma. Dermatologic emergences. Emerg Med. Clin North Am 3:641, 1985 2. Guzmn M. Herpes simple, varicela zoster. En: Medicina Interna. Segunda Edicin. Editado por F Chalem, JE Escandn, J Campos, R Esguerra. Fundacin Instituto de Reumatologa e Inmunologa. Editorial Presencia Ltda. Santaf de Bogot, 1992 3. Guerra Flecha J, Lizarraga Bonelli S. Enfermedades de transmisin sexual: herpes genital Trib Med 79:29, 1989 Jaramillo AC. Infecciones virales de la piel y sus anexos. En: Fundamentos de Medicina. Enfermedades Infecciosas. Cuarta edicin. Corporacin para Investigaciones Biolgicas. CIB. Medelln, 1989.

17.- Masculino de 33 aos, acude al servicio de urgencias por referir dolor en pabelln auricular izquierdo, dificultad para mover la hemicara del mismo lado. Antecedentes: sin importancia. Exploracin fsica: TA 120/80mmHg, FC 72 x, FR 16 x, Temp 36 , se observan vesculas en concha auricular y paresia de la hemicara izquierda. El agente causal ms probable en este caso es:

a) b) c) d)

Haemophylus influenzae Herpes virus Virus del Papiloma Humano Epstein Barr

Sndrome de Ramsay-Hunt: Representa 6.8% de las causas de parlisis faciales intratemporales. La enfermedad se presenta en adultos entre 20 y 30 aos y 50 a 70 aos de edad. Se caracteriza por la presencia de vesculas herpticas en pabelln auricular, conducto auditivo externo y, con menor frecuencia paladar blando y cara. Por lo general se inicia con otalgia intensa seguida en pocos das de la erupcin vesicular y parlisis facial perifrica; adems puede haber hipoacusia, acfenos y vrtigo. Escajadillo J, Odos, narz, garganta y ciruga de cabeza y cuello, Ed. Manual Moderno, 2 Edicin, Pg. 152

18.- Masculino de 38 aos diagnosticado por trastorno obsesivo-compulsivo. Los actos obsesivos ms frecuentes en este trastorno son: a) b) c) d) Recuentos mentales. Evitar pisar las cruces de las baldosas. Acumulacin y coleccin de objetos. Comprobaciones y rituales de limpieza.

Sntomas del TOC Obsesiones

Las obsesiones son ideas o impulsos no deseados que aparecen repetidamente en la mente de la persona que padece TOC. Los pacientes suelen tener miedo a sufrir dao ellos mismos, o alguien al que quieren, se preocupan irracionalmente por no contaminarse, o tienen una necesidad excesiva de hacer las cosas correctamente o con perfeccin. Una y otra vez, la persona piensa algo que le inquieta, como por ejemplo, "Mis manos pueden estar contaminadas--debo lavarlas" o "Puedo haber dejado el gas abierto" o "Estoy hacindole dao a mi hijo." Estos pensamientos angustiosos se inmiscuyen con los otros pensamientos del paciente, y causan ansiedad. A veces, las obsesiones son de carcter violento o sexual, o tienen que ver con enfermedades. Compulsiones En respuesta a sus obsesiones, la mayora de las personas con TOC recurren a comportamientos repetitivos llamados compulsiones. Los ms frecuentes son los rituales de limpieza y las comprobaciones. Otros comportamientos compulsivos incluyen recuentos (a menudo al mismo tiempo que se realizan otras acciones compulsivas, tales como lavarse las manos), hacer repeticiones, acaparamiento, y recolocaciones sin fin de objetos en un esfuerzo para mantenerlos perfectamente alineados. Tambin son corrientes los problemas mentales, tales como repetir frases mentalmente y hacer listas. Estos comportamientos, en general, tienen por objeto proteger de peligros a la persona que padece TOC, o a los otros. Algunas personas con TOC tienen rituales establecidos; otros tienen rituales que son complejos y cambiantes. El ejecutar estos rituales slo proporciona alivio temporal de la ansiedad, pero no hacerlos incrementa la ansiedad de la persona.

19.- El estudio donde se toman un grupo de personas sanas que se clasifican en subgrupos segn su exposicin a una causa o enfermedad se conoce como: a) b) c) d) Cohorte Prospectivo Observacional Doble ciego

En los estudios de cohorte se eligen dos grupos uno de expuesto y otro de no expuestos los cuales son seguidos a travs del tiempo para detectar las posibles consecuencias. Posteriormente se analiza la informacin calculado la incidencia en el grupo de expuestos y en el grupo de no expuestos; y una vez obtenidos se calcula el Riesgo Relativo.

Ruiz M. A. Epidemiologa Clnica, Panamericana, 1. Ed. 2004; pgs: 287-289

20.- Debemos sospechar un retinoblastoma en un nio que presenta los siguientes sntomas: a) Dolor, fotofobia y lagrimeo. b) Lagrimeo, fotofobia y aumento del dimetro corneal. c) Fotofobia y quemosis conjuntival. d) Estrabismo y leucoria.

RETINOBLASTOMA TUMOR OCULAR MS FRECUENTE EN INFANCIA. 1/20.000 RN 80% en < 3 aos Uni o bilaterales. Herencia 1. AD alta penetrancia (90-95%) 2. Espordicos

Diagnstico: leucocoria estrabismo mala AV ojo rojo y doloroso celulitis orbitaria examen de rutina

Annals d.Oftalmologia 2001;9(2):74-92 N. Martn, MD. Coll, J. Garca, J. Snchez de Toledo, E. Trivio, M. Guitart, JJ. Gil. 1Unidad Oftalmologa Peditrica. Hospital Maternoinfantil Vall d.Hebron 2Departamento de biologa-celular, fisiologa e inmunologa de la Universidad Autnoma de Barcelona 3Servicio oftalmologa Hospital General Vall d.Hebron

21.- Se trata de paciente de 34 aos que cursa con 39 SDG; a la exploracin fsica reflejos patelares hiperactivos, inquieta, se reportan cifras de TA 145/95, se realiza laboratorio que reporta proteinuria 2+,. El tratamiento mas adecuado para esta paciente es: a) Reposo en cama b) Sulfato de magnesio oral d) Propranolol e) Inducir el trabajo de parto

Interrupcin del embarazo La mayor parte de las guas de prctica clnica que contestan esta pregunta se basan en estudios retrospectivos o recomendaciones de los comits de expertos (Nivel III/IV). La interrupcin del embarazo se considera la mejor opcin de tratamiento para la preeclampsia y, sin duda, lo es para la embarazada al prevenir la preeclampsia severa y la eclampsia. Sin embargo, para el feto no siempre es la mejor opcin. Dos estudios observacionales retrospectivos mostraron que a las 37 semanas la terminacin del embarazo es la mejor opcin, tomando en cuenta que si existen condiciones cervicales favorables, la induccin del parto es una via segura y de bajo riesgo para el feto (Nivel III). En embarazos pretrmino debe considerarse la prolongacin del embarazo hasta alcanzar el mayor peso y madurez fetal. Atencin conservadora de pacientes con preeclampsia leve Cuando el embarazo cursa entre las 28 y 34 semanas, el manejo conservador puede reducir las complicaciones propias de la prematuridad. La decisin debe basarse en la estabilidad del estado materno y fetal, as como en la capacidad del centro hospitalario para brindar atencin optima a un recin nacido prematuro (Nivel III). Durante el periodo del monitoreo ambulatorio se indica a la paciente que se realice en forma externa una medicin diaria de la presin arterial, y que acuda semanalmente a la consulta para descartar evolucin o agravamiento de la enfermedad (Nivel IV). No existen estudios controlados que hayan determinado la magnitudde los riesgos maternos y fetales durante el tratamiento conservador de la paciente con preeclampsia leve. Recomendaciones La paciente con preeclampsia leve, seleccionada y clasificada despus de su estancia hospitalaria, puede atenderse en forma ambulatoria, con vigilancia semanal en la consulta, como una medida efectiva y segura.

(Grado de recomendacin B)
En las pacientes con embarazo 37 semanas y preeclampsia leve, la terminacin del embarazo es la mejor opcin, valorando la induccin del parto como una medida de bajo riesgo para el feto.

(Grado de recomendacin C)
La atencin conservadora del embarazo menor de 34 semanas, complicado por preclampsia leve, mejora el pronstico del feto.

(Grado de recomendacin C)

Ginecol Obstet Mex 2010;78(6):S461-S525 Guas de prctica clnica.

Diagnstico y tratamiento de la preeclampsia-eclampsia Fecha de busqueda de informacin: marzo 2009 Fecha de elaboracion: Septiembre 2009 Fecha de actualizacion: Junio 2012 Institucin responsable: Colegio Mexicano de Especialistas en Ginecologa y Obstetricia. Coordinador del grupo Dra. Maria Teresa Leis Marquez

Especialista en Ginecologa y Obstetricia y Medicina Materno Fetal. Certificada por el Consejo Mexicano de Ginecologa y Obstetricia. Jefa de la Clnica de Medicina Materno Fetal del Hospital ngeles Lomas. Coordinadora del Comit para la elaboracin de las Guas de Prctica Clnica del Colegio Mexicano de Especialistas en Ginecologa y Obstetricia, periodo 2008-2010. Miembro de la Junta de Gobierno y del Comit de Exmenes del Consejo Mexicano de Ginecologa y Obstetricia. Vicepresidenta para Amrica del Norte de la Sociedad Iberoamericana de Diagnstico y Tratamiento Prenatal.

22.- Femenino de 32 aos segunda gesta a trmino sin anormalidades en el transcurso de ste. Inicia trabajo de parto de forma espontnea, con evolucin normal hasta que se rompe la bolsa, con una dilatacin de 4 cm. A partir de entonces, comienza con hemorragia de sangre roja, en moderada cantidad y aparecen signos de sufrimiento fetal agudo. El estado general de la mujer es bueno y la dinmica uterina es normal. Este cuadro corresponde a:

a) b) c) d)

Abruptio placentae. Rotura de vasa previa. Rotura uterina. Placenta previa central

La vasa previa es una condicin de alto riesgo obsttrico en la cual vasos fetales o placentarios cruzan el segmento uterino por debajo de presentacin. Estos vasos estn desprotegidos de gelatina de Wharton o tejido placentario, lo que los hace altamente vulnerables y susceptibles de ruptura o laceraciones en cualquier perodo del embarazo, principalmente en el momento del parto. Tambin es frecuente la compresin de estos vasos, especialmente durante el tercer trimestre de la gestacin, lo que puede condicionar asfixia y muerte fetal.

Esta condicin ocurre como resultado de que vasos velamentosos cruzan por el segmento uterino debido a una insercin velamentosa del cordn, situacin en la cual el cordn umbilical se inserta en las membranas ovulares en vez del tejido placentario (vasa previa tipo I), o por el cruce de vasos fetalesentre uno o ms lbulos accesorios de la placenta (vasa previa tipo II) (Figura 1).

Figura 1. A, Vasa previa tipo I, debida a insercin velamentosa. B, Vasa previa tipo II, debida a cotiledones aberrrantes. Reproducido de Daly-Jones y cols. Ultrasound 2008.

Vasa previa se puede presentar si existe alguna (o ninguna) de las siguientes condiciones: placenta baja (que puede ser causa de abortos previos seguidos por legrado o por operaciones uterinas, que provocan cicatrices en el tero), placenta bilobada o de lbulo succensuriado, embarazos resultado de fertilizacin in vitro, o embarazos mltiples (5-6). El sangrado por vasa previa no es doloroso. Otros sangrados por complicaciones o por nacimiento no necesariamente son sin dolor.

Referencias: 1. Oyalese Y, Smulian JC. Placenta previa, placenta acreta, and vasa previa. Obstet Gynecol 2006; 107: 927-941. 2. Oyalesse KO, Turner M, Less C, Campbell S. Vasa previa: an avoidable obstetric tragedy. Obstet Gynecol Surv 1999; 54: 138-145. 3. Seplveda W, Sebire NJ, Harris R, Nyberg DA. The placenta, umbilical cord, and membranas. In Diagnostic Imaging of Fetal Anomalies, Nyberg DA, MaGahan JP, Pretorius DH, Pilu G (eds). Philadelphia, PA: Lippicont Williams & Wilkins 2003, 85-132. 4. Daly-Jones E, John A, Leahy A, McKenna C, Sepulveda W. Vasa praevia; a preventable tragedy. Ultrasound 2006; 16: 8-14. 5. Derbala Y, Grochal F, Jeanty P. Vasa previa. J Prenat Med 2007; 1: 2-13. 6. Fung TY, Lau TK. Poor perinatal outcome associated with vasa previa. It is preventable? A report of three cases and review of the literature. Ultrasound Obstet Gynecol 1998; 12: 430-433. 7. Robert JA, Sepulveda W. Fetal exsanguination from ruptured vasa previa: still a catastrophic event in modern obstetrics. J Obstet Gynaecol 2003; 23: 574. 8. Cordero DR, Helfgott AW, Landy HJ, et al. A non-hemorrhagic manifestation of vasa previa: a clinicopathologic case report. Obstet Gynecol 1993; 82: 698-700. 9. Schachter M, Tovbin Y, Arieli S, et al. In vitro fertilization as a risk factor for vasa previa. Fertil Steril 2002; 78: 642-643.

23.- Recibe usted los resultados histopatolgicos de una paciente de 24 aos de edad que acudi a revisin rutinaria, los resultados reportan imagen histolgica de coilocitos lo cual sugiere infeccin por:

a) b) c) d)

Herpes virus tipo 2 Citomegalovirus Vaginosis bacteriana. Virus del papiloma humano

El coilocito es un tipo de clula hallada en lesiones precancerosas cervicales. Tambin es comn apreciarla microscpicamente en lesiones reaccionales en la mucosa oral, debido a su similitud con la mucosa vaginal, en enfermedades como Papilomas, o en Condiloma acuminado. El coilocito es la manifestacin clsica de la infeccin por VPH en la clula. Fue descrito por primera vez por Koss y Durfee en 1956. Esta clula tambin ha sido llamada clula en baln.

El coilocito es una clula epitelial escamosa, ms comnmente superficial e intermedia, aunque tambin puede verse en clulas parabasales y metaplsicas. Esta clula presenta cambios tpicos tanto en su ncleo como en su citoplasma, pierde los bordes angulados usuales de la clula escamosa superficial y su forma tiende a ser redondeada y ovoide. El citoplasma muestra una condensacin perifrica que le da un aspecto en asa de alambre, es opaco, denso y de aspecto creo, anfoflico, acidoflico o de color rojo/naranja brillante. Adems se observa una gran cavidad o halo con un margen muy bien definido, de forma oval o ligeramente festoneado. El ncleo de la clula se localiza de manera excntrica, lo que lo convierte en un halo paranuclear, no perinuclear. Ocasionalmente puede encontrarse material fagocitado dentro del espacio coiloctico.

Lesin Intraepitelial de Bajo Grado. Alteraciones Celulares compatibles con Infeccin (Coilocitos). Schlecht, N.F., Kulaga, S., Robitaille, J., Ferreira, S., Santos, M., Miyamura, R.A., Duarte-Franco, E., Rohan, T.E., Ferenczy, A., Villa, L.L., & Franco, E.L. (2002) Persistent Human Papillomavirus Infection as a Predictor of Cervical Intraepithelial Neoplasia. JAMA, 286, 3106-3114

24.- Mujer de 24 aos de edad atendida de parto eutcico en tocociruga , al encontrase reparando episiotoma media, aprecia un marcado incremento en el sangrado transvaginal. La causa ms probable de este fenmeno es: a) Retencin de restos placentarios b) Laceracin vaginal c) Laceracin cervical d) Atona uterina

La mortalidad materna es un indicador de disparidad social y econmica. Cada ao en todo el mundo mueren cerca de 600,000 mujeres, entre 15 y 49 aos. Como resultado de complicaciones relacionadas con el embarazo, el parto y el puerperio. Entre las causas principales destacan: hemorragia postparto (25 %), se psis (15%), eclampsia (12 %) y labor prolongada o detenida (8 %).1 1 DEFINICIN Se define la hemorragia posparto (HPP) como la prdida sangunea de 500 mL. o ms en las primeras 24 horas despus del parto o el descenso del hematocrito en un 10 % o ms. CLASIFICACIN HPP Inmediata.- Prdida sangunea de 500 mL. o ms originada en el canal del parto dentro de las 24 horas posteriores al parto. HPP Tarda.- Sangrado anormal o excesivo originado en el canal del parto que se presenta entre las 24 horas posteriores al parto y el final del puerperio (42 das). FACTORES DE RIESGO Se han descrito los siguientes factores de riesgo para la HPP: Embarazo mltiple Polihidramnios Macrosoma Trabajo de parto disfuncional Gran multiparidad Corioamnionitis Uso inadecuado de oxitcicos Endometritis en el puerperio Prpura trombocitopnica Anestesia general Administracin previa y reciente de inhibidores uterinos Placenta previa Enfermedad de von Willebrand Desprendimiento prematuro de placenta Acretismo placentario CAUSAS DE HEMORRAGIA POSPARTO A continuacin se lisian las causas ms frecuentes de HPP. Segn su origen, se dividen en uterinas y no uterinas: 2 Uterinas - No Uterinas Hipotona o atona uterina Retencin de placenta o restosplacentarios o membranas Placentacin anormal (acretismo) Inversin uterina Traumatismo uterino (rotura uterina, desgarro cervical) Laceraciones del canal del parto, incluyendo la episiotoma Coagulopatas Hematomas

CUADRO CLNICO La HPP se caracteriza por los siguientes signos y sntomas: Sangrado transvaginal de moderado a grave. tero flcido (no contrado) o desgarros. Alteraciones hemodinmicas que se manifiestan como: mareos, sudoracin, nuseas, taquicardia y/o hipotensin arterial. Oliguria. tero nacido (no contrado). Al considerar la HPP, deben diferenciarse dos tipos de situaciones: la prevencin orientada a minimizar la probabilidad de que una mujer presente hemorragia tras el parto y el manejo o tratamiento de la hemorragia, cuando sta ya se haya producido

Referencias Bibliogrficas
1. AbdaRabbo SA: Stepwise uterine devascularization: A novel technique for management of uncontrollable postpartum hemorrhage with preservation of the uterus. Am J Obstet Gynecol 1994;171:694-700. Bakri YN, Linjawi T: Angiographic embolization for control of pelvic genital tract hemorrhage. Report of 14 cases. Acta Obstet Gynecol Scand 1992;71:17-21. Bick RL: Disseminated intravascular coagulation. Objetive criteria for diagnosis and manegement.- Med Clin N Am Vol 1994;78(3):511-43. B-Lynch C, Coker A, Lawal A II, Abu J, Cowen MC: The B-Lynch surgical technique for the control of massive postpartum haemorrhage: an alternative to? Five cases reported. Br J Obstet Gynaecol 1997;104:372-5. Brahaems D: Unwanted hysterectomies. Lancet 1993;342-61.

2.

3.

4.

5.

6. Braithwaite JL: Variations in origin of the parietal branches of the internal iliac artery. J Anatomy 1952;1:423-30.

25.- Femenimno de 26 aos , G-1, que cursa con embarazo de 37 SDG, presenta prdida del estado de alerta posterior a crisis convulsivas tnico-clnicas, signos vitales con T-A 170.120mmhg Fc 95x, reflejos osteotendinosos aumentados, se aprecia una Fc fetal de 132x y edema importante de miembros inferiores, no se aprecian datos de trabajo de parto ni modificaciones cervicales, El diagnstico ms probable es?

a) b) c) d)

Pre eclampsia severa Crisis epilptica de gran mal Hipertensin inducida por el embarazo Eclampsia

CUADRO 1. DIAGNSTICO* Preeclampsia Leve: Se presenta despus de la semana 20 de gestacin, durante el parto, o en las primeras 6 semanas despus de ste. Presin sistlica a 140 mm Hg o presin diastlica 90 mm Hg Proteinuria a 300 mg / orina de 24 hrs o su equivalente en tira reactiva Preeclampsia Severa: Se presenta despus de la semana 20 de gestacin, durante el parto, o en las primeras 6 semanas despus de ste Presin sistlica a 160 mm Hg o presin diastlica 110 mm Hg Proteinuria a 2 gr en orina de 24 horas o su equivalente en tira reactiva Creatinina srica > a 1.2 mg/dl Trombocitopenia 150 000 cel/mm3 Incremento de la deshidrogenasa lctica a 600 UI Elevacin al doble de los valores de TGO/AST o TGP/ALT Cefalea, alteraciones visuales o auditivas Epigastralgia Oliguria a 500 ml en 24 horas 7 Edema agudo de pulmn Dolor en hipocondrio derecho Restriccin en el crecimiento intrauterino Oligohidramnios Eclampsia Preeclampsia mas convulsiones sin otra causa. Se presenta despus de la semana 20 de gestacin, durante el parto, o en las primeras 6 semanas despus de ste. Sndrome de HELLP Criterios para establecer el diagnstico del sndrome de HELLP: Plaquetas < 100 000/mm3 TGO/AST 70U/L DHL 600U/LBilirrubina total > 1.2 mg/dl Se presenta despus de la semana 20 de gestacin, durante el parto, o en las primeras 6 semanas despus de ste. Hipertensin Crnica: Se diagnostica cuando existe hipertensin arterial a 140/90 mm Hg antes de la semana 20 de gestacin o si persiste despus de doce semanas posteriores al parto.

Las pacientes con hipertensin crnica deben ser evaluadas antes del embarazo para determinar la severidad de la hipertensin y facilitar la planeacin de un embarazo mediante el cambio de medicamentos y de hbitos higinicos y dietticos para evitar complicaciones. Hipertensin Gestacional: Presencia de hipertensin arterial a 140/90 mm Hg despus de la semana 20 de gestacin y se mantiene hasta las doce semanas despus del parto Ausencia de proteinuria Presencia o no de cefalea, acfenos y fosfenos Despus de 12 semanas de la interrupcin del embarazo se revalorar la presencia de hipertensin, si contina, se reclasifica como hipertensin crnica: es un diagnstico retrospectivo. 8 Si no hay, se clasifica como hipertensin transitoria.

1.

Aagaard-Tillery KM, Belfort MA. Eclampsia: morbidity, mortality, and management. Clin Obstet Gynecol 48:12-23, 2005.

2. Atallah AN, Hofmeyr GJ, Duley L. Calcium supplementation during pregnancy for preventing hypertensive disorders and related problems. Cochrane Database Syst Rev 1:CD001059, 2001. 3. Barton JR, Sibai BM. Diagnosis and management of hemolysis, elevated liver enzymes, and low platelets syndrome. Clin Perinatol 31:807-33, 2004. 4. Baxter JK, Weinstein L. HELLP syndrome: the state of the art. Obstet Gynecol Surv 59:838-45, 2004. 5. Cetin A. Eclampsia. In Mohler III ER, Townsend RR. Advanced therapy in hypertension and vascular disease. Ontario: B.C. Decker Inc. pp. 407-15, 2006. 6. Cetin A. Hemolysis, elevated liver enzymes, and low platelets (HELLP). In Mohler III ER, Townsend RR. Advanced therapy in hypertension and vascular disease. Ontario: B.C. Decker Inc. pp. 416-20, 2006.

26.- Femenino de 36 aos nulpara tras 2 aos de relaciones sexuales sin contracepcin, que desde hace 1 ao presenta dismenorrea, dispareunia y sangrado vaginal intermenstrual. Su primer orientacin diagnstica es? a) Insuficiencia lutenica. b) Enfermedad inflamatoria plvica. c) Dismenorrea funcional. d) Endometriosis.

La endometriosis consiste en la aparicin y crecimiento de tejido endometrial fuera del tero, sobre todo en la cavidad plvica como en los ovarios, detrs del tero, en los ligamentos uterinos, en la vejiga urinaria o en el intestino. Es menos frecuente que la endometriosis aparezca fuera del abdomen como en los pulmones o en otras partes del cuerpo. La endometriosis es una enfermedad relativamente frecuente, que puede afectar a cualquier mujer en edad frtil, desde la menarquia hasta la menopausia, aunque algunas veces, la endometriosis puede durar hasta despus de la menopausia. La endometriosis altera la calidad de vida de las mujeres que la padecen, afectando a sus relaciones de pareja, familiares, laborales y de reproduccin.

Sntomas Los sntomas clsicos son la dismenorrea, dolor intermestruales y en muchos casos, esterilidad. El dolor no tiene que ver con el tamao y la severidad de la lesin; generalmente cuanto menor es la lesin mayor dolor produce. El dolor se agrava con las menstruaciones y en los casos en que la lesin ocupa el fondo de saco de Douglas, puede dar dispareunia. Existe un aumento de la PGF2 alfa y PGE2 y un aumento de las contracciones uterinas que podra deberse a un depsito de endometrio en la cavidad peritoneal. La esterilidad debido a la endometriosis podra deberse a distintas causas de acuerdo a la severidad de la patologa. En los casos de endometriosis severa puede haber un factor tuboperitoneal con adherencias y alteracin en la anatoma de la pelvis que interfiera con el transporte del esperma y el vulo. En los casos de endometriosis leve hay varios mecanismos propuestos que justifican su relacin con la infertilidad: foliculognesis alterada, fase ltea inadecuada, fagocitosis espermtica, mala calidad ovocitaria, embriotoxicidad y alteracin a nivel de la implantacin.. La produccin de prostaglandinas por el endometrio ectpico puede afectar la motilidad tubaria, la foliculognesis y la funcin del cuerpo lteo. Puede haber un aumento de la activacin de los macrfagos peritoneales en la endometriosis que cause la fagocitosis de los espermas o la secrecin de citoquinas que pueden ser txicas para el embrin. Segn algunos investigadores habra un 60% de las mujeres con endometriosis que presentan un sndrome de Folculo Luteinizado no roto (LUF) en el cual el folculo no se rompe en la ovulacin y el vulo queda atrapado. plvico, dispareunia, sangrados

Referencias bibliogrficas

1. Ruiz V. Endometriosis y fertilidad. Ed. Acosta y Warman, pp. 99. 2. Lpes,VH. Palomo E. Incidencias de endometriosis en una poblacin infrtil. XXI Congreso nacional de Ginecologa y Obtetricia. Guatemala, 1993. 3. El-Eoley, et al. Danazol but not ginadotropin releasing hormone agonists suppresses autoantibodies in endomeriosis. Fertil Steril 1990; 54:725. 4. Acosta AA. Buttram VC Jr. Besch PK, Malinak LR, Van Der Heyden J. A.proposed classfication of pelvic endometriosis. Obstet Gynecol 1973;42:19. 5. Buttran VC Jr. Evolution of the revised American Fertility classification of endometriosis. Fert. Steril 1985; 43: 347. 6. Lpez VH. Tratamiento mdico-quirrgico de la endometriosis. Simposio El rostro cambiante de la endometriosis panam 3. 12. 1993. 7. Steinleitner A. Heterolous transplation of activated murine peritonel macrophages inhibitis gamete interaction in vivo; A paradigm fo endometriosis associted subfertility. Fertil Steril 1990; 54:725. 8. Damewood M. Effect of serum from patients with minimal to mild endometriosis on mouse embryo growth. Fertil Steril 1990; 54: 917. 9. Proug S. Peritoneal fluid fracctions from patients with endometriosis do not promote two-cell mouse embryo growth. Fertil Steril 1990; 54: 927.

27.- Femenino de 19 aos, gesta 1, tuvo un parto con un producto nico masculino de 3,600 g. la calificacin de Apgar en el RN fue de 9 al primer minuto y 9 a los 5 minutos. La revisin de sus registros de trabajo de parto mostr que tuvo ruptura de membranas 7 horas antes del parto. 40 horas despus del parto la paciente presenta: temperatura 38.2 C, FC 105 x, TA 110/70, FR 16x; E.F. dolor leve a la palpacin del tero, las mamas se encuentran sin eritema, ni dolor a la palpacin, no dolor a la compresin de las pantorrillas. Cul de los siguientes es el mejor tratamiento antes de iniciar los antibiticos?

a) b) c) d)

Hemocultivo Cultivo de secrecin vaginal Examen general de orina y cultivo Espirometra por incentivo

El vaciamiento incompleto ocasiona orina residual, distensin vesical excesiva y estasis, adems del cateterismo intermitente con sonda vesical durante el trabajo de parto. Por lo tanto, la vejiga en el puerperio est predispuesta a infecciones. El dolor leve a la palpacin del tero puede ser normal en el puerperio y no se debe suponer de inmediato endometritis puerperal. Cuando se sospecha endometritis, los cultivos de secrecin vaginal tienen poca utilidad porque se encuentran los mimos microorganismos que en mujeres purperas sanas. Los hemocultivos son apropiados para la valoracin diagnstica de la fiebre puerperal, pero no son el paso inicial. La espirometra por incentivo se utiliza en el posoperatorio de inmediato para fomentar la expansin pulmonar y disminuir las atelectasias. El legrado uterino se utiliza para tratar la hemorragia. Morgan M, Siddighi S. Ginecologa y obstetricia, National Medical Series. 5 edicin. Mc Graw Hill. Pp. 29.

28.- Femenino que cursa con 36. 5 semanas de gestacin acude al servicio por referir malestar general, fosfenos, nausea y vmito, aprecia moderada ictericia, usted sospecha de un sndrome de HELLP Que alteraciones de laboratorio espera encontrar al confirmar el diagnstico? a) b) c) d) Trombocitosis, Enzimas heptica elevadas, Anemia hemoltica. Anemia hemolitica, Trombocitosis, Fosfatasa Alcalina elevada. Anemia Hemoltica, trombocitopenia, enzimas hepticas elevadas. Trombocitopenia, Leucopenia, Hipertensin Arterial.

DEFINICIN: Es una complicacin de la preeclampsia en la cual adems de la Hipertensin Arterial y proteinuria hay presencia de anemia hemoltica, enzimas hepticas elevadas y recuento bajo de plaquetas

EPIDEMIOLOGIA: Se presenta en un 4 a 10% de las preeclmpticas, diagnosticndose anteparto en un 70% de los casos preferentemente antes de las 37 semanas, mientras que el 30% de los casos restantes enferma en los primeros 7 das del puerperio, sobre todo en las 48 h iniciales. La proteinuria e hipertensin pueden estar ausentes en un 15 al 20% de los casos. Incidencia mayor en multigestantes y en edades avanzadas.

Ocurre ms frecuentemente cuando se demora la salida del feto y cuando se presenta desprendimiento de la placenta. Mortalidad materna del 24% y mortalidad perinatal del 30-40%.

CLASIFICACION: Sndrome de HELLP. Clasificacin de Mississipi. CLASE Plaquetopenia LDH 1 2 Severa <50000 Moderada >50000 <100000 Ligera >100000 <150000 >150000 (sin >600 IU/L >600 IU/L

AST-ALT >70 IU/L >70 IU/L

3 PE severa Eclampsia HELLP)

>600 IU/L <400 IU/L

>40 IU/L <70 IU/L <40IU/L

MANIFESTACIONES CLINICAS: Malestar general, fatiga y molestias inespecficas 90% Cefalea 70% Epigastralgia 64% Vmito 22% Fosfenos 15% Visin Borrosa 11% Acfenos 3% Ictericia Anemia no explicada Oliguria

Si se aade una HEMORRAGIA HEPTICA, el paciente puede quejarse de dolor en el HOMBRO DERECHO y EL CUELLO, adems de las molestias abdominales. Equimosis en los sitios de punciones venosas, petequias en los sitios de presin del brazo, pero pueden tener pruebas de Rumpel Leed negativas. En casos severos se pude presentar ascitis como causa de hipertensin portal.

DIAGNOSTICO: El diagnstico clnico del sndrome de HELLP se plantea en gestantes o purperas con preeclampsia severa-eclampsia, excepto en el 15-20%, en las cuales esta asociacin no puede ser demostrada, en tanto se cumplan los criterios de Sibai:

MANIFESTACIONES CLINICAS: Malestar general, fatiga y molestias inespecficas 90% Cefalea 70% Epigastralgia 64% Vmito 22% Fosfenos 15% Visin Borrosa 11% Acfenos 3% Ictericia Anemia no explicada Oliguria

Si se aade una HEMORRAGIA HEPTICA, el paciente puede quejarse de dolor en el HOMBRO DERECHO y EL CUELLO, adems de las molestias abdominales. Equimosis en los sitios de punciones venosas, petequias en los sitios de presin del brazo, pero pueden tener pruebas de Rumpel Leed negativas.

En casos severos se pude presentar ascitis como causa de hipertensin portal.

DIAGNOSTICO: El diagnstico clnico del sndrome de HELLP se plantea en gestantes o purperas con preeclampsia severa-eclampsia, excepto en el 15-20%, en las cuales esta asociacin no puede ser demostrada, en tanto se cumplan los criterios de Sibai:

HEMOLISIS Frotis perifrico anormal (eritrocitos fragmentados) Hematocrito (>24%) Bilirrubina indirecta (>1.2mg/dL) Deshidrogenasa lctica (>218 UI/L)

ENZIMAS HEPTICAS ELEVADAS

LDH >218UI/L AST >30UI/L ALT >37UI/L

PLAQUETAS BAJAS <100.000/mm3

BIBLIOGRAFIA: Sibai baha, El sndrome HELLP. Universidad de Valencia , revista quincenal de Obstetricia clnica y ginecologa, Octubre 2003. V. Cararach, Sndrome de HELLP y Repercusiones maternas. X curso intensivo de formacin continuada materno fetal. Enero de 2003. Toirac, Abelardo. Sndrome de Weistein HELLP Hospital Ginecoobstetrico Tamara Bunke. Junio 2002.

De la Fuente, David. Sndrome HELLP. Medicina Universitria 2003; 5 (19): 101 -9 Andrea G. Witlin, DO, Baha M. Sibai, MD. Diagnosis and Management of women with Hemolysis Elevate Liver Enzymes, and Pletelet Count (HELLP) syndrome. Hospital Physician. Febrero 1999. CIFUENTES B, Rodrigo. Ginecologa y obstetricia basadas en las evidencias. Bogot: Distribuna, 2006. Sexta edicin. 447 - 283 p.

29.- Femenino de 28 aos acude al servicio de consulta externa refiriendo presentar baches amenorreicos, acn, hirsutismo y esterilidad de aproximados 2 aos de evolucin E.F. con ndice de masa corporal 31 KG/M2, es diagnstico ms probable es:

a) b) c) d)

Hipotiroidismo. Fallo ovrico precoz. Sndrome del ovario poliqustico. Amenorrea de causa uterina.

El sndrome de ovarios poliqusticos (SOPQ) afecta aproximadamente a un 4% de mujeres en edad reproductiva y se caracteriza por anovulacin crnica e hiperandrogenismo. Es la causa ms comn de infertilidad en mujeres. Se caracteriza clnicamente por acn, alopecia, hirsutismo, irregularidades menstruales e infertilidad. Los hallazgos de laboratorio ms frecuentes son: aumento de la hormona luteinizante (LH), aumento de la relacin LH/FSH (hormona folculoestimulante), aumento de andrgenos (tanto ovricos como adrenales) y de estrgenos circulantes. Otros hallazgos de laboratorio habituales son una prueba tolerancia oral a la glucosa anormal y alteraciones en el perfil lipdico. Todo esto junto con las imgenes ecocardiogrficas caractersticas definen al sndrome. La teraputica permite dos grandes enfoques que pueden superponerse: la correccin de las manifestaciones de hiperandrogenismo y el tratamiento de las alteraciones del eje reproductivo (anovulacin, esterilidad). Los antiandrgenos estn fundamentalmente indicados para tratar los sntomas virilizantes. Las alternativas para inducir la ovulacin son numerosas: al citrato de clomifeno y a la antigua reseccin en cua se agregan las gonadotrofinas humanas, pulsos de GnRH (hormona liberadora de gonadotrofinas), medidas o frmacos para modificar los niveles de insulina, y finalmente tcnicas quirrgicas endoscpicas para reducir la masa ovrica. Revista de Posgrado de la VIa Ctedra de Medicina - N 125 Marzo 2003 Pg. 37-40 SINDROME DE OVARIOS POLIQUISTICOS Dra. Sandra Beneyto, Dra. Mara Andrea Ferreyra, Dr. Andrs Galfrascoli, Dr. Andrs Gonzlez, Dra. Susana Sosa

30.- Femenino de 34 aos, es atendida en sala de partos secundario a eutocia, durante la reparacin de la episiotoma media hay un marcado incremento en el sangrado transvaginal. La medida teraputica inmediata en esta paciente es:

a) Masaje y compresin del fondo uterino b) 20 unidades IV de oxcitocina c) 0.2mg Im de metilergonovina d) Empaquetar con gasas

El tratamiento clsico de la atona uterina est constituido por la reposicin volumtrica (sangre, coloides y cristaloides), la utilizacin de masajes y por la administracin de drogas que promueven la contraccin del msculo uterino. Cuando estas Medidas no son eficientes, se procede casi invariablemente a la histerectoma de hemostasia. Es de notar, que la atona uterina puede aparecer en el primer embarazo y sin ningn antecedente previo. La atona uterina constituye, en casi todas las series mundiales, ms del 50% de las hemorragias graves del posparto. Patologa de Urgencia, Ao 9, Nro. 3, Septiembre de 2001

31.- Mujer de 35 aos que acude a consulta con antecedentes de G3 C2 A1, refiere que ha presentado durante el primero y segundo trimestres de su embarazo manchado con frecuencia intermitente ,a las 34 SDG inicia con hemorragia abundante, repentina e indolora, su principal sospecha es:

a) Coriocarcinoma b) Ruptura uterina c) Placenta previa d) Desprendimiento grave de placenta normoinserta

PLACENTA PREVIA DEFINICIN: Es cuando la placenta se implanta sobre o muy cerca del orificio cervical interno y una parte de la placenta precede a la parte fetal que se presenta. INCIDENCIA: Esta es difcil determinar ya que muchos casos pasan desapercibidos, sobre todo cuando ocurren los abortos en embarazos tempranos.

La prevalencia vara de 1 en 100 a 1 en 850 nacidos vivos, pero solo el 20% total. Etiologa: Edad avanzada, multparas, paciente con cesreas previas, paciente con aborto de repeticin, esto debido a las gestaciones previas. TIPOS: Insercin baja.- Es cuando el borde placentario se encuentra en el segmento inferior a menos de 6 cm del orifico cervical interno. Marginal.- Es cuando el borde placentario alcanza los mrgenes del orificio cervical interno. Parcial.- Es esta la placenta cubre parcialmente el orificio cervical interno. Total.- La placenta cubre la totalidad del orificio cervical interno an con dilatacin cervical avanzada. DIAGNOSTICO: La caracterstica es el STV de aparicin brusca en forma indolora en el segundo o tercer trimestre. Frecuentemente hay ausencia de dolor a actividad uterina que son parmetros para hacer el diagnstico. La mayor incidencia de sangrado aparece a las 33-34 sdg. ESTUDIOS DE GABINETE: El estudio mas utilizado es la ultrasonografa obsttrica. TRATAMIENTO: El manejo va a depender de factores como son: Edad gestacional, magnitud del sangrado, si hay trabajo de parto, variedad de placenta previa y complicaciones materna.

BIBLIOGRAFA:

1.- Waxler P, Gottesfeld KR. Early diagnosis of placenta previa. Obstet Gynecol 1979;54:231-32. 2.- Cabrero-Roura L. Riesgo elevado obsttrico. Ed. Masson 1996; pp; 109-118. 3.- Patrick J, Placenta Previa, Clinical Obst and Gynecology 1990;33(3): 414-421. 4.- Chapman M, Furtenes ET, Significance of ultrasound in location of placenta in early pregnancy Br J Obst Gynecol 197;86: 846.

32.- Se trata de femenino de 20 aos con menarca a los 13 aos ritmo menstrual 45x4. Refiere vida sexual activa desde los 17 aos con frecuencia de 4 veces por semana. A la exploracin se encuentra acn intenso en la frente, mejillas y mentn. Acude a consulta por que desea adoptar un tratamiento anticonceptivo por va oral, el tratamiento ms adecuado es:

a) b) c) d)

Norgestimato Gestodeno Levonorgestrel Ciproterona

El efecto antiandrognico especfico del acetato de ciproterona acta por inhibicin competitiva de la unin de la 5 - alfa - dihidrotestosterona con el receptor citoslico de las clulas blanco, que disminuye la produccin y la excrecin de sebo y el aumento y el desarrollo del vello. Es un derivado de la 17 - alfa - hidroxiprogesterona que posee accin progestgena. Su accin antigonadotrfica se suma a la del etinilestradiol. El acetato de ciproterona no posee accin estrognica sino un efecto antiestrognico, y tampoco posee accin nociva sobre la funcin de la corteza suprarrenal;

Indicaciones en la mujer: Manifestaciones de androgenizacin de grado severo, por ejemplo, hirsutismo grave, alopecia androgentica de tipo grave, a menudo acompaados por manifestaciones graves de acn y/o seborrea. Indicaciones en el hombre: Atenuacin del impulso en las desviaciones sexuales. Tratamiento antiandrgeno del carcinoma de prstata inoperable. BIBLIOGRAFA: 1. Swift S. Current opinion on the classification and definition of genital tract prolapse. Curr Opin Obstet Gynecol 2002; 14: 503-7. 2. De Caro R, Aragona F, Herms A, Guidolin D, Bizzi E, Pagano F. Morphometric analysis of the fibroadipose tissue of the female pelvis. J Urol 1998; 160: 707-13. 3. Gill E, Hurt W. Pathophysiology of pelvic organ prolapse. Clin Obstet Gynecol 1998; 25(4): 757-69. 4. DeLancey, J. Anatomic aspects of vaginal eversion after Hysterectomy. Am J Obstet Gynecol. 1992; 166(6 pt 1): 1717-24.

33.- Acude a consulta mujer de 23 aos la cual presenta un ndulo mamario palpable de aparicin brusca. La ecografa revela un ndulo anecognico, de limites muy precisos, morfologa regular y refuerzo posterior, nico de 3.5 cms. de dimetro. El diagnstico ms probable es? a) Cncer. b) Displasia fibrosa. c) Fibroadenoma. d) Quiste

Quistes. Los quistes mamarios son fciles de detectar con la ecosonografa. Pueden ser lesiones nicas o mltiples que se observan como imgenes redondeadas, anecognicas, de paredes delgadas, contornos bien definidos, con importante reforzamiento acstico posterior y sombras laterales delgadas. Pueden presentar septos intraqusticos y, en ocasiones, se pueden observar ecos internos que sugieren detritus celulares o proceso inflamatorio. Se debe descartar la presencia de lesiones intraqusticas o la coexistencia de otras alteraciones benignas o malignas.

En caso de ser sintomticos, el tratamiento adecuado es la puncin y aspiracin de la lesin con aguja guiada por palpacin o ecosonografa de acuerdo con ell tamao, profundidad y caractersticas del contenido. El uso del ultrasonido garantiza el vaciamiento completo.

REFERENCIAS: Barth V, Prechtel K. Mama normal. En: Barth V, Prechtel K, editores. Atlas de patologa de la glndula mamaria. 2da ed. Madrid: Editorial Mdica Panamericana, 1991. Bush H, McCredie A. Carcinoma of the breast during pregnancy and lactation. In: Allen HH, Nisker JA. Cancer in pregnancy. New York: Futura Publishing Co. Inc., 1986. Byrd BF, Bayer DS, Robertson JC, Stephenson JE Jr.

34.- Femenino de 35 aos de edad, a quien se realiza diagnstico de placenta percreta, el tratamiento e eleccin en esta patologa es:

a) b) c) d)

Ergonovina a dosis altas. Histerectoma. Hemostasia con puntos transfictivos. Taponamiento uterino.

La placenta anormalmente adherida es poco comn y tiene importancia clnica por su morbimortalidad, a consecuencia de hemorragia, perforacin, invasin y lesin de las vas urinarias. Esta adherencia anormal est asociada con la implantacin placentaria sobre cicatrices de cesrea previa, incisiones uterinas o legrados. La placenta percreta consiste en la penetracin del tejido placetario a travs de toda la pared uterina, traspasando la serosa de la misma. La identificacin de esta anormalidad antes del parto es posible mediante mtodos de imagen (escala de grises por ultrasonido, ecografa Doppler color pulsado o resonancia magntica nuclear). El tratamiento conservador se acompaa de elevada morbilidad en muchos casos, por lo que el tratamiento quirrgico se convierte en el definitivo. La literatura sugiere un aumento previsto en la incidencia de esta condicin con base en el incremento del nmero de cesreas, por lo que la histerectoma postcesrea ser una decisin que enfrentarn los especialistas con mayor frecuencia.

1. Perucca E, Domnguez C, Yahng Ch, Garca R. Placenta previa percreta con invasin vesical. Rev Chil Obstet Ginecol 1997; 62(3): 206-10. 2. Abbas F, Talati J, Wasti S et al. Placenta percreta with bladder invasion as a cause of life threatening hemorrhage. J Urol 2000; 164: 1270-4. 3. Perucca E, Cazenave H, Barra A, Ochoa N, Villagrn G, Espinoza R, Estay R, Bustamante R, Siebert A. Placenta previa percreta con invasin vesical. Rev Chil Obstet Ginecol 2002; 67(5): 364-7. 4. Price F, Resnik E, Heller K, Christopherson W. Placenta previa percreta involving de urinary bladder. A report of two cases and review of the literature. Obstet Gynecol 1991; 78(3):508-11.

35.- Femenino de 29 aos con antecedentes de G/3, P/2, C/1 acude al servicio de consulta externa, refiere que presenta una secrecin transvaginal bastante lquida, de baja viscosidad, maloliente de color amarillo y gris, espumoso. Esta entidad es propia de infeccin por:

a) b) c) d)

Cndida albicans Gardenerella Gonococos Tricomonas

El protozoario Thricomona Vaginalis es el responsable del 25% de las vaginitis. 20-50% de las mujeres cursan asintomticas. La tricominiasis es predominantemente una infeccin transmitida sexualmente. Debe sospecharse de abuso sexual en caso de encontrar Trichomona en pacientes peditricos.

DIAGNOSTICO: Flujo vaginal amarillo-verdoso con burbujas, muy ftido, irritacin vulvo-vaginal, disuria. El pH suele ser mayor de 4.5. Estudio en fresco en donde se observa al microscopio el organismo flagelado caracterstico. 75% se diagnostican con el estudio del Papanicolaou.

Referencias bibliogrficas: 1. Secretara de Salud. Norma Oficial Mexicana NOM -039-SSA2-2002, Para la prevencin y control de las infecciones de transmisin sexual. D.O.F. 19 de Septiembre 2003. 2. Kettler H, White K, Hawkes S. Mapping the landscape for sexually transmitted infections: key findings and recommendations. Geneva, TDR (TDR/STI/ IDE/04.1). 3. CDC. Trends in Reportable Sexually Transmitted Diseases in the United States. CDC, National Report. 2004. 4. Distribucin de los casos nuevos de enfermedades por mes Estados Unidos Mexicanos 2004. Sistema nico de Informacin para la Vigilancia Epidemiolgica/Direccin General de Epidemiologa/SSA. 5. Aral S O. Sexual risk behaviour and infection: epidemiological considerations. Sex. Transm. Inf. 2004;80:8-12

36.- A 71 years-old man complains of occasional lower back pain. His blood pressures 150/85 mmHg and his pulse is 80/min. Cardiac examination reveals an S4 gallop. Abdominal examination reveals a pulsatile mass approximately 5.0 cm in diameter palpable in the epigastric area. Peripheral pulses are normal. Which of the following is the most likely diagnosis?

a) b) c) d)

Abdomninal aortic aneurysm Cancer of the proximal colon Chronic pancreatitis Lipoma of the abdominal wall

Epidemiologa: Los aneurismas abdominales, afectan al 2-5% de los mayores de 60 aos y ms en hombres que en mujeres, en una proporcin 4/1 (1).

La mayora son de etiologa arterioesclertica y de ubicacin infrarrenal (90%); y pueden extenderse a una o ambas arterias ilacas (70% de los casos) (1). Segn la forma, los aneurismas son fusiformes (75%) cuando afecta toda la circunsferencia del vaso, o saculares (con cuello) cuando solo est englobado una porcin de dicha circunsferencia. Estos ltimos son infrecuentes de observar en la aorta abdominal, y por lo general se ubican proximales al origen de las arterias renales. Clnica: El 75% de los pacientes son asintomticos, y se descubren por hallazgos en exmenes de rutina o por una masa pulstil en determinadas posiciones corporales. Ocasionalmente el diagnstico se hace por laparotoma por otras patologas (1).

Los pacientes sintomticos consultan por: Dolor epigstrico o lumbar. Masa pulstil dolorosa a la palpacin. Sntomas gastrointestinales como nauseas, vmitos y prdida de peso, en aneurismas de gran tamao. Asociado a episodios isqumicos en miembro inferior (raro).

Lo ms frecuente es el hallazgo de una masa pulstil umbilical o supraumbilical (se palpa cuando su dimetro es > a 4,5 cm).

BIBLIOGRAFIA: Ferraina P, Oria A. Ciruga de Michans. Buenos Aires: El Ateneo, 2000: 949-955 Farreras R. Medicina Interna. Madrid: Mosby/ Doyma Libros,1995: Vol 2: 650-651 Frutos Ortiz E, Moirano J, Fassi J. Ciruga. Buenos Aires: El Ateneo, 1993: 67.11 Chang JB, Stein TA, Liu JP, Dunne ME. Risk factors associated with rapid growt of small abdominal aortic aneuryms. Surgery 1997, 121: 117-122 5. Matsushita M, Nishikimi N, Sakurai T, Nimura Y. Relationship betwen aortic calcification and atherosclerotic disease in patients with abdominal aortic aneurym: Int Angiol 2000, 19: 276- 279 6. Du Toit DF, Louwrens H, Klompje J, Grenewald JH. Ruptured abdominal aortic aneurym and horseshoe kidney. A Afr Med J 1983, 64: 750-751 1. 2. 3. 4.

37.- En su jurisdiccin se reportaron 44 casos de hepatitis A entre los alumnos de segundo grado de una escuela secundaria durante la primera quincena del mes de septiembre. La tasa de ataque es de 26.4%. Esta situacin nos indica que estamos ante un(a): a) Brote b) Endemia c) Epidemia d) Pandemia

De acuerdo a la NOM- 017 un brote se define como la ocurrencia de dos o ms casos asociados epidemiolgicamente (tiempo, lugar y persona) entre s. La medida cuantitativa de la extensin de un brote es la Tasa de Ataque (TA) que se calcula dividiendo el nmero de casos nuevos entre el total de personas expuestas por 100. Greenberg R. S; Epidemiologa mdica, Manual Moderno, 2. Ed. Pgs. 77-79

38.- Masculino de 44 aos de ead con antecedentes de DM de 5 aos de evolucin as como dislipidemia mixta. Actualmente controlado con sulfonilureas y estatinas .La primera lesin que se observa en la retinopata diabtica temprana es:

a) b) c) d)

Microaneurismas Exudados blandos Exudados duros Hemorragias intrarretinianas

La retinopata diabtica tiene cuatro etapas: 1. Retinopata no proliferativa ligera. Esta es la etapa ms temprana de la enfermedad en la que aparecen los microaneurismas. Estas son pequeas reas de inflamacin, que parecen ampollas, en los pequeos vasos sanguneos de la retina. 2. Retinopata no proliferativa moderada. Segn avanza la enfermedad, algunos vasos sanguneos que alimentan la retina se obstruyen. 3. Retinopata no proliferativa severa. En esta etapa muchos ms vasos sanguneos se bloquean, haciendo que varias partes de la retina dejen de recibir sangre. Entonces estas reas de la retina envan seales al cuerpo para que haga crecer nuevos vasos sanguneos. 4. Retinopata proliferativa. En esta etapa avanzada, las seales enviadas por la retina para alimentarse causan el crecimiento de nuevos vasos sanguneos. Esto se llama la retinopata proliferativa. Estos nuevos vasos sanguneos son anormales y frgiles. Crecen a lo largo de la retina y de la superficie del gel vtreo, el gel incoloro que llena el interior del ojo.

Retinopata diabtica. Forma leve de retinopata diabtica, en la que destacan microaneurismas /microhemorragias de predominio en arcada temporal superior, junto con exudados duros que se acercan a la mcula. 1. The Diabetes Control and Compications Trial Research Grop. The Effect of Intensive treatment of Diabetes on the development and progression of long term complications in insulin dependent Diabetes Mellitus N Engl J. Med; 1993 ; 329 : 977-986. 2. Early treatment Diabetic Retinopathy Study Research Group Early photocoagulation for diabetic Retinopathy ETDRS Report 9. Ophthalmology, 1991 ; 98 : 1316-26.

3. Kahn HA Hiller R. Blindness caused by diabetic retinopathy Am. J. Ophthalmol, 1974 ; 78 ; 58-67. 4. Ladas ID. Theossiadis GA Long term efectiveness of modified grid photocoagulation for diffuse Macular edema Acta Ophthalmol, 1993 ; 71(3) ; 393-7. 5. Klein R, Klein BEK , Moss SE , et al The Eisconsin Epidemiology Group Ophthalmology, 1984 ; 91 : 1464-74

39.- Se presenta ante usted femenino de 34 aos con referencia por diagnstico de lupus eritematoso generalizado, Cul es el sntoma dentro de los criterios de clasificacin de de ste padecimiento?

a) b) c) d)

Fotosensibilidad Alopecia Artralgias Prdida de peso

Criterios de Clasificacin para el Diagnstico de Lupus Eritematoso Sistmico (LES) Erupcin malar: Eritema fijo, plano o alto, sobre las eminencias malares, que no suele afectar los surcos nasogenianos. Erupcin discoide: Placas eritematosas altas, con descamacin queratsica adherente y tapones foliculares; puede haber cicatrices atrficas en las lesiones ms antiguas. Fotosensibilidad: Erupcin cutnea a causa de una reaccin inslita a la luz solar, referida por el paciente u observada por el mdico. lceras bucales: Ulceracin nasofarngea, por lo comn indolora, observada por un mdico. Artritis: Artritis no erosiva que afecta dos o ms articulaciones perifricas, caracterizada por dolor a la palpacin, tumefaccin o derrame. Serositis: Pleuritis o pericarditis documentada por electrocardiograma o frote o evidencia de derrame pericrdico. Enfermedad renal: Proteinuria persistente mayor a 0,5g/da o 3+ o cilindros celulares. Transtorno neurolgico: Convulsiones o psicosis en ausencia de otra causa conocida. Transtorno hematolgico: Anemia hemoltica o leucopenia (< 4.000/mm3) o linfopenia: (<1.500/mm3) o trombocitopenia (< 100.000/mm3) en ausencia de frmacos que produzcan esta alteracin. Trastorno inmunolgico: Anti-DNA, anti-Sm, y/o Anticuerpos antifosofolipdicos (AFL). Anticuerpo antinuclear: Un ttulo anormal de ANA por inmunofluorescencia o anlisis equivalente en cualquier momento y en ausencia de medicamentos relacionados con el sndrome de lupus de origen farmacolgico. Cualquier combinacin de 4 o ms de los 11 criterios, bien documentado durante cualquier intervalo de la historia del paciente, hace el diagnsticos de LES (especificidad y sensibilidad son del 95% y 75%, respectivamente).

Petri M. Review of classification criteria for systemic lupus erythematosus. Rheum Dis Clin North Am. 2005 May;31(2):245-54.

40.- Masculino de 44 aos, acude a consulta por presentar de manera espontnea una erupcin de lesiones pruriginosas, habonosas, diseminadas por toda la superficie corporal, de evolucin fugaz as como una amplia placa edematosa en hemicara izquierda. El diagnstico ms probable es:

a) b) c) d)

Angioedema hereditario. Erisipela. Vasculitis. Urticaria aguda.

La urticaria se define como un sndrome reaccional de piel y mucosas caracterizado por edema y ronchas pruriginosas ocasionadas por edema vasomotor transitorio y circunscrito de la dermis que dura algunas horas; puede ser recidivante y de origen inmunolgico, no inmunolgico o desconocido.1 La lesin elemental es una ppula edematosa drmica (roncha).2 El angioedema se manifiesta tpicamente como un edema asimtrico causado por la presencia de plasma dentro de tejido celular subcutneo y mucosas.1 Clasificacin La urticaria se puede clasificar de acuerdo con diferentes parmetros: 1) segn la evolucin: en aguda o crnica; 2) segn el cuadro clnico: en urticaria ordinaria (urticaria propiamente dicha), urticaria fsica (por estmulo detonador), urticaria por contacto (inducida por un contacto qumico o biolgico) y angioedema (sin ronchas), en el cual el espectro de las manifestaciones clnicas de los diferentes tipos es muy amplio; y 3) segn el mecanismo potencial de su desarrollo: inmunolgico, no inmunolgico, mediada por el complemento, o bien urticaria autoinmune.3 Clasificacin por evolucin La urticaria aguda se define tradicionalmente por la presencia de ronchas de forma espontnea, casi la mayora de los das, por menos de seis semanas.4 La urticaria crnica se define como la presencia de ronchas de forma espontnea por ms de seis semanas, diariamente o casi la mayora de los das de la semana. BIBLIOGRAFIA: 1- Yadav S, Upadhyay A, Bajaj A. Chronic urticaria: An overview. IJD [en lnea] 2006 [fecha de acceso 4 de marzo de 2007];51-3:171-177. Disponible en: 2- Woscoff A, Kaminsky A, Marini M, Allevato M. Dermatologa en Medicina Interna. Buenos Aires, Edicin de los Autores, 2003: 24-27. 3- Criado PR, Fachini Jardim Criado R, Maruta C, Costa Martins JE, Rivitti E. Urticaria. An.Bras.Dermatol 2005,v.80 n.6: 183-185.

41.- Femenino de 34 aos de edad G4 P3, acude a consulta en busca de informacin acerca de los mtodos anticonceptivos. No tiene APP de importancia, ni toma medicamentos. Ha mantenido una relacin mongama con su pareja por los ltimos 9 aos. No quiere volver a embarazarse, pero se niega a que se le practique una salpingoclasia. Su mdico recomienda un DIU. Con ste mtodo anticonceptivo existe mayor probabilidad de?

a) b) c) d)

Amenorrea Embarazo ectpico Embarazo intrauterino Aumento de peso

ETIOLOGIA

III.1 Factores que dificultan el camino del huevo. Este se implanta all donde se encuentra
en el 6-7 da postfecundacin. A/ Procesos inflamatorios, causa ms frecuente de embarazo ectpico. Las salpingitis deterioran la actividad ciliar y la motilidad tubrica. Hay otros procesos inflamatorios, como, apendicitis que afectan secundariamente a las trompas, con produccin de adherencias y acodaduras. B/ Alteraciones de la motilidad tubrica, como ocurre con la administracin de gestgenos a bajas dosis de forma continua (minipildora) o el empleo de contracepcin postcoital con estrgenos. C/ DIUs. 9-17% de embarazos en portadoras de DIU son ectpicos, lo que se explica porque el DIU protege ms eficazmente frente a la gestacin intrauterina (995% de seguridad) que frente al tubrico (95% de seguridad) con lo que aumenta el riesgo relativo de E.E. D/ Tratamientos de esterilidad, sobre todo los quirrgicos, cirugia conservadora de E.E tubricos, ligadura tubrica o tcnicas de fecundacin asistida (in vitro y transferencia embrionaria intrauterina) III.2 Factores que favorecen la nidacin en la trompa: endometriosis tubrica, por un cierto quimiotactismo del endometrio ectpico, as como, por la estenosis y adherencias que se producen incluso con rganos vecinos. III.3 Factores dependientes del propio huevo. Las anomalas genticas del huevo, mediante alteracin en el momento de implantacin, o quiz por un mayor volumen, puede favorecer la anidacin ectpica.

BIBLIOGRAFA EMBARAZO ECTOPICO: Cabero Roura, Ll y cols. Protocolos de Medicina Materno-fetal (Perinatologa), 2 edicin. Ed. Ergon, S.A. Madrid. 2000. 120-122. Bajo Arenas, J.M, Castellanos Bolado, P. Embarazo ectpico. En: Manual de asistencia a la patologa obsttrica. Fabre E, Ed. INO: Zaragoza. 1997. 89 -107. Carrera Macia, J.M y cols. En: Protocolos de obstetricia y Medicina perinatal del I. U. Dexeus. 3 Edicin. Masson. Barcelona. 2000. 135-139.

42.- Una mujer de 67 aos refiere intenso prurito vulvar y sensacin quemante, al examen el introito vaginal se encuentra estentico. Cul de los siguientes es el tratamiento apropiado? a) b) c) d) 5-fluoracilo Estrgeno tpico Testosterona tpica Corticoesteroides fluorados

Vulvovaginitis atrfica
El hipoestrogenismo conduce a atrofia de la vagina y el vestbulo vulvar, que los hace fcilmente irritables y susceptibles a infecciones secundarias. Las pacientes refieren sensacin de quemadura, prurito, disuria, hipersensibilidad y dispareunia. Puede encontrarse al examen fsico atrofia, fisuras superficiales, y un flujo vaginal acuoso1. Hay disminucin del tamao del introito2, prdida de la rugosidad y la vagina toma una apariencia lisa y brillante. Los hallazgos histolgicos revelan un epitelio vaginal delgado, disminucin de los lechos capilares, y la citologa muestra, a medida que la atrofia progresa, aumento de las clulas basales y disminucin o ausencia de las clulas superficiales2. Se aconseja evitar el uso de jabones y dems irritantes de la piel. Se pueden utilizar lubricantes simultneamente con los estrgenos o como terapia nica, si hay alguna contraindicacin a las hormonas. El tratamiento con estrgenos por va sistmica o transvaginal mejora y restaura los signos y sntomas, y una a dos semanas despus de iniciar el tratamiento los cambios de atrofia empiezan a mejorar rpidamente, se reduce el pH y se induce maduracin vaginal y de la mucosa uretral, reduciendo la frecuencia de las infecciones urinarias3. La dosis y va de administracin debe ser debidamente individualizada4. Contraindicaciones al tratamiento con estrgenos, incluyen: la presencia de tumores estrgenosensibles, falla heptica terminal y antecedentes de tromboembolizacin relacionada con ellos.

Menopausia y Piel. Parte II: Manifestaciones clnicas dermatolgicas durante la menopausia MARA ISABEL BARONA C. Docente adjunto. Dermatloga Universidad del Valle-Cali.

43.- Femenino de 27 aos, es atendida en consulta en la clnica de displasias por papanicolau con lesin NIC I. Antecedentes: menarca 14 aos, ritmo 30x5 eumenorreica, inicio de vida sexual a los 15 aos, 2 parejas sexuales, mtodo de planificacin familiar oclusin tubaria bilateral, gestas 3 partos 3, crvix con lesin acetoblanca con extensin lineal de 2 cm. Para confirmar el diagnstico se debe realizar:

a) b) c) d)

Crioterapia de lesin. Captura de hbridos. Repetir colposcopa Biopsia de la lesin

9.5.2 Las pacientes a quienes se les realiz citologa cervical, cuyo resultado es LEIBG (infeccin por VPH, displasia leve o NIC 1); LEIAG (displasia moderada y grave o NIC 2 y 3) o cncer deben enviarse a una clnica de colposcopa, para realizar estudio colposcpico. 9.5.3 Si el resultado de la citologa es LEIBG, la colposcopa es satisfactoria y sin evidencia de LEIBG, se realizar control citolgico en un ao (Apndice Normativo A) 9.5.4 Si la citologa es de LEIBG, la colposcopa es satisfactoria y existe evidencia de lesin, se debe tomar una biopsia dirigida. 9.5.4.1 Si la biopsia dirigida es negativa, se realizar nueva colposcopa para verificar el diagnstico y en caso necesario, tomar nueva biopsia dirigida y revalorar. 9.5.4.2 Si la biopsia dirigida es reportada como LEIBG se podr dar tratamiento conservador: criociruga, electrociruga o laserterapia (slo si cumple con las condiciones referidas en el Apndice 1) o se podr mantener a la paciente en vigilancia en la clnica de colposcopa, con colposcopa y estudio citolgico cada seis meses, durante 24 meses. Jueves 31 de mayo de 2007 DIARIO OFICIAL (Primera Seccin) 9.5.4.3 Si la biopsia dirigida es reportada como LEIAG (Lesin Intraepitelial Escamosa de Alto Grado) se realizar tratamiento conservador (electrociruga o laserterapia). En las mujeres posmenopusicas, dependiendo de las condiciones anatmicas del crvix, se realizar tratamiento conservador en la clnica de colposcopa o tratamiento quirrgico (histerectoma extrafascial) en el servicio que corresponda. 9.5.4.4 Si la biopsia dirigida reporta cncer microinvasor o invasor, la paciente se transferir a un Servicio o Centro Oncolgico para su tratamiento correspondiente. 9.5.4.5 Si la citologa reporta LEIBG y la colposcopa es no satisfactoria, se tomar cepillado endocervical (Apndice Normativo A) 9.6 En caso de colposcopa no satisfactoria, negativa a LEIBG y con cepillado endocervical negativo, se continuar su control en la clnica de colposcopa en seis meses, con colposcopa y citologa. 9.6.1.1 Si el cepillado endocervical reporta LEIBG se tratar a la paciente como LEIAG, con mtodos conservadores escisionales.

Jueves 31 de mayo de 2007 DIARIO OFICIAL (Primera Seccin) Modificacin a la Norma Oficial Mexicana NOM-014-SSA2-1994, Para la prevencin, deteccin, diagnstico, tratamiento, control y vigilancia epidemiolgica del cncer crvico uterino. Al margen un sello con el Escudo Nacional, que dice: Estados Unidos Mexicanos.- Secretara de Salud. MODIFICACION A LA NORMA OFICIAL MEXICANA NOM-014-SSA2-1994, PARA LA PREVENCION, DETECCION, DIAGNOSTICO, TRATAMIENTO, CONTROL Y VIGILANCIA EPIDEMIOLOGICA DEL CANCER CERVICO UTERINO.

7.3 El resultado del estudio citolgico es descriptivo y debe ser informado de la siguiente manera: a.- Negativo a cncer. b.- Negativo con proceso inflamatorio. c.- Displasia leve (NIC 1). d.- Displasia moderada (NIC 2). e.- Displasia grave (NIC 3). f.- Cncer del cuello del tero in situ (NIC 3). g.- Cncer microinvasor e invasor. h.- Adenocarcinoma. i.- Maligno no especificado.

44.- Acude a consulta femenino de 19 aos con diagnstico de amenorrea, se acompaa de resultados de laboratorio. Reportan los siguientes niveles hormonales: GnRH elevada, FSH y LH elevadas, hormonas ovricas (estrgenos y progesterona) bajas. El defecto est a nivel de: a) b) c) d) Hipotlamo. Hipfisis. Ovario. Endometrio.

Evaluacin de la paciente con amenorrea secundaria La mayora de las pacientes con AS que no estn embarazadas o no entraron en el climaterio tienen una alteracin en algn nivel de la cascada reguladora del ciclo menstrual femenino. A continuacin, esquematizamos la evaluacin de la AS en pasos. Cada mdico deber adaptar los tiempos de la evaluacin a la situacin particular de cada paciente. No obstante, recomendamos no saltear ninguno de estos pasos ya que un diagnstico preciso permitir realizar un tratamiento racional y dar un pronstico respecto de la futura funcin menstrual y de la fertilidad. Primer paso (descartar el embarazo)

Su prueba se basa en el dosaje cuali o cuantitativo de la subunidad beta de la gonadotrofina corinica humana. Si se certifica la ausencia de embarazo podr avanzarse al siguiente paso, pero teniendo en cuenta siempre que si la probabilidad de embarazo es muy alta se deber repetir la prueba nuevamente. Se recomienda no obviar esta prueba aunque la paciente asegure que no ha mantenido relaciones sexuales. Segundo paso (prueba de progesterona)

Debe realizarse slo si el test de embarazo es negativo y consiste en dar medroxiprogesterona entre 30 a 50mg por va oral (un comprimido de 10mg durante 5 das) o progesterona oleosa 100 a 200mg por va intramuscular en una sola dosis. El resultado de esta prueba provee informacin acerca de si se produjeron estrgenos. La prueba se considera positiva si se produce un sangrado luego de 2 a 14 das de la suspensin de la progesterona. La respuesta positiva indica que existe integridad anatmica del aparato genital femenino y que el ovario produce estrgenos. En estos casos, la AS se debe a que no ha habido ovulacin. La causa ms frecuente de AS con prueba de progesterona positiva es la disfuncin hipotalmica leve. En este caso, no se dispara el pico de LH necesario para producir la ovulacin, no se ovula (ciclo anovulatorio), no hay cuerpo lteo y no hay produccin de progesterona.

El sistema reproductor queda en un estado folicular, con grados variables de desarrollo folicular, acompaados de concentraciones cambiantes de estradiol. Puede producirse hemorragia en ausencia de ovulacin dado que el endometrio proliferativo sufrir ocasionalmente un desprendimiento parcial en respuesta a estos niveles cambiantes de estradiol. Estos sangrados luego de un ciclo anovulatorio se denominan sangrados uterinos disfuncionales (SUD). Como dijimos, ejemplos frecuentes de situaciones de anovulacin son el estrs, la prdida brusca de peso y el ejercicio intenso. Otras causas menos frecuentes son la hiperprolactinemia, el hipotiroidismo y el sindrome del ovario poliqustico. Otras veces no hay una causa clara (idioptica) de este trastorno. La prueba se considera negativa si NO se produce un sangrado luego de 2 a 14 das de la suspensin de la progesterona, lo que ocurre cuando los niveles de estrgenos son bajos (menores a 40 pg/ml). La causa ms frecuente de prueba negativa es la insuficiencia gonadal de la menopausia. Tercer paso

Para avanzar al tercer paso es preciso distinguir si la prueba de progesterona fue positiva o negativa.

Pacientes con prueba de progesterona positiva

Una prueba de progesterona positiva hace diagnstico de ciclos anovulatorios. Segn los antecedentes, el examen clnico y la probabilidad previa el mdico podr orientarse hacia cul es el diagnstico. Hay situaciones de estrs, crisis vitales, cambios de peso brusco o ejercicio extremo que ocasionan un trastorno del ciclo aislado. En estos casos, la prueba de progesterona es diagnstica y teraputica. Es decir, la paciente no requiere estudios posteriores. Es conveniente reasegurarla y explicarle que su periodo se normalizar una vez solucionado el problema. En una paciente con trastornos anovulatorios frecuentes, que presenta obesidad, acn e hirsutismo, existe alta sospecha de sndrome de ovario poliqustico (SOP). En este caso, debemos solicitar una ecografa pelviana y un dosaje de LH/FSH o de hormonas masculinas: testosterona libre (To), dehidroepiandrosterona sulfato (DHEA-S) y 17OH progesterona. Si la paciente tiene trastornos del ciclo y galactorrea, se debe solicitar un dosaje de prolactina (PRL). El valor normal de PRL para la mujer no embarazada es de 20 a 25ng/ml. Cuando el valor de PRL es menor de 100ng/ml, generalmente es de causa idioptica o farmacolgica; si est entre 100 y 200ng/ml, puede ser por causa farmacolgica o por enfermedad hipotalmica; si es mayor de 200ng/ml, la causa ms probable es el adenoma hipofisario. Cuando el valor de la PRL es mayor de 100ng/ml, se debe solicitar un estudio por imgenes que puede ser una tomografa computada de cerebro con contraste o una resonancia magntica nuclear con gadolinio.

Si se observa una imagen menor de 10mm, se trata de un microadenoma de hipfisis y si es mayor de 10mm, se hace diagnstico de macroadenoma. Si la paciente consulta por trastornos del ciclo frecuentes y no presenta galactorrea o signos o sntomas de hipotiroidismo, igualmente deber solicitarse un dosaje de tirotrofina (TSH) y de prolactina (PRL) sricas.

Pacientes con prueba de progesterona negativa

Deber dosarse el nivel srico de FSH cuyo valor refleja los niveles de estrgenos circulantes (por el fenmeno de retroalimentacin negativa). Si la FSH es mayor de 40 UI/ml, los niveles de estrgenos son bajos. En este caso, se asume que existe una insuficiencia ovrica. El valor de la FSH puede hacer el diagnstico diferencial entre la falla ovrica (castracin temprana, menopausia precoz o menopausia normal) y la disfuncin hipotlamo-hipofisaria severa. En la primera, la ausencia de retroalimentacin negativa estimula la liberacin de las gonadotrofinas y, en consecuencia, stas se encuentran elevadas en la sangre. En la segunda, los valores de gonadotrofinas son bajos debido a un defecto en su produccin. El dosaje de FSH es ms sensible que el de LH para este propsito, por lo tanto, si la prueba de progesterona es negativa y la FSH es baja, lo ms probable es que la causa sea una disfuncin hipotlamo-hipofisaria severa. En este caso debera solicitarse un estudio de diagnstico por imagen del cerebro (RMN con gadolinio o TAC con contraste). En el cuadro 4 se muestran los valores normales de las hormonas mencionadas. Cuadro 4: valores hormonales normales

PRL (prolactina): 20 a 25ng/ml en mujeres no embarazadas. TSH: de 0.5 a 5mUI/ml. FSH: 5 a 30mUI/ml. LH: 5 a 20mUI/ml (en el pico ovulatorio este valor se debe multiplicar por dos o tres). Falla ovrica: FSH mayor de 40mUI/ml. Disfuncin hipotlamo hipofisaria: FSH menor a 5mUI/ml y LH menor a 5mUI/ml.

En general, una paciente con disfuncin hipotalmica con gonadotrofinas bajas est expresando una gravedad mayor que la disfuncin hipotalmica leve con prueba de progesterona positiva.

En los casos en los que la prueba de progesterona es negativa, la FSH es normal, la paciente no est en una edad cercana a la menopausia y existe alta sospecha de que exista una enfermedad uterina, se recomienda realizar una prueba de estrgenos y progesterona.

Esta prueba sirve para conocer si el efector (el tero) responde a los estmulos hormonales. Consiste en administrar estrgenos (1.25mg por da durante 20 das) ms progesterona (10mg por da durante los ltimos 5 das en que se administran los estrgenos). Si no hay sangrado, se debe repetir la prueba y, si nuevamente no hay sangrado, la prueba se considera negativa. Esto indica que hay una falla en el efector, o sea, en el tero. En este caso la paciente debe ser derivada al gineclogo. Las causas ms probables de falla uterina son la endometritis por abortos o partos spticos y las sinequias por curetajes vigorosos. Cuando falla el efector, la ecografa transvaginal puede ser normal. Por eso debe realizarse una prueba de estrgenos y progesterona para evaluar directamente la funcionalidad del tero. La prueba se considera positiva si hay sangrado (aunque sean slo gotitas). Esto significa que el tero est indemne para responder a estmulos hormonales y el problema es de origen hipotlamo- hipofisario.

Jonathan R, Pletcher, Gail B. Slap. Menstrual Disorders Amenorrhea. Pediatric Clinics of North America. June 1999; 46: Issue 3. 2. Bryan McIver, Susan A Romanski; Todd B Nippoldt. Evaluation and Management of Amenorrhea. Mayo Clinic Proceedings Dec 1997; 72: 1161-1169. 3. Tarannun Master Hunter, Diana L.Heiman. Amenorrhea: Evaluation and Treatment. Am Fam Physician 2006; 73: 1374-1382. 4. The Practice Committee of the American Society for Reproductive Medicine. Current Evaluation of Amenorrhea. Fertility and Sterility Nov 2006: 86 Supl 4: 148-155.

1.

45.- Se trata de mujer de 54 aos G.2 P.2, a quien se ha diagnosticado prolapso uterino acompaado de incontinencia urinaria de esfuerzo, el procedimiento de eleccin en esta paciente es:

a) b) c) d)

Histerectomia total abdominal. Histerectoma vaginal reconstructiva Colpoperineoplasta. Uterosuspensin.

El prolapso genital y su tratamiento ha sido siempre un importante captulo de la ginecologa. No existe consenso clnico del concepto que define al prolapso genital como patolgico. Cierto grado de descenso y relajacin de la pared vaginal es considerado normal en la gran mayora de las mujeres, siendo ms frecuente en mujeres de mayor edad.

En la poblacin general solo un 3% presenta prolapso genital severo, entendiendo como tal al de III y IV grado. Es probable que alrededor de 3 a 6% de la poblacin femenina desarrolle un prolapso de esas caractersticas en algn momento de su vida. La intervencin quirrgica por excelencia para el tratamiento quirrgico del prolapso uterino es la histerectoma vaginal. Esta intervencin se clasifica como "limpia-contaminada

INDICACIONES PARA LA HISTERECTOMA VAGINAL Prolapso uterino Hemorragia uterina disfuncional Carcinoma cervico-uterino "in situ" Miomatosis uterina poco voluminosa Hiperplasia endometrial Piometra

VENTAJAS DE LA HISTERECTOMA VAGINAL No deja cicatriz abdominal Mnimo trauma abdominal Escasa hemorragia transoperatoria Mnima manipulacion intestinal Menos dolor postoperatorio

CUADRO CONTRAINDICACIONES PARA LA HISTERECTOMA VAGINAL Impericia tero muy voluminoso Tumores ovricos Endometriosis Enfermedad plvica inflamatoria Ciruga previa en tero, trompas y ovarios Cncer de endometrio Histerectoma obsttrica

46.- Femenino de 22 aos de edad, cursa con 12 semanas de gestacin, lleva tres das sangrando por genitales, con nuseas continuas y constantes, tero mayor aumentado de tamao y valores de beta HCG muy elevados. El diagnstico ms probable es:

a) Amenaza de aborto. b) Aborto diferido. c) Mola hidatdica. d) Amenaza de aborto en un tero con miomas.

La enfermedad trofoblstica gestacional agrupa a diferentes entidades interrelacionadas: mola completa, generalmente diploide con origen cromosmico paterno, mola parcial generalmente triploide, tumor trofoblstico del lecho placentario y coriocarcinoma, con tendencias variables a la invasin local y a las metstasis, cuyo denominador comn es la hipersecrecin de hCG. El coriocarcinoma es diploide y proviene de ambos progenitores, excluyendo probablemente su origen directo en la mola completa. El tumor trofoblstico del lecho placentario est constituido por trofoblasto mononuclear intermedio no conteniendo vellosidades corinicas e inmunohistoquimicamente caracterizado por expresar muchas de sus clulas hPL y unas pocas hCG.

Cuadro clnico
Tras un periodo de amenorrea, y a partir del segundo mes, hay un aumento de los sntomas subjetivos del embarazo, sobre todo nuseas y vmitos o hipermesis en un 30%. Hay metrorragias irregulares en el 96% de los casos, en principio de escasa cantidad, pero que se va incrementando. Dicha hemorragia no es continua sino que se repite cada dos o tres das, de color roja o negruzca, que se produce por la ruptura de vasos maternos al separarse las vesculas de la decidua. Ocasionalmente, hay expulsin de restos molares, que lo refiere la paciente como expulsin de vesculas en el 11 % de los casos y que es patognomnico pero aparece tardamente. Tambin puede presentar mal estado general, dolor difuso en hipogastrio y anemia. Esta ltima en relacin con las metrorragias. Pueden haber signos y sntomas de hipertiroidismo, como taquicardia, sudoracin, y temblores, en el 7%, y es debido a que la fraccin -hCG es similar a la hormona TSH. Existen signos de preeclampsia o hipertensin gestacional del primer trimestre hasta en el 50% de los casos, y signos de insuficiencia respiratoria aguda en el 2% por embolismo pulmonar de las clulas trofoblsticas, o por la asociacin entre hipertiroidismo e hipertensin arterial. Como complicaciones pueden aparecer coagulopatas y metstasis.

A la exploracin el tero esta aumentado de tamao, en el 50% de los casos, por encima de lo que correspondera a la edad gestacional,3 de consistencia blanda, sin signos de actividad fetal a partir de la semana 12 y siempre que se trate de una mola total. Tambin puede que la paciente presente un tamao uterino menor al esperado para la edad gestacional.3 El cuello est cerrado, con metrorragia en cantidad variable, y raramente se observa la expulsin de vesculas. Pueden haber quistes teca-lutenicos bilaterales en ovario en cerca del 20% de los casos,3 debido al estmulo de la -hCG. La enfermedad trofoblstica maligna va a cursar con metrorragias por lo general intensas, hay una elevacin de la -hCG y en la ecografa se objetiva la cavidad uterina con signos de ocupacin atpica.

Diagnstico:
Por la clnica, y pruebas complementarias como la determinacin de la -hCG y la ecografa. La determinacin de la -hCG se basa en que el trofoblasto produce la hormona gonadotropina corinica, presentando cifras elevadas, y su cuantificacin va a servir para diagnstico, valorar el pronstico, y el seguimiento postratamiento. La ecografa revela un tero aumentado de tamao que no corresponde con la amenorrea, con ecos en su interior, puntiformes que corresponderan a las vesculas y que asemejan copos de nieve o panal de abeja. No se aprecia saco gestacional ni estructuras fetales y, en ambos ovarios se aprecian quistes teca-lutenicos como formaciones ovricas redondas, econegativas, con mltiples tabiques en su interior

BIBLIOGRAFA 1. Mazur MT, Kurman RJ. Gestational trophoblastic disease and related lesions. En: Kurman RJ editor. Blaunsteins pathology of the female genital tract. 4th ed. New York: Springer-Verlag. 1994, p. 1049-93. 2. Kurman RJ, Young RH, Norris HJ, Main CS, Lawrence WD, Scully RE. Immunocytochemical localization of placental lactogen and chorionic gonadotrophin in the normal placenta and trophoblastic tumors, with emphasis on intermediate trophoblast and the placental site trophoblastic tumor. Int J Gynecol Pathol 1984; 3: 101-21. 3. Berkowitz RS, Golstein DP. The management of molar pregnancy and gestational trophoblastic tumours. En Knapp RC, Berkowitz RS, editores. Gynecologic Oncology, 2nd ed. New York: Mc Graw-Hill 1992, p. 328-38. 4. De Agustn P, Ruiz A, Lpez F, Contreras F. Patologa de la enfermedad trofoblstica. Simposio Enfermedad Trofoblstica 1972; 79-98.

5. Salem S. Ultrasound diagnosis of trophoblastic disease. En: Sanders RC, James AE(Jr) editores. Ultrasonography in Obstetrics and Gynaecology. New York: Appleton-Century Crofts: 1977; p. 255-66. 6. Silverberg SG, Kurman RJ. Tumors of the uterine corpus and gestational trophoblastic disease. En: Rosai J, Sobin LJ, editores. Atlas of tumor pathology: tumors of the uterine corpus and gestational trophoblastic disease, fasc. 3, ser. 3. Washington DC: Armed Forces Institute of Pathology; 1992, p. 219-85. 7. Szulman AE, Surti U The syndromes of hydatiform mole II. Morphologic evolution of the complete and partial mole. Am J Obstet Gynecol. 1978; 132: 20-7.

47.- Femenino de 29 aos. Acude al servicio de urgencias por presentar salida de lquido vaginal. Antecedentes: G2, P1, cursa embarazo de 36 semanas de gestacin exploracin fsica: cervix con 10% de borramiento, 1 cm de dilatacin y Tarnier positivo. La complicacin ms frecuente en esta paciente es?

a) b) c) d)

Parto pre trmino. Sepsis neonatal. Corioamnioitis. Endometritis.

Corioamnioitis: El diagnstico de la infeccin intraamnitica (IIA) es bsicamente clnico. La corioamnionitis se debe descartar en toda gestante que presente fiebre sin foco aparente, sobre todo si se sospecha o se ha confirmado una rotura de membranas. Los criterios ms empleados para el diagnstico son: fiebre materna y, al menos, 2 de los siguientes signos: taquicardia materna, taquicardia fetal, irritabilidad uterina, leucocitosis materna o lquido amnitico purulento o maloliente. Progresos de obstetricia y ginecologa: revista oficial de la Sociedad espaola de ginecologa y obstetricia, ISSN 0304-5013, Vol. 48, N. 6, 2005 , pags. 316-317.

48.- Femenino de 22 aos de edad, acude al servicio de urgencias por referir malestar general, fosfenos, nusea y vmito. Antecedentes: cursa con embarazo de 36.5 SDG, G2, A1, C0. Exploracin fsica: TA 185/110 mmHg, FC 120 lpm, FR 35 x, T37.9, alerta, inquieta, aprecia ictericia, cardiopulmonar sin compromiso, abdomen globoso a expensas de tero grvido, se detecta PUVI, longitudinal, ceflico, dorso a la derecha, FCF 142 x, al tacto genital crvix, central, formado, cerrado, extraccin de guantes sin evidencia de prdidas genitales, extremidades inferiores edema +++.

El dato de laboratorio que apoya el diagnstico en esta paciente es: a) b) c) d) Hematuria. Proteinuria. Coluria. Piuria.

Referencia: La preeclampsia se define por aumento de la presin arterial y la presencia de proteinuria durante el embarazo. 1) Leve: las pacientes suelen tener unas cuantas manifestaciones, y su presin arterial diastlica es menor de 110 mmHg. En ocasiones hay edema. La cifra de plaquetas es mayor de 100 000/l. 2) Grave: los sntomas son ms notorios y persistentes. La presin arterial casi siempre es con niveles mayores de 160/110 mmHg. Puede haber trombocitopenia que avance hasta coagulacin intravascular diseminada. Se requiere hospitalizacin para las mujeres con preeclampsia; debe obtenerse una biometra hemtica completa con cifra de plaquetas y determinacin de electrolitos, que adems incluya enzimas hepticas. Se obtiene una muestra de orina de 24 horas para determinar la depuracin de creatinina y protenas totales al ingreso hospitalario. Se debe controlar la hipertensin arterial, para evitar sufrimiento fetal, as como empeoramiento de la paciente. Bibliografa: 1. McPhee S, Papadakis M, et. al. Diagnstico Clnico y Tratamiento 2010. Lange, McGraw Hill, 49 edicin, Mxico, 2010. 2. Sibai BM, Diagnosis, prevention, and management of eclampsia. Obstet Gynecol. 2005. Feb; 105;: 402 410.

49.- Femenino de 33 aos gestante, acude a consulta para realizarse ultrasonido de control. Reportando: Placenta marginal, embarazo de 37 SDG. Antecedentes: G/5 P/4. La complicacin esperada en esta paciente es:

a) b) c) d)

Desprendimiento de placenta normoinserta. Hemorragia materna. Parto pretrmino. Restriccin en el crecimiento intrauterino.

Placenta previa: La placenta en lugar de estar implantada en el cuerpo del tero por fuera del orificio cervical interno, se localizaa por encima del orificio interno o cerca de l. En la placenta marginal el borde la placenta est en el margen del orificio. La multiparidad y la edad avanzada parecen favorecer la placenta previa. El hecho ms carcterstico de la placenta previa es la hemorragia no dolorosa, la cual no suele aparecer hasta casi del final de segundo trimestre o ms tarde. Complicaciones maternas: Hemorragia y muerte. Pritchard, Williams Obstetricia, 3Edicin, Salvat editores, pag. 395-39

50.- Acude femenino de 55 aos, se queja de presin plvica y una masa en la entrada vaginal, hace 3 aos tuvo su ltimo periodo menstrual. No tomo terapia de reemplazo, tiene dificultad para evacuar. Tiene una tos crnica y una historia de tabaquismo positivo a razn de 30 cajetillas ao. Ha tenido 3 partos y el ultimo bebe pes 4,500 gramos. Tiene una orina de 60 cc. Cul de los siguientes hallazgos es ms probable que encuentres en la exploracin plvica?

a) b) c) d)

Rectocele Cistocele Enterocele Uretrocele

El Rectocele es una hernia de la pared anterior del Recto hacia la porcin posterior de la vagina. La incidencia real de esta deficiencia anatmica es desconocida y en muchas ocasiones es un resultado del paso del tiempo. Es un hallazgo muy frecuente del examen perineal, siendo en mltiples ocasiones asintomtico. El rectocele puede ser un hallazgo importante del sndrome de Obstruccin Defecatoria (SOD). No debe tomarse como una deficiencia anatmica nica, sino como parte importante de un problema anatomo-fisiolgico complejo.

Un principio importante es la etiologa y anatomo-patologa del rectocele. Existen varias teoras sin consenso principal. La existencia, deficiencia o alteraciones del septo rectovaginal son controversias importantes. No existe una fascia visceral que separe el recto de la vagina o que forme un septo especfico. Existe frontera entre donde termina la pared anterior del recto y donde comienza la pared de la vagina, pero mltiples estudios no han encontrado un septo rectovaginal especfico. El septo puede estar formado de una pelcula casi transparente hasta una pared de consistencia fibromuscular fuerte. El rectocele es un hallazgo comn. Se presenta en el 80% de las pacientes femeninas y 13% de los masculinos en una defeco grafa (> a 1cm.) Entre mas grande es el rectocele, mayor son los sntomas asociados, Dificultad en la evacuacin, constipacin crnica, dolor rectal y perineal, sensacin de masa y en ocasiones sangrado. La necesidad de presin manual para ayudar a la evacuacin o para vaciar el rectocele es comn en ms del 50% de los pacientes. El diagnostico se hace con un simple tacto rectal e inspeccin vaginal, pero debe recordarse que rara vez es el rectocele un hallazgo aislado. La presencia cistocele, peritoneocele, enterocele u otros prolapsos perineales y problemas funcionales deben ser descartados antes de proponer la reparacin quirrgica del rectocele. Defeco grafa, Pruebas de funcin fisiolgica del piso plvico, pruebas de funcin urinaria, evaluacin del esfnter anorectal por ultrasonido y hasta la resonancia magntica han sido propuestos antes de la ciruga.

Bibliografa:
Rectocele: Pathogenesis and surgical managment. Zbar AP, Linemann A, Fritsch H,
Beer-Gabel M, Pescatori M. Int J Colorectal Dis. (2003) 18:369-384. Evaluation and Treatment of Women with rectocele. Cundiff GW, Fenner D, Obstetrics and Ginecology 104(6): 1403-1416.

Stapled transanal rectal resection to treat obstructed defecation caused by rectal intussusseption and rectocele. Renzi A, Izzo D, Di Sanrno (26) 21:661-667. Rectocele repair using biomaterial augmentation. Altman D, Melgren A, Zetterstrom J.
Obstet Gynecol (2005) 60(11)753-760.

51.- Mujer de 26 aos, con tumor anexial de 6 cm lquido, dolor abdominal, fiebre, leucorrea, con historia de cervicovaginitis de repeticin y dispareunia crnica, ltima menstruacin hace una semana. El manejo ms adecuado es: a) Histerocopa b) Colposcopa c) Histerosalpingografa

d) Laparotoma exploradora

El avance de la laparoscopia en los ltimos aos ha hecho que se convierta en una tcnica fundamental en el diagnstico y en la estadificacin de la EIP, pero adems nos ofrece la posibilidad de realizar endoscopia quirrgica por medio de una serie de procedimientos que nos permitiran abreviar y mejorar la evolucin natural de la enfermedad. Los procedimientos quirrgicos que podemos realizar durante la laparoscopia son: 1. Liberacin de adherencias: Es indispensable en muchos casos para realizar el diagnstico y puede ser dificultosa en funcin de la antigedad del proceso y de la laxitud de dichas adherencias. En los casos de adherencias periheptica se ha descrito mejora del cuadro clnico al liberarlas mediante lser. 2. Drenaje de los abscesos: Son diversos los procedimientos que podemos realizar laparoscpicamente pero el ms frecuente es la apertura de la trompa cerca de la fimbria con aspiracin del contenido purulento y posteriores lavados. 3. Extirpacin de la enfermedad. 4. Lavados peritoneales: Con sistemas de aspiracin-irrigacin con povidona yodada diluida han sido recomendados o con suero salino. 5. Establecer el diagnstico diferencial con otras entidades. Biopsia de endometrio: La presencia de clulas plasmticas en biopsia endometrial sugiere EIP.

CLASIFICACION LAPAROSCOPICA DE LA ENFERMEDAD PELVICA INFLAMATORIA El stndar de oro para el diagnstico de EPI es la laparoscopia, ya que adems de visualizar directamente los rganos plvicos, permite la toma de muestras para estudios bacteriolgicos. Los criterios laparoscpicos para el diagnstico de EPI, se describen en la tabla No.3. Tabla No. 3 CLASIFICACION LAPAROSCOPICA DE LA EPI Leve Eritema, edema, las trompas se mueven libremente. No hay exudado purulento. Eritema, edema ms marcado, material purulento evidente. No hay movimiento libre de las trompas. La fimbria puede no ser evidente.

Moderada

Severa

Presencia de pioslpinx y/o absceso

REFERENCIAS BIBLIOGRFICAS 1. Barie P, Hydo L, Eachempati S. Longitudinal outcomes of intra-abdominal infection complicated by critical illness. Surg Infect. 2004;5:365-73. 2. Boucher H, Talbot G, Bradley J, Edwards J, Gilbert D, Rice L, et al. Bad Bugs, No Drugs: No ESKAPE! An Update from the Infectious Diseases Society of America. Clin Infect Dis. 2009;48:1-12. 3. Washington AE, Katz P. Cost and payment source for pelvic inflammatory disease. JAMA. 1991;266:2565-9. 4. Center for Disease Control. Guidelines for treatment of sexually transmited diseases. MMWR Morb Mortal Wkly Rep. 2006;55 (RR-11):56. 5. Jossens MO, Shachter J, Sweet RL. Risk factors associated with pelvic inflammatory disease of diflering microbial etiologies. Obstet Gynecol. 1994;83:989-97. 6. Botella Llusi J, Clavero Nez JA. Tratado de Ginecologa. 14ta ed. Madrid: Ediciones Daz de Santos; 1993. 7. Berek J, Berek S. Novak's Gynecology. 14th ed. Philadelphia,Pa: Lippincott Williams & Wilkins; 2007. 8. Adimora A, Hamilton H, Holmes K, Sparling PF. Pelvic Inflammatory disease in sexually transmitted diseases. 2nd International Edition. New York: Mc Graw-Hill Inc. 1994. p. 254270.

52.- Femenino de 25 aos, con embarazo de trmino, sin antecedentes de control prenatal. G 3. C-1. Se ingresa al servicio de obstetricia por presentar actividad uterina regular y dolorosa. Ef.: Deambulante, tranquila, adecuada coloracin de tegumentos, abdomen con fondo uterino a 32 cm. con producto nico vivo en situacin transversa dorso inferior FCF 144, al tacto vaginal crvix dilatado a 3 cm. y membranas ntegras. Se realiza cesrea con retencin de placenta e invasin a vejiga. La alteracin placentaria que presenta esta paciente es: a) b) c) d) Placenta increta Placenta acreta Placenta percreta Placenta marginal

Es la penetracin y adherencia anormal de la placenta en la pared uterina. Se divide en: Placenta acreta. Placenta increta. Placenta percreta. ACRETA: Las vellosidades se adhieren al miometrio. INCRETA: Penetran ms de la mitad del espesor del miometrio. PERCRETA: Atraviesa todo el espesor del miometrio, llegando a la serosa, incluso atravesndola y adhirindose a rganos vecinos.

Factores: Endometrisis previa. Tumores submucosos. (Miomas) Cicatrz uterina previa. (Cesrea, miomectoma) Implantacin baja. (Placenta previa) Malformaciones placentarias. (Placenta extracorial) Legrado enrgico previo. Extraccin manual previa de una placenta.

Diagnstico transparto-: Placenta retenida por ms de 20 minutos. Imposibilidad para encontrar un plano de separacin placentaria cuando se intenta su extraccin manual. Hemorragia incontrolable despus de la pseudoextraccin. El diagnstico histopatolgico corrobora el diagnstico clnico. Escenario menos deseable. Tratamiento: Histerectoma Obsttrica. Constituye una ciruga no planeada y secundaria al hallazgo del acretismo placentario con sangrado incohercible. Cesrea-Histerectoma. (Con diagnstico previo) Ciruga planificada ante un correcto diagnstico prenatal.

Recomendacin ACOG: Maduracin pulmonar intrauterina. Inyectar al cordn umbilical 50 mg de metrotexate. Ligar el cordn en el nacimiento placentario y dejar la placenta in-situ. Embolizacin inmediata de arterias uterinas bilaterales, as como de ramas de la divisin anterior de la arteria iliaca interna con alcohol polivinlico. Continuar con 5 dosis I.M. de 50 mg de metrotexate y cuantificar niveles de hCG. Programar Histerectoma Total Radical Abdominal y/o Cistectoma parcial y/o reseccin pared anterior recto.

Lee et al. Conservative Management of Placenta Percreta. Obstet Gynecol, 112(2):421-424

53.- Se presenta paciente de 40 aos de edad a su consultorio refiriendo mastalgia que es ms severa antes de la menstruacin. A la palpacin hay nodularidad excesiva, hiperestesia y reas qusticas que la paciente refiere disminuyen en tamao despus de la menstruacin. El diagnstico ms probable es:

a) b) c) d)

Fibroadenomas Papiloma intraductal Mastopata fibroqustica Cncer de mama

Es raro encontrar una mujer mayor de 35 aos a quien no le hayan dicho, en un examen fsico mamario, ecogrfico o mamogrfico, que tiene quistes en la mama o que su mama es mastoptica. Es el trastorno benigno de la mama ms frecuente y consiste en un aumento del tejido mamario, especialmente en las zonas superiores y externas de las mamas, hacia las axilas, que las hace ms densas. La mastopata fibroqustica suele presentarse en ambas mamas, aunque puede ser de diferente intensidad en una que en otra.

Puede presentarse a cualquier edad despus del inicio de la menstruacin, pero es ms probable que aparezca entre los 30 aos y la menopausia. Raramente se presenta ms tarde de esa edad.

El origen de este trastorno es funcional y responde a desequilibrios de las hormonas sexuales femeninas y puede condicionar la aparicin de quistes mamarios.

Los sntomas pueden fluctuar de leves a severos en una mastopata fibroquistica mamaria, se acentan tpicamente antes de cada perodo menstrual y desaparecen inmediatamente despus. Los sntomas abarcan: Consistencia de protuberancias (como de "guijarros"), irregular y densa del tejido mamario o generalmente ms notoria en la parte superior externa de la mama Molestia en las mamas o generalmente en ambas mamas o puede ser persistente o puede aparecer y desaparecer Sensacin de llenura en las mamas Sensibilidad y dolor sordo e intenso Sensibilidad y edema premenstrual Secrecin ocasional del pezn

Bibliografa:
Jones III HW, Wentz AC. Tratado de Ginecologa de Novak. Editorial Interamericana-McGraw Hill. Undcima Edicin 1994. DiSaia-Creasman. Oncologa Ginecolgica Clnica. Editorial Mosby. Cuarta Edicin 1994. Van Dinh T. Sumario de Patologa Ginecolgica. Editorial La Prensa Mdica Mexicana. 1992. Pernoll ML. Diagnstico y Tratamiento Ginecoobsttricos. El Manual Moderno. Mxico. Sexta Edicin 1991. Alvarez-Bravo A. Diagnstico de los trastornos menstruales y hemorrgicos. En: Alfonso Alvarez Bravo y su obra. Editorial Marketing y Publicidad SA. Tomo I. 1993. Vzquez E. Aspectos histoqumicos del endometrio humano despus del tratamiento con progestgenos sintticos. Gac Md Mx 1966; 96: 1277-93. Huerta MR, Malacara JM, Rivera-Cisneros A, Daz Cisneros FJ. Sntomas en adolescentes de dos ciudades de Mxico y su asociacin con el ciclo menstual. Ginec Obstet Mx 1994; 62: 146-50.

54.- Se trata de femenino de 32 aos, acude a consulta por elevacin de T/A AGO. G/2 P/1 se diagnostica preclampsia leve, el frmaco de eleccin que se administra en esta patologa es: a) b) c) d) Nifedipina. Inhibidores de la enzima convertidora de angiotensina. Clonidinas. Alfametildopa.

Prevenir complicaciones a corto plazo de las mujeres con PA elevada que comprometa el bienestar fetal Cuando la PAS es mayor o igual a 150 mmHg y la PAD mayor o igual a 100 mmHg. El propsito es alcanzar cifras de TA alrededor de 140/90. La medicacin antihipertensiva se reserva para los casos en que la PAD 100 mmHg. Se recomienda continuar el tratamiento antihipertensivo previo al embarazo, exceptuando el uso de IECA. La alfametildopa y la hidralazina va oral son los frmacos de eleccin dado su uso extensivo con seguridad y eficacia y sin efectos colaterales para el feto (excepto hidralazina en lupus).

ALFA METILDOPA 500-2000 MG/DA HIDRALAZINA 50-200 MG/DA LABETALOL 100-400 MG/DIA ATENOLOL 50-200 MG/DA NIFEDIPINA 10-30 MG/DA

1. Aagard K, Belfort M. Eclampsia: Morbility, mortality, and management. Clin Obstet Gynecolol. 2005; 48: 12-23. 2. Oyarzn E. Sndrome hipertensivo del embarazo en Oyarzn E. Ed. Embarazo de alto riesgo. Ediciones Universidad Catlica de Chile. Santiago. 1997: 157-175. 3. Roberts J, Redman C. Pre-eclamsia: More than pregnancy induced hypertens

55.- Paciente de 25 aos, Gesta 1, Para 1. Con dos citologas lesin de alto grado, prueba de Schiller positiva y biopsia de crvix que demuestra carcinoma In Situ. La conducta es: a) b) c) d) Conizacin. Histerectoma total abdominal. Histerectoma y salpingooforectoma bilateral. Electrocauterizacin del crvix.

La conizacin cervical es el tratamiento de eleccin en pacientes con cncer cervicouterino microinvasor y ms si existedeseo de fertilidad. Asimismo, la histerectoma extrafasciales un mtodo adecuado en lesiones de 0.5 a 3 mm de invasin.Adems se propone que, para pacientes con lesiones de 3.1 a 5 mm de invasin, a partir de la membrana inicial sinfactores de mal pronstico como invasin vascular y linftica, sean tratadas con histerectoma extrafascial, ya que en aquellasa las que se realiz linfadenectoma plvica, con este tipode lesin, no se encontr metstasis a ganglios linfticos.

Resultados del tratamiento en cncer cervicouterino microinvasor en el Instituto Nacional de Cancerologa de Mxico (1980-1999) 1.- Mestwerdt G. Fruhdiagnose des Kollumkarzinoms. Zentralb Gynaekol, 1947 ;69 :326. 2. - Morrow CP, Curtin JP. Surgery for cervical neoplasia. In Gynecologic Cancer Surgery. New York, Churchill Livingstone, 1996, p 472.3. 3. - Burghardt E, Holzer E. Diagnosis and treatment of microinvasive carcinoma of the cervix uteri. J Obstet and Gynecol 1977; 49:641-653. 4.- Sedlis A, Sall S, Tsukada Y, et al. Microinvasive carcinoma of the uterine cervix: a clinical-pathologic study. Am J. Obstet Gynecol. 1979;133:64.

56.- The Virchow triad of thrombus in the etiology of pulmonary embolism is constituted by: a) b) c) d) Venous stasis, endothelial injury and hypoxemia. Hypoxemia, hypercoagulability and cough Venous stasis, hypercoagulability, and endothelial damage. Dry Cough

Tromboembolismo pulmonar Etiologa: El 90% de los casos de tromboembolismo pulmonar tienen su origen en venas de las extremidades. Es favorecida por la triada de Virchow: stasis venosa, dao de la ntima, aumento de la coagulabilidad (puerperio, ciruga mayor, cncer, uso de anticonceptivos, policitemia vera, sndrome de hipercoagulabilidad (deficiencia de PC, PS, ATIII, resistencia a la protena C). Fisiopatologa. Efectos respiratorios: aumento del espacio muerto alveolar, broncoconstriccin, taquipnea e hipoxemia, alteracin ventilacin/perfusin (V/Q) por redistribucin de flujo. Efectos hemodinmicos: La reduccin mecnica leve o moderada no produce aumento de la resistencia en forma significativa. Sobre el 50% de los casos presentan incremento brusco de la resistencia y presin. Al efecto del mbolo se suman las aminas liberadas por las plaquetas. En el 60-70% se desencadena cor pulmonale agudo con disminucin brusca del gasto cardiaco. Manifestaciones clnicas del tromboembolismo pulmonar: Disnea, Taquipnea, Dolor pleurtico, Crepitantes, Tos, Taquicardia, Hemoptisis, broncoespasmo, cianosis, sncope, palpitaciones. Sndromes: disnea aguda de causa desconocida, hemoptisis y/o dolor pleurtico, shock cardiognico. British Thoracic Society, Standards of Care Committee. Suspected acute pulmonary embolism: A practical approach. Thorax 1.997; 52 (suppl 4): S1-S23.

The PIOPED Investigators. Value of the ventilation/perfusion scan in acute pulmonary embolism: results of the Prospective Investigation of Pulmonary Embolism (PIOPED). JAMA 1.990; 263: 2.753-9. Goodman PC. Spiral CT for pulmonary embolism Sem Resp Crit Care Med 2.000; 21(6): 503-10.

57.- Masculino de 47 aos de edad que cursa con un ataque agudo de gota, la articulacin que se afecta con mayor frecuencia en ste caso es: b) La rodilla c) La mueca a) La primera metatarso-falngica d) El codo

El American College of Rheumatology tiene 11 criterios, y la presencia ms o menos de seis sugiere la presencia de gota. Los 11 criterios son: 1. Ms de un ataque de artritis activa 2. Inflamacin mxima desarrollada a lo largo de un da. 3. Ataque de oligoartritis. 4. Enrojecimiento observado en la articulacin. 5. Primera articulacin metatarsofalngica dolorosa o inflamada 6. Ataque unilateral de la primera articulacin metatarsofalngica 7. Ataque unilateral de la articulacin del tarso 8. Tofo (probado o sospechado) 9. Hiperuricemia 10. Hinchazn asimtrica en la radiografa dentro de una articulacin 11. Terminacin completa de un ataque.

Bibliografa:
Coll JM, Blanch J. Hiperuricemia y gota. Aspectos teraputicos. Jano 1997;1201:4752. Gonzlez Barber A, Carlavilla AB. Gua en Hiperuricemia y Gota. Madrid: EDIMSA, 2003. Nadal A. Farmacovigilancia. Alopurinol: aumento en la incidencia de episodios agudos de gota. Offarm 2000;19(1):128. Prez Ruiz F, Calabozo Raluy M, Ugalde Espieria J, Herrero Beites AM. Artropatas microcristalinas I. Hiperuricemia y gota. Medicine 2001;8(34):1765-72. Snchez Pozo A, Faus MJ. Hiperuricemia y gota. Pharm Care Esp 2003;5:105-9.

Sancho Bueso T, Bernardino de la Serna I, Garca Puig J. Consulta diaria. Qu hara usted ante un paciente con hiperuricemia? Medicina Integral 2000;3(35):10011. Sancho Bueso T, Garca Puig J. Criterios teraputicos ante la hiperuricemia. Revista Clnica Espaola 2001;2(201):85-7.

58.- Cuando se observa una imagen histolgica de acantosis con elongacin de las crestas interpaplares que incluso se fusionan entre s, hiperparaqueratosis y acmulos epidrmicos de leucocitos polimorfonucleares, estamos hablando de:

a) b) c) d)

Psoriasis. Icitiosis. Dermatitis (eccema). Epidermlisis.

Psoriasis:
La psoriasis es una enfermedad cutnea caracterizada por un curso crnico, que cursa a brotes y tiene hallazgos clnicos variables. Las manifestaciones cutneas de esta enfermedad son tan caractersticas (Tabla 1) que el diagnstico suele realizarse con facilidad. En la actualidad se contempla la psoriasis como una enfermedad de base inmunolgica, mediada por los linfocitos T, que asocia inflamacin drmica y secundariamente hiperplasia epidrmica. Es probablemente la enfermedad dermatolgica mediada inmunolgicamente ms prevalente. Tabla 1 Caractersticas clnicas de la psoriasis 1. 2. 3. 4. 5. Placa eritematosa Escamas blanquecinas Buena delimitacin Signo de Auspitz Fenmeno de Koebner

Psoriasis. Hiperparaqueratosis con acmulos intracrneos de neutrfilos (microabscesos de Munro). HE, x 400

Cambios histolgicos de la psoriasis: Infiltracin de clulas mononucleares Hiperplasia epidermica Acantosis epidermica regular, con hipogranulosis, hiperqueratosis y paraqueratosis Atrofia epidermica suprapapilar Infiltracininflamatoria Pustula espongiforme de Kogog Microabscesos de Munro Infiltrado de linfocitos CD4 y CD8 en dermis Cambios vasculares Proliferacin de vasos capilares dilatados ocupando las papilas

Bibliografa:
Arnold HL, James WD, Odom RB. Andrews : tratado de dermatologa. 1993. 4 ed. Barcelona: Masson; 1993 Ashcroft DM, Po AL, Williams HC, Griffiths CE. Systematic review of comparative efficacy and tolerability of Calcipotriolin treating chronic plaque psoriasis. BMJ 2000; 320:963-967. [Medline][Texto completo] Chalmers RJG, O'Sullivan T, Owen CM, Griffiths CEM. Intervenciones para la psoriasis guttata (Revisin Cochrane). En: La Cochrane Library Plus, Nmero 2, 2002. Oxford: Update Software Ferrndiz Foraster C. Dermatosis eritematoescamosas (I). Psoriasis. Eritrodermias. En: Ferrndiz C, ed. Dermatologa Clnica.2 ed. Madrid: Harcourt; 2001. p. 165-175. Freedberg I, Eisen A, Katz SI, Wolff K, Fitzpatrick TB, Goldsmith LA et al, eds. Dermatology in general medicine. 5th ed. New York: McGraw-Hill; 1999 Riffiths C E, Clark C M, Chalmers R J, Li Wan Po A, Williams H C. A systematic review of treatments of severe psoriasis. Health Technology Assessment, 2000;4(40):1-125. Naldi L, Rzany B. Chronic plaque psoriasis. Clin Evid 2002; 7: 1488-1507.

59.- Masculino de 66 aos que cursa con postinfarto agudo de miocardio una medicacin generalmente indicada, por disminuir la mortalidad, es: a) b) c) d) Nitritos. Betabloqueadores. Anticoagulacin oral. Antiarrtmicos.

Los BB son recomendados en todos los pacientes que han presentado un IAM,

siempre que no tengan contraindicaciones para su uso, y de modo permanente (indefinidamente): clase de recomendacin I, nivel de evidencia A. Se ha puesto en evidencia que los BB son infrautilizados en esta indicacin. Ms de 35.000 pacientes han sido incluidos en estudios postinfarto con BB. Se ha demostrado una reduccin de la mortalidad total, muerte sbita y reinfarto

del orden del 20-25%.

Situacin clnica/indicacin Todos los pacientes contraindicaciones, indefinidamente Para mejorar supervivencia Para prevenir reinfarto Prevencin primaria de la muerte sbita Prevencin/tratamiento ventriculares tardas de arritmias sin

Clase de Nivel recomendacin evidencia

de

I I I IIb

A A A B

Los BB siguen estando indicados en el tratamiento de la HTA, aunque existen

crticas a su empleo en el primer escaln.

Pese a ello, son sin duda de primera eleccin si el/la paciente presenta angina, cardiopata isqumica en general, insuficiencia cardaca, taquiarritmias, glaucoma o embarazo. Los BB son frmacos de primera eleccin en la insuficiencia cardaca con disfuncin

sistlica. Slo el carvedilol, el bisoprolol o el nebivolol son BB que puedan ser empleados a las dosis disponibles en nuestro medio para dicho tratamiento.

Todos los pacientes sin contraindicaciones deben recibir BB si presentan

cardiopata isqumica y, muy especialmente, si existe angina o han presentado un IAM.

ESC Expert consensus document on b-adrenergic receptor blockers. The Task Force on Beta-Blockers of the European Society of Cardiology. Eur Heart J 2004; 25: 13411362.

60.- Ante un paciente al que se ha determinado una deficiencia intrnseco, se puede traducir en el siguiente tipo de anemia: a) Micorctica b) Drepanoctica c) Talasmica d) Perniciosa

deficiencia de factor

Carencia del factor intrnseco El factor intrnseco es una sustancia natural que normalmente se encuentra en el estmago y es necesaria para absorber la vitamina B12 de los alimentos. Una carencia del factor intrnseco ocasiona anemia perniciosa y deficiencia de vitamina B12, lo cual puede causar anemia y problemas del sistema nervioso y del cerebro (neurolgicos). Las causas ms comunes de anemia perniciosa abarcan: Debilitamiento del revestimiento del estmago (atrofia de la mucosa gstrica) El sistema inmunitario ataca las clulas que producen el factor intrnseco (autoinmunidad contra las clulas parietales gstricas) Autoinmunidad contra el factor intrnseco en s

El comienzo de la enfermedad es lento y puede tomar dcadas para establecerse por completo. Aunque la forma congnita ocurre en nios, la anemia perniciosa por lo general no aparece antes de los 30 aos en adultos y la edad promedio del diagnstico es a los 60 aos. Referencias Antony AC. Megaloblastic anemias. In: Goldman L, Ausiello D, eds. Cecil Medicine. 23rd ed. Philadelphia, Pa: Saunders Elsevier; 2007: chap 170.

61.- Femenino de 33 aos que cursa con 38.5 semanas de gestacin que ingresa al servicio con trabajo de parto. En el transcurso de trabajo de parto durante la dilatacin presenta dolor intenso y brusco. A la exploracin usted observa metrorragia escasa y aumento del tono uterino a la palpacin abdominal que resulta muy doloroso. El diagnstico ms probable es:

a) b) c) d)

Placenta previa. Desprendimiento de placenta. Rotura de vasos previos. Crioamnionitis hemorrgica.

Fisiopatologa de la hemorragia La hemorragia es el signo fundamental que domina el cuadro clnico de la placenta previa. Para explicar su mecanismo existen distintas teoras: Mecanismo de Jacquemier: Se produce crecimiento armnico de la placenta y del tero hasta la semana 26, 28. Despus el segmento inferior crece ms deprisa y favorece el despegamiento lo que origina la hemorragia en el embarazo. Mecanismo de Schroeder: Las contracciones uterinas en el parto traccionan del segmento inferior hacia arriba y empujan al feto hacia abajo despegando la placenta. Mecanismo de Pinard: Explica las hemorragias gestacionales y del parto. El estiramiento de las membranas de la zona de menor radio (orificio interno cervical) como consecuencia de las contracciones tira de la placenta y la desprenden. Mecanismo de Bartholomew: Explica la hemorragia en los casos de placenta previa central. La zona placentaria que reviste el orificio interno es un rea isqumica ya que no recibe vasos deciduales. A este nivel disminuye la presin sangunea, por lo que la sangre tiende a dirigirse hacia esta zona y escapa por la cara materna. En el alumbramiento tambin puede haber una hemorragia importante producida por un doble mecanismo: -desprendimiento parcial antes de la expulsin en los casos de placenta oclusiva. -atona uterina en la zona de insercin despus de expulsada la placenta y vascularizacin anmala. Manifestaciones clnicas de la placenta previa en el embarazo - Sntomas: Principalmente la hemorragia. Toda hemorragia vaginal acontecida en el tercer trimestre debe hacer pensar en una placenta previa. Las hemorragias suelen ser espontneas, no acompaadas de dolor, de sangre roja y se presentan de forma intermitente, con intervalos variables entre las mismas. Progresivamente se van haciendo ms frecuentes y ms graves. La primera hemorragia suele aparecer en forma inesperada generalmente nocturna, cesando en menos de media hora. Las hemorragias ulteriores son ms graves y ms precoces.

- Signos: La consecuencia fundamental es la anemia materna que depende de la cuanta de la hemorragia (la sangre es de origen materna ya que procede de espacios intervellosos). - Exploracin: Exploracin general para valorar la existencia de signos de anemia. Exploracin obsttrica: valorar el tamao del tero (adecuado para la edad gestacional), es blando e indoloro. A menudo la esttica fetal est alterada (transverso, oblicuo, nalgas). No debe efectuarse tacto vaginal cuando haya existido hemorragia en embarazo avanzado por el riesgo de infeccin y de despegamiento y aumentar as la hemorragia. Auscultacin fetal normal. Exploracin ecogrfica: es una tcnica fundamental en el diagnstico de la placenta previa. Permite determinar la localizacin placentaria y la variedad de la placenta previa. En general, la placenta puede identificarse a partir de la 9 semana. No obstante, a lo largo de la gestacin por crecimiento uterino se produce un cambio en sus relaciones con el tero ("emigracin placentaria", imagen de desplazamiento). As el diagnstico de certeza de lmites placentarios solo puede establecerse hacia la semana 34. Siempre ser necesario hacer una adecuada identificacin del orificio cervical interno (ms fcil con sonda transvaginal). Aadiendo al estudio Doppler color se observa la vascularizacin y las zonas que sangran.

62.- Una mujer de 23 aos de edad G/1, tuvo un parto vaginal espontneo con un producto con peso de 4,350 g. despus de 5 minutos de traccin suave del cordn umbilical se expuls la placenta, que parece estar intacta. Se inici el masaje del fondo uterino y se pidi a la enfermera que administrara 20 unidades de oxitocina en 100 ml de solucin Ringer lactato. Despus de una inspeccin cuidadosa del canal del parto se observa una laceracin de segundo grado y una laceracin de 2 cm en la pared vaginal izquierda que se intent reparar. En la E.F.se encuentra un fonso uterino blando y atnico. Los signos vitales son: temperatura 37.1C, TA 164/92, FC 130x, FR 18 X. Cul de los siguientes es el mejor tratamiento? a) Oxitocina 10 unidades directas en goteo intravenoso b) Metilergonovina 0.2 mg IM c) Prostaglandina F 0.25 mg IM d) Legrado

Morgan M, Siddighi S. Ginecologa y obstetricia, National Medical Series. 5 edicin. Mc Graw Hill. Pp. 28. La atona uterina es la causa ms comn de hemorragia puerperal. El masaje energtico y la oxitocina diluida no han sido tiles para interrumpir la hemorragia y por tanto el siguiente paso es agregar un frmaco uterotnico. La metilergonovina est contraindicada porque la paciente se encuentra hipertensa a pesar de la hemorragia intensa, el siguiente frmaco es la prostaglandina. La administracin de oxitocina no diluida, 10 UI por va IV podra causar hipotensin grave. La exploracin manual podra ser apropiada si se sospecha laceracin como causa de hemorragia. El legrado es apropiado para la hemorragia puerperal tarda, cuando se sospecha retencin de los productos de la concepcin.

63.- Femenino de 37 aos de edad, G-3, C-2. Es ingresada a hospital presentando cefalea, acfenos, fosfenos y epigastralgia en barra con embarazo de 34 semanas. E.F T/A 160/110, FC 84 x, FR 18 x, no presenta fiebre, somnolienta, sin agregados cardioventilatorios, hepatalgia. F.U. de 25 cm. Producto nico vivo. FCF 110 lpm, genitales sin prdidas ni modificaciones cervicales. Laboratorio: hb 9.8 g/dl, plaquetas de 54 mil, TP 11 seg TPT 27, TGO 160 ng/dl TGP 160 ng/dl, hiperbilirrubinemia indirecta, albuminuria 300 mg/dl, Acido rico de 8.1 mg/dl, creatinina de 1.5 mg/dl. El diagnstico ms probable es:

a) b) c) d)

Sx anticuerpos antifisfolpidos Sindrome de hellp Prpura trombocitopnica trombtica Hgado graso

DEFINICIN: Es una complicacin de la preeclampsia en la cual adems de la Hipertensin Arterial y proteinuria hay presencia de anemia hemoltica, enzimas hepticas elevadas y recuento bajo de plaquetas

MANIFESTACIONES CLINICAS: Malestar general, fatiga y molestias inespecficas 90% Cefalea 70% Epigastralgia 64% Vmito 22% Fosfenos 15% Visin Borrosa 11%

Acfenos 3% Ictericia Anemia no explicada Oliguria

BIBLIOGRAFIA: Sibai baha, El sndrome HELLP. Universidad de Valencia , revista quincenal de Obstetricia clnica y ginecologa, Octubre 2003. V. Cararach, Sndrome de HELLP y Repercusiones maternas. X curso intensivo de formacin continuada materno fetal. Enero de 2003. Toirac, Abelardo. Sndrome de Weistein HELLP Hospital Ginecoobstetrico Tamara Bunke. Junio 2002 De la Fuente, David. Sndrome HELLP. Medicina Universitria 2003; 5 (19): 101 -9 Andrea G. Witlin, DO, Baha M. Sibai, MD. Diagnosis and Management of women with Hemolysis Elevate Liver Enzymes, and Pletelet Count (HELLP) syndrome. Hospital Physician. Febrero 1999. CIFUENTES B, Rodrigo. Ginecologa y obstetricia.

64.- Se trata de paciente femenino de 29 aos de edad que cursa con 34 SDG, G2. C.1 inicia con datos compatibles de amenaza de parto pre-trmino y ruptura prematura de membranas La principal causa de esta patologa est relacionada estrechamente al siguiente diagnstico: a) b) c) d) Traumatismo. Idioptico. Infeccin de vas urinarias. Infeccin vaginal por cndida albicans.

La infeccin de vas urinarias es la complicacin infecciosa ms frecuente del embarazo. De hecho, las mujeres son ms susceptibles a la infeccin de vas urinarias debido a los siguientes factores: Una uretra ms corta. Fcil contaminacin de la uretra por bacterias de la vagina y el recto. Posibilidad de la que la mujer no vace por completo la vejiga cada vez que orina. Movimiento de bacterias al interior de la vejiga con cada relacin sexual.

Se suman adems, los cambios que el propio embarazo produce en el aparato urinario como son la relajacin del msculo liso de los urteres que impide que la orina llegue adecuadamente a la vejiga para ser eliminada, y la compresin que ejerce la matriz sobre la vejiga lastimando su cubierta interna y dejndola incapacitada para vaciarse por completo.

Bibliografa: 1. Velasco MV. Prevencin y tratamiento del parto pretrmino. Lo nuevo acerca del viejo problema Rev Med IMSS 2001, 39 (%) 417-42. 2. Lastra ELG El parto pretrmino como problema de salud pblica .Perinatol Reprodud. Human. Vol 15 No 2.Abril-junio 2001 113-14.

65.- Femenino de 28 aos G/4, P/0 con 6 SDG acude a su primera visita prenatal. Su historia obsttrica pasada es importante porque tiene tres prdidas de producto en el segundo trimestre. Refiere que en las tres ocasiones al presentarse al hospital presentaba dilacin cervical completa. No recuerda haber tenido contracciones dolorosas. Niega antecedentes mdicos y quirrgicos. El examen fsico es normal incluyendo un examen plvico que muestra un cervix largo y cerrado. Despus de una larga discusin con la paciente ella pide que se le practique un cerclaje durante este embarazo. Cual de los siguientes es el momento ms apropiado para realizarlo.

a) b) c) d)

Inmediatamente 12 a 16 semana 24 a 28 semanas 32 a 36 semanas

El cerclaje cervical tiene sus indicaciones en la profilaxis y tratamiento de la incompetencia cervical. La incompetencia o insuficiencia cervical representa un 10% de las causas de parto pretrmino y est asociada a una importante morbimortalidad neonatal. Las modificaciones cervicales en el segundo trimestre de gestacin son causa de parto prematuro y pueden deberse a: 1) Incompetencia cervical. 2) Prdida de tejido conectivo tras una ciruga cervical (conizacin). 3) Defectos congnitos como la hipoplasia cervical tras exposicin a dietilestilbestrol. 4) Infeccin intrauterina. Hasta un 51.5% de las pacientes con clnica compatible con incompetencia cervical enmascaran un cuadro de infeccin intraamnitica subclnica.

Diferenciamos tres tipos de cerclaje: 1. El cerclaje se considera profilctico o electivo (o primario) cuando se realiza de forma electiva por historia previa de incompetencia cervical antes de evidenciar cambios en el cerviz y generalmente suele realizarse entre las 13 y 16 semanas de gestacin. 2. El cerclaje teraputico secundario que se realiza tras la deteccin, en el seguimiento obsttrico, de modificaciones en el crvix antes de las 26 semanas de gestacin. Se realiza en pacientes con un riesgo potencial de parto pretrmino. 3. El cerclaje teraputico terciario, en caliente, de rescate o emergent cerclage que se realiza en pacientes que presentan la membrana amnitica visible a travs del orificio cervical externo o en vagina.

GUIA CLNICA: INDICACIONES DEL CERCLAJE Unitat de Prematuritat. Servei de Medicina Maternofetal. Institut Clnic de Ginecologia, Obstetrcia i Neonatologia, Hospital Clnic de Barcelona Responsables del protocolo: T.Cobo, M. Lpez, M. Palacio Creacin: 24/01/07 Modificaciones: 05/09/07 ltima actualizacin: 17/01/10

66.- Mujer de 26 aos, con tumor anexial de 6 cm lquido, dolor abdominal, fiebre, leucorrea, con historia de cervicovaginitis de repeticin y dispareunia crnica, ltima menstruacin hace una semana. El diagnstico ms probable es: a) b) c) d) Cistadenoma Embarazo ectpico Quiste de ovario Enfermedad plvica inflamatoria

La EIP puede cursar con los siguientes sntomas: Dolor abdominal bajo (incluyendo dolor anexial, dispareunia). Es el sntoma ms frecuente (95%) Aumento del flujo vaginal, flujo de caractersticas anormales (74%) Sangrado anormal (intermestrual, poscoital) (45%) Sntomas urinarios (35%) Vmitos (14%) Es posible la ausencia de sntomas

Y en ella podemos encontrar estos signos: Dolor a la movilizacin del cuello, dolor anexial en la exploracin vaginal bimanual (99%) En el examen con espculo observamos cervicitis y descarga endocervical purulenta (74%) Fiebre (> 38 C) (menos del 47%). Masa plvica: sugiere abceso tuboovrico (ATO) Peritonitis

CRITERIOS CLINICOS PARA EL DIAGNOSTICO DE SALPINGITIS a. Dolor abdominal con o sin rebote. b. Sensibilidad a la movilizacin del crvix. c. Sensibilidad anexial. Los tres criterios anteriores son necesarios para establecer el diagnstico, con uno o ms de los siguientes: a. Extendido de Gram de endocrvix positivo, para diplococos gram negativos intracelulares b. Temperatura mayor de 38C c. Leucocitosis (mayor de 10.000 por c.c.) d. Material purulento (positivo para leucocitos) en la cavidad peritoneal obtenido por culdocentesis o laparoscopia. Establecido el diagnstico clnico de EPI, se debe hacer la definicin del estado clnico y anatmico de la patologa plvica: a) No complicada (limitada a trompas u ovarios) 1) Sin peritonitis plvica 2) Con peritonitis plvica b) Complicada (masa inflamatoria o absceso que compromete trompa (s) u ovario (s) 1) Sin peritonitis plvica 2) Con peritonitis plvica

Bibliografa:
Beigi RH, Wiesenfeld HC. Pelvic inflammatory disease: new diagnostic criteria and treatment. Obstet Gynecol Clin Norh Am. 2003; 30 (4): 777 93 Center for Disease Control. Guidelines for treatment of sexually transmited diseases. MMWR Recomm Rep. 2002 May 10;51(RR-6):1-78 Center for Disease Control. Guidelines for prevention and management (MMWR. 40: 1 25 1991) Pelvic inflammatory disease: guidelines for prevention and management. MMWR Recomm Rep. 1991 Apr 26;40(RR-5):1-25. Hager WD, Eschenbach DA, Spence MR, Sweet RL. Criteria for diagnosis and grading of salpingitis. Obstet Gynecol. 1983 Jan;61(1):113-4. Prodigy Guidance. Pelvic inflammatory disease. [Internet]. UK : NHS, Department of Health; 2003. [Acceso 18 de Junio de 2005]. Disponible en: Ross J. Pelvic inflammatory disease. Clin Evid. 2004 Dec;(12):2259-65. Royal College of Obstetricians and Gynaecologists. Pelvic Inflammatory Disease. Guideline n 32. [Internet]. RCOG; Mayo 2003. [Acceso 18 de Junio de 2005].

67.- Una mujer de 21 aos, nuligrvida, acude a consulta para hablar sobre anticoncepcin. Es sexualmente activa desde hace 2 semanas y actualmente utiliza condn. Tiene antecedente de asma, la cual se ha mantenido inactiva por 2 aos. No toma medicamentos y niega alergias. No hay AHF de cncer. Su EF es normal. Despus de una pltica con su mdico, escoge tomar anticonceptivos orales combinados, y contina tomndola por 6 aos. Ahora ha disminuido su riesgo de desarrollar: a) b) c) d) Cncer de mama Cncer cervical Cncer heptico Cncer ovrico

PROTECCION CONTRA EL CANCER EPITELIAL OVARICO. Debido a la falta de estrategias eficaces para el diagnostico y tratamiento temprano del cncer de ovario, es de capital importancia la prevencin (76). Esta patologa es una importante causa de morbilidad y mortalidad. Se estim que para 1980 ocurrieron 137.600 casos nuevos en el mundo (86). Estudios a gran escala realizados por el Centro para el Control de las Enfermedades de EE.UU. y el Royal Collage of General Practitioners del Reino Unido (RCGP) indican que la supresin de la ovulacin causada por los anticonceptivos orales protege contra el desarrollo del cncer epitelial ovrico (20,76). Este efecto benfico es directamente proporcional al tiempo de uso y persiste muchos aos despus de suspendida la planificacin con este mtodo (87). La evaluacin norteamericana denominada: Estudio sobre cncer y hormonas (CASH) demostr que el uso de uno a cinco aos de anovulatorios orales disminuye en un 50 a 70% el riesgo de cncer ovrico (88).

Este efecto protector aumenta entre ms sea el tiempo de uso y se extiende por lo menos hasta diez aos despus de interrumpido (70,89). La Organizacin Mundial de la Salud tambin realiz un estudio multicntrico confirmando el efecto protector de los anticonceptivos orales contra el cncer epitelial ovrico (90). Dos estudios de Cohortes realizados en Gran Bretaa, confirmaron el efecto protector de la pldora al encontrar riesgos relativos de 0.3 y o.6 en mujeres que haban usado el mtodo en algn momento (83,91). El efecto protector es tanto para tumores malignos como para Bordenline (92) y cada uno de los principales subtipos histolgicos de cncer epitelial (70,93).

TABLA N 3 BENEFICIOS NO CONTRACEPTIVOS DE LOS ANTICONCEPTIVOS COMBINADOS MEJORIA DE LA DISMENORREA CORRECCION DE LOS CICLOS MENSTRUALES IRREGULARES PREVENCION DE QUISTES OVARICOS FUNCIONALES PROTECCION CONTRA EL CANCER EPITELIAL OVARICO MEJORIA DEL MITTELSCHMERZ PROTECCION CONTRA EL CANCER ENDOMETRIAL PROTECCION CONTRA TUMORES BENIGNOS MAMARIOS DISMINUCION DE LA ENFERMEDAD PELVICA INFLAMATORIA DISMINUCION EN LA INCIDENCIA DE EMBARAZO ECTOPICO PREVENCION DE ANEMIA FERROPENICA MENOR INCIDENCIA DE ARTRITIS REUMATOIDEA MENOR INCIDENCIA DE OSTEOPOROSIS POST-MENOPAUSICA MEJORIA DEL SINDROME PREMENSTRUAL PREVENCION DE LA MIOMATOSIS UTERINA MEJORIA DEL ACNE ORALES

1. Bagshaw S. the combined oral contraceptives. Risk and adverse effects in perspective. Drug-Saf 1995; 12 (2): 91 - 96. 2. American Collage of Obstetricians and Gynecologist. Hormonal contraception. ACOG technical bulletin N. 198 - October de 1994 Int J Gynaecol Obstet 1995; 48 (1): 115 - 126. 3. Mishell Jr DR. Oral contraception: past, present and future perspectives. Int J Fertil 1992; 37 (1) Suppl: 7 - 18. 4. Melo NR, Pinotti J.Advances in hormonal contraception. Adv. in contraception 1994; 10 (suppl 1): 33 - 39. 5. Winkler UH, Schindler AE, Endrikat J, et al. A comparative study of the effects of the hemostatic system of two monophasic Gestodene oral contraceptive containing 20 ug and 30 ug Etinil-Estradiol. Contraception 1996; 53: 75 - 84.

6. Coenen CMH, Thomas CMG, Borm GF, et al. Changes in androgens during treatment with four low-dose contraceptives. Contraception 1996; 53: 171 - 176 7. Wilde MI, Balfour JA. Gestodeno. A review of its pharmacology, efficacy and tolerability in combined contraceptive preparation. Drug 1995; 50 (2): 364 - 395. 8. Monterrosa A. Anticoncepcin hormonal. EN : Caraballo J, Parra E, Taylor H. Memorias del 1 Curso de actualizacin en Ginecologa y Pediatra. Imprenta U. de Cartagena. Cartagena.1994; 241 - 250. 9. Hannaford PC, Combined oral contraceptives : do we know all of their effects. Contraception 1995; 51: 325-327. 10. Rosenberg MJ, Waugh MS, Meehan T. Use and misuse of oral contraceptives : risk indicators for poor pill taking and discontinuation. Contraception 1995; 51: 283- 288.

68.- Acude a consulta una mujer de 25 aos, cursando su 14 semana de gestacin, por tenesmo vesical, disuria y escalofro. Por su estado actual, cul de los siguientes antimicrobianos recomendara?

a) b) c) d)

Metronidazol Tetraciclina Ampicilina Levofloxacina

El Metronidazol no ha mostrado efectos txicos en humanos, pero es teratognico en modelo animal. Las tetraciclinas ocasionan coloracin anormal de los dientes, hepatotoxicidad y alteracin en el desarrollo de huesos. Las sulfas podran tener un efecto deletreo en el primer trimestre dada su actividad como antimetabolitos, y en los ltimos meses pueden favorecer kernicterus en el recin nacido si es que tiene alteraciones metablicas que favorezcan anemia hemoltica. Las quinolonas se han asociado a malformaciones seas en modelos animales, y se recomienda evitarlas si existen mejores opciones. Kasper DL, Braunwald E, Fauci AS, Hauser SL, Longo DL, Jameson JL. Harrisons Principles of Internal Medicine. McGraw Hill. 16 Ed. 789-806 pp.

69.- Femenino de 23 aos acude al servicio l servicio de ginecologa, por referir ciclos opsomenorreicos desde el inicio de su menarquia, en los ltimos 7 das ha incrementado 15 Kg. de lo que pesaba habitualmente, se aprecia una gran cantidad de acne, pero adems refiere depilarse el rea del bigote cada semana, y cree que esto le sensibiliza la piel para que aumente el acne. En el caso de ovario poliqustico el dato clnico que con ms frecuencia les acompaa es:

a) b) c) d)

Anovulacin y esterilidad Hirsutismo Amenorrea Obesidad

El sndrome de ovarios poliqusticos (SOPQ) afecta aproximadamente a un 4% de mujeres en edad reproductiva y se caracteriza por anovulacin crnica e hiperandrogenismo. Es la causa ms comn de infertilidad en mujeres. Se caracteriza clnicamente por acn, alopecia, hirsutismo, irregularidades menstruales e infertilidad. Los hallazgos de laboratorio ms frecuentes son: aumento de la hormona luteinizante (LH), aumento de la relacin LH/FSH (hormona folculoestimulante), aumento de andrgenos (tanto ovricos como adrenales) y de estrgenos circulantes. Otros hallazgos de laboratorio habituales son una prueba tolerancia oral a la glucosa anormal y alteraciones en el perfil lipdico. Todo esto junto con las imgenes ecocardiogrficas caractersticas define al sndrome. La teraputica permite dos grandes enfoques que pueden superponerse: la correccin de las manifestaciones de hiperandrogenismo y el tratamiento de las alteraciones del eje reproductivo (anovulacin, esterilidad). Los antiandrgenos estn fundamentalmente indicados para tratar los sntomas virilizantes. Las alternativas para inducir la ovulacin son numerosas: al citrato de clomifeno y a la antigua reseccin en cua se agregan las gonadotrofinas humanas, pulsos de GnRH (hormona liberadora de gonadotrofinas), medidas o frmacos para modificar los niveles de insulina, y finalmente tcnicas quirrgicas endoscpicas para reducir la masa ovrica.

BIBLIOGRAFIA 1. Guzick D.Polycystic ovary syndrome: Symptomatology, pathophysiology, and epidemiology. Am J Ostetric Gynecol 1998; 179 (6): 89-93. 2. Stephen Franks. Polycystic ovary syndrome. N Engl J Med 1995; 333(13): 853-861. 3. Gori J.R., Larusso A. Ginecologa de Gori. 2 Edicin. Buenos Aires, Argentina. Editorial El Ateneo. 2001. 4. Adams J., Polson D. W., Franks S. Prevalence of polycystic ovaries in women with anovulation and idiopathic hirsutism. Br Med J 1986; 293: 355-9. 5. Copeland L. J . Ginecologa. Buenos Aires, Argentina. Editorial Panamericana. 1 Edicin. 1994.

6. Ehrmann D.A., Rosenfield R.L., Barnes R.B., Brigell D.F., Sheikh Z. Detection of functional ovarian hyperandrogenism in women with androgen excess. N Engl J Med 1992; 327:157-162. 7. Kahasar-Miller M., Conway Myers B., Boots L., Azziz R. Steroidogenic acute regulatory protein (StAR) in the ovaries of healthy women and those with polycystic ovary syndrome. Am J Obstet Gynecol 2001; 185(6): 1381-7. 8. Prez Snchez A. Ginecologa. Santiago de Chile. Publicaciones Tcnicas Mediterrneo. 3 Edicin. 1995. 9. Velzquez E., Mendoza S., Hamer T., Sosa F., Glucck C. Metformin therapy in women with polycistic ovary syndrome reduces hiperinsulinemia, insulin resistence, hyperandrogenemia, and systolic blood pressure, while facilitating menstrual regularity and pregnancy. Metabolism 1994 ; 43: 647-655.

70.- Se trata de femenino de 31 aos, con un cuadro clnico de sangrado menstrual irregular, dismenorrea, dispareunia, y una esterilidad de 3 aos de evolucin, con resultados de laboratorio hormonal normal y un estudio ecogrfico transvaginal que informa de un tero normal y sendas formaciones qustica ovricas bilaterales de 4 cms . Cul sera la orientacin diagnstica? a) Hemorragia uterina disfuncional. b) Sndrome del ovario poliqustico. c) Endometriosis. d) Quistes dermoides bilaterales.

Causas y sntomas de endometriosis Las causas de la endometriosis an no se conocen. Las clulas del revestimiento interno del tero de alguna manera se desplazan hasta zonas externas al mismo y siguen creciendo. Este desplazamiento podra quizs deberse a que pequeos fragmentos del revestimiento uterino, desprendidos durante la menstruacin, retrocedan hacia las trompas de Falopio en direccin a los ovarios hasta entrar en la cavidad abdominal, en lugar de salir con el flujo menstrual a travs de la vagina. La endometriosis causa dolor en la parte inferior del abdomen y la zona plvica, irregularidades menstruales (como manchar antes de la menstruacin) e infertilidad. Algunas mujeres con endometriosis grave no presentan sntomas, mientras que otras con la enfermedad en grado mnimo sufren un dolor invalidante. Con frecuencia, el dolor menstrual debido a la endometriosis no aparece hasta aos despus de desarrollar la enfermedad. En algunos casos, se constata dolor durante el coito (dispareunia), antes o durante la menstruacin. El tejido endometrial adherido al intestino grueso o a la vejiga urinaria puede provocar hinchazn abdominal, dolor durante las deposiciones, hemorragia rectal

durante la menstruacin o dolor en la parte inferior del abdomen durante la miccin. As mismo, cuando el tejido se localiza en un ovario o una estructura cercana puede dar lugar a la formacin de una masa llena de sangre (endometrioma). En ocasiones, el endometrioma se rompe bruscamente o se escapa algo de su contenido, lo que causa un agudo y repentino dolor abdominal.

71.- Paciente de 40 aos con ndulo mamario indoloro, de bordes imprecisos. La mamografa revela imagen nodular, con espculas en todos sus mrgenes, y 10 microcalcificaciones finas, agrupadas en el interior. El diagnstico ms probable, entre los que se citan, es: a) b) c) d) Fibroadenoma. Quiste Mamografa normal para la edad de la paciente. Carcinoma

Tcnicas diagnsticas Exploracin Masa palpable o engrosamiento unilateral. La posibilidad de que una masa palpable en la mama sea maligna est en relacin con mayor edad, postmenopausia y con las siguientes caractersticas en el examen fsico: consistencia firme, aspecto slido, bordes irregulares, escaso desplazamiento sobre la piel, la regin costal o los tejidos que le rodean, unilateral, no dolorosa y la presencia de adenopatas axilares. Sin embargo, an en ausencia de estos factores un 10% pueden ser malignas, algunas veces una zona de engrosamiento que no llega a masa puede ser cncer. La coexistencia de masa y adenopata axilar palpable debe considerarse cncer mientras no se demuestre lo contrario. El 90 % de las masas suelen ser lesiones benignas. Las masas de superficie lisa y consistencia elstica estn asociadas a fibroadenoma en mujeres entre 20-30 aos y a quistes en las mujeres de 30 a 40. La exploracin a realizar ante esta situacin es una mamografa si hay antecedentes de cncer de mama y una ecografa sobre todo si existe dolor (ICSI, 2005). Secrecin por el pezn. Siempre se debe estudiar. Hay mayor riesgo de lesin maligna en el caso de que la secrecin contenga restos hemticos y est asociado a masa. La citologa del lquido expulsado slo puede ser tenida en cuenta si es positiva. Est indicado realizar mamografa y galactografa en el caso de que el exudado se presente en un solo conducto. La presencia de secrecin lechosa bilateral orienta a causa endocrinolgica se ha de realizar el diagnstico diferencial de galactorrea (ICSI, 2005). Dolor. Es uno de los motivos de consulta mas frecuente. En ausencia de masa otros sntomas de sospecha suele ser debida a tensin premenstrual, dolor condrocostal y a otras causas (ICSI, 2005). Est asociado con mayor frecuencia a cambios fibroqusticos en la mama premenopusica. Sntomas cutneos. La Enfermedad de Paget afecta al pezn y areola de forma unilateral, clnicamente muy similar a la dermatitis crnica crnica eccematosa se asocia a un carcinoma mamario intraductal subyacente. (Fitzpatrick, 2001)

La retraccin del pezn o de la piel de presentacin reciente se debe evaluar cuidadosamente. Los fenmenos inflamatorios del tipo de eritema, induracin, aumento de temperatura y dolor pueden ser indicativos de un tumor inflamatorio de mal pronstico. En ocasiones un tumor evolucionado puede dar lugar a un cncer ulcerado. Imgenes Mamografa Tiene una sensibilidad y especificidad del 90%, siendo el mtodo aislado de diagnstico ms eficaz, aunque en mamas densas pierde sensibilidad. Utilizada para el screening puede reducir la mortalidad del cncer de mama en un 33%. Nos puede dar el diagnostico, la presencia de multicentricidad o de lesiones sincrnicas. La mamografa nos va a valorar distintos tipos de imgenes: 1: Signos primarios: A. Masa dominante: Valorando tamao, densidad, forma nitidez y estabilidad en el seguimiento. Es la lesin ms frecuentemente hallada. Considerando la clnica y los datos mamogrficos, ecogrficos, etc. se establece la probabilidad de malignidad del ndulo, pudiendo expresarla en las siguientes cuatro categoras:

o o o o

Benigno Probablemente benigno Probablemente maligno Maligno

NDULO BENIGNO

NDULO PROBABLEMENTE BENIGNO

NDULO PROBABLEMENTE MALIGNO

NDULO MALIGNO

B. Lesiones estrelladas (o de alteracin de la arquitectura): Representadas por reas de distorsin de la arquitectura mamaria, de bordes irregulares y que adoptan una morfologa radiada. Suele ser un signo temprano en el carcinoma de mama y es de difcil interpretacin. Posee menos valor predictivo positivo que el ndulo o las microcalcificaciones, por lo que se recomienda biopsia quirrgica en todos los casos excepto en los que los antecedentes de traumatismo, ciruga previa o inflamacin permitan optar por el seguimiento de la lesin. En estos casos siempre es conveniente haber realizado una citologa con resultado negativo.

C. Microcalcificaciones: Son hallazgos frecuentes y el anlisis de sus caractersticas nos puede ayudar a diferenciar las benignas de las sospechosas y de las claramente malignas. Las benignas no requieren mas pruebas diagnsticas complementarias, las probablemente benignas precisas de un seguimiento mamogrfico no inferior a los dos aos y en las sugestivas de malignidad, la biopsia es preceptiva. Hay que analizar las siguientes caractersticas: Tamao: Las superiores a 2 mms. se clasifican de macrocalcificaciones y suelen ser benignas. Por debajo de los 2 mms. se denominan microcalcificaciones y cuanto ms pequeas y agrupadas ms sospechosas son de malignidad Morfologa: Las calcificaciones malignas suelen ser heterogneas en forma y tamao, puntiagudas, anguladas, irregulares, en "coma", ramificadas y con forma de punto y raya. Las benignas suelen ser homogneas, redondas y en ocasiones anulares y de centro claro. Nmero: Se considera que cuando hay cinco o ms calcificaciones menores de 1 mm. En un rea de 1x1 cm. de mamografa, existe sospecha de malignidad. Cuanto mayor es el n de calcificaciones en esa rea, ms sospechosas son. Distribucin: Las calcificaciones distribuidas de forma segmentaria, no al azar, son sospechosas e indicativas de biopsia. Variacin en el tiempo de las calcificaciones: Las calcificaciones malignas varan con el tiempo. La estabilidad de las calcificaciones durante ao y medio - dos aos, se consideran como benignas. Calcificaciones asociadas a mama: Los carcinomas de mama calcifican en un 50 %. Cuando hay calcificaciones internas en lesiones con signos de malignidad, aumentan las posibilidades de malignidad. Se hallan en un 75% de los cnceres ocultos y suponen el 3047% de hallazgo aislado en los cnceres de mama. Son el primer marcador de cncer de mama en las mujeres jvenes.

2: Signos secundarios:

1. 2. 3. 4. 5.

Engrosamiento de la piel. Permeabilidad linftica. Aumento de la vascularizacin. Afectacin linftica. Dilatacin ductal.

Bibliografa:
Apantaku LM. Breast cancer diagnosis and screening. [Internet]. American Family Physician; 2000 [ acceso 28/6/2007]. Disponible en: http://www.aafp.org/afp/20000801/596.html Barratt A, Howard K, Irwig L, Salkeld G and Houssami N. Model of outcomes of screening mammography: information to support informed choices. BMJ 2005;330;936940.[Texto completo] Brewer NT, Salz T, Lillie SE. Systematic review: the long-term effects of falsepositive mammograms. Ann Intern Med. 2007;146(7):502-10 [PubMed] [Texto completo] Breast Imaging reporting and data system (BI-RADS). 2nd ed. Reston (VA): American College of Radiology; 2007 [Resea] Cantin J, Scart H, Levine M, Hugi M. Clinical practice guidelines for the care and treatment of breast cancer: 13. Sentinel lymph node biopsy. Can. Med. Assoc. J. 2001; 165: 166 173 [Texto completo] Dynamed. Breast cancer (female9) [Internet]. Ebsco industries ;2007 [acceso 3/7/22007]. Disponible en http://dynamed102.ebscohost.com/Detail.aspx?id=114433 Eberl MM, Fox ChH, Edge SB, Carter CA, Mahoney MC. BI-RADS Classification for Management of abnormal Mammograms. J Am Board Fam Med 2006;19:161-4 [PubMed] [Texto completo] Fitzpatrick TB, Johnson RA, Wolff K, Suurmod D. Atlas color y sinopsis de dermatologa clnica. Madrid: McGraw Hill Interamericana; 2001. p.494-5

72.- Femenino de 32 aos de edad es llevada a quirfano por una laparoscopa diagnstica debido a dolor plvico y en cuadrante superior izquierdo crnicos (durante los ltimos 2 aos). No tiene alteraciones funcionales vesicales o intestinales. Tiene antecedente de 2 episodios de gonorrea previos. Bebe una cerveza al da. Labs: HCG urinaria negativa; Hto 39%; Leuc. T 8 000; Plt 200 000; AST 12; ALT 14. Intraoperatoriamente se observan adhesiones densas que involucran los oviductos, ovarios y tero. Tambin se observan adhesiones perihepticas que se extienden desde la superficie heptica hacia el diafragma. Cual de los siguientes es el diagnstico ms probable?

a) b) c) d)

Sx de Fitz-Hugh-Curtis Hepatitis Carcinoma hepatocelular Sx Wolff-Parkinson-White

El sndrome de Fitz-Hugh-Curtis se define como la presencia de una perihepatitis asociada a salpingitis. Los agentes etiolgicos reconocidos hasta la fecha son Chlamydia trachomatis y Neisseria gonorrhoeae. El cuadro clnico de este sndrome es inespecfico y puede ser confundido con procesos inflamatorios o infecciosos del tubo digestivo, aparato urinario y respiratorio, en los cuales la manifestacin sintomtica fundamental es el dolor en hipocondrio derecho. El diagnstico debe de sospecharse en aquella mujer joven con vida sexual activa que tenga antecedentes de promiscuidad en ella o en su pareja, que se queje de dolor subcostal derecho. Es ms probable el diagnstico si se cuenta con el antecedente de enfermedad plvica inflamatoria y ms an, si se tiene evidencia de que sta sea causada por Neisseria gonorrehoeae y/o Chlamydia trachomatis. El diagnstico definitivo se realiza con la visualizacin directa de la adherencia periheptica por laparoscopa o laparotoma. Se recomienda la primera. El tratamiento mdico es a base de cefalosporinas y dicloxacilina y en algunos casos se requiere de la extirpacin quirrgica del proceso adherencial para mitigar el dolor.

El Sndrome de Fitz-Hugh-Curtis. Causa frecuente de error de diagnstico en hepatologa y gastroenterologa / The Fitz-Hugh-Curtis Syndrome. a frequent misdiagnosis in hepatology and gastroenterology Rev. gastroenterol. Mx;60(4):223-8, oct.-dic. 1995.

73.- Femenino de 20 aos de edad refiere irregularidades menstruales tipo hipo-opsooligomenorrea desde hace 3 aos. Niega tener vida sexual activa y no recuerda su fecha de ltima menstruacin. No hay antecedente de galactorrea ni de uso de hormonales exgenos. Mide 164cm y pesa 60kg. Sin datos de hirsutismo, las mamas, tero y anexos son normales. El ultrasonido plvico es normal, as como el perfil hormonal. El diagnstico clnico ms probable es:

a) b) c) d)

Hiperplasia del endometrio Sangrado uterino disfuncional Alteracin menstrual fisiolgica Endometriosis

El ciclo menstrual normal ha cambiado en el curso de los siglos, dependiendo de las modificaciones en el patrn reproductivo y el estado nutricional de la poblacin.

El debut del sangrado menstrual, definido como menarquia, marca una etapa importante en la madurez biolgica durante la pubertad. En el Programa Nacional de Atencin Integral a la Salud de los Adolescentes se mencionan los trastornos menstruales como una de las primeras causas de consulta o de urgencia en servicios clnicos de primero y segundo nivel. Diversas alteraciones menstruales, ya sea las que inducen aumento en la cantidad o frecuencia o las que se presentan como episodios infrecuentes e irregulares se observan en la adolescencia y pueden impactar la calidad de vida de las pacientes, con repercusiones significativas en el mbito reproductivo y metablico. Por esas razones podemos afirmar que los trastornos menstruales constituyen un problema de salud pblica y requieren un enfoque ms integral en aspectos preventivos y curativos en la etapa de la adolescencia. Aproximadamente la mitad de todas las adolescentes tienen perodos irregulares durante el primer ao despus de la menarquia. Estos periodos irregulares pueden persistir hasta cinco aos despus de la menarquia en 20 % de estas adolescentes. La principal complicacin es la anemia, que puede ser severa y raramente tiene consecuencias fatales. PATRN MENSTRUAL NORMAL

La Federacin Internacional de Ginecologa y Obstetricia considera el patrn menstrual normal entre tres y cuatro das, aunque flucta entre dos y siete das. El intervalo entre menstruaciones es de veintiocho das, considerndose como lmites de veintiuno a treinta y cinco das. El volumen de sangre menstrual es de sesenta a ochenta mililitros; no obstante, se consideran normales los rangos de sangrado entre cincuenta y ciento cincuenta mililitros. El aspecto de la sangre es rojo oscuro incoagulable. En la prctica mdica se utiliza un grupo de trminos para hacer referencia a las diversas alteraciones del ciclo menstrual, que requieren precisin por la frecuencia en que son diagnosticados. Segn Schiavon (2000), las alteraciones menstruales ms frecuentes son: oligoamenorrea: episodios de sangrado infrecuentes, irregulares, con intervalo de ms de cuarenta das;

Polimenorrea: episodios frecuentes pero regulares de sangrado uterino, que ocurren a intervalos menores de veintin das;

Menorragia: sangrado excesivo, tanto en cantidad como en duracin, que ocurre con regularidad y es sinnimo de hipermenorrea;

Metrorragia: sangrado generalmente no excesivo, que ocurre a intervalos irregulares; Menometrorragia: sangramiento generalmente excesivo y prolongado, que ocurre a intervalos frecuentes e irregulares; uterino regular, pero disminuido en cantidad; Hipomenorrea: sangrado Sangrado intermenstrual: sangrado uterino generalmente no excesivo, que ocurre entre perodos menstruales regulares. Las irregularidades menstruales son causa frecuente de consulta en las adolescentes, siendo 95 % de las veces de naturaleza disfuncional, por inmadurez del eje hipotlamo hipofisoovrico (HHO). Uno de los primeros problemas que hay que plantear ante estas irregularidades menstruales, es la hemorragia uterina disfuncional (HUD). Su definicin guarda relacin con las caractersticas en cantidad y frecuencia que difieren del sangrado menstrual normal. Con mayor frecuencia se encuentra en forma de sangrados excesivos y prolongados, asociados a ciclos anaovulatorios, en ausencia de una patologa o enfermedad existente, aunque raramente la HUD puede presentarse con ciclos ovulatorios. De forma prctica, consideramos una hemorragia uterina (HU) como anormal cuando el sangrado es excesivo, con cualquier desvo o alteracin de su duracin, cantidad o intervalo. El diagnstico de HUD supone una alteracin de origen endocrino (eje HHO); por lo tanto, su diagnstico impone haber descartado cualquier patologa orgnica y sistmica que produzca hemorragia genital. Es un diagnstico por exclusin. ETIOPATOGENIA DE LA HUD EN LA ADOLESCENCIA Los ciclos anovulatorios son ms frecuentes en las adolescentes por la inmadurez del eje HHO en el primer ao tras la menarquia. En esos casos de HUD, como ya se mencion, se producen ciclos anovulatorios que se traducen en una proliferacin desorganizada del endometrio por falta de efecto progestagnico. Una vez que el endometrio alcanza un grosor crtico, comienza a descamarse en forma irregular, traducindose en un sangrado permanente de cuanta variable.

Las manifestaciones clnicas de la HUD son:

Fases de amenorrea de dos a cuatro meses, seguidas de salida de sangre abundante durante tres o cuatro semanas; en oportunidades existe irregularidad completa en el sangrado; sangrado de ms de seis compresas (bien empapadas) al da;

Presencia de cogulos; Suele ser indolora;

Menstruaciones de ms de siete das de duracin;

Ciclos de menos de veintin das. La gravedad de esta hemorragia se clasifica, de acuerdo con el grado de anemia que produzca, en metrorragia leve, moderada o grave: Leve: metrorragia leve y prolongada, ciclo menstrual acortado, hemoglobina y hematocrito normales. Moderada: metrorragia copiosa prolongada, ciclo menstrual acortado, anemia leve (cifras de hemoglobina inferior a diez gramos por litro). Grave: metrorragia copiosa prolongada, ciclo acortado e irregular, anemia grave (cifras de hemoglobina de ocho gramos por litro o menos).

Ante un sangramiento uterino en estas edades se debe realizar el diagnstico diferencial con: a) Gestacin y problemas relacionados con sta, tales como abortos y gravidez ectpica; b) Coagulopata: 20 % de las adolescentes con hemorragia uterina tienen un defecto de la coagulacin. La manifestacin ms precoz de alteraciones de la coagulacin sangunea puede ser evidenciada por un sangramiento genital anormal, lo cual puede estar relacionado con deficiencias de plaquetas, leucemias, prpuras, enfermedad de Von Willebrand, deficiencia de protrombina u otros factores de la coagulacin; c) Malformaciones del aparato genital, traumatismosgenitales, presencia de cuerpos extraos; d) Dispositivos intrauterinos; e) Tumores uterinos, sarcoma botroides o tumores anexiales;

f) Hipo o hipertiroidismo; g) Insuficiencia renal o heptica.

Como el diagnstico de HUD es de exclusin, hay que hacer una historia clnica minuciosa, exmenes complementarios y sin falta descartar las otras causas de sangramiento transvaginal. Se debe precisar con detalle el nivel de desarrollo puberal, la actividad sexual y la presencia de situaciones concomitantes como: a) contacto sexual sin proteccin contraceptiva; b) uso irregular de anticonceptivos orales o antecedentes de insercin de dispositivos intrauterinos; c) ejercicios fsicos extenuantes; d) historia previa de sangrado excesivo, asociado a extracciones dentarias, pequeas heridas, epistaxis y otras; e) dolencias renales u hepticas preexistentes.

Ante cualquier demanda de atencin por adolescentes con sangramiento genital con las caractersticas descritas, se requiere de un examen fsico general que incluya exploracin general completa, toma de tensin arterial y pulso, bsqueda de exoftalmia, fascie Cushing, visceromegalias o presencia de masas abdominales palpables, edemas parpebrales y de miembros superiores, as como puntos hemorrgicos en epidermis y otros signos de coagulopata. El examen ginecolgico debe realizarse en todas las adolescentes, con excepcin de las que no han tenido actividad sexual y presentan sangramiento leve. Adems del examen de sus genitales, hay que efectuar una valoracin citolgica y microbiolgica en particular en quienes presenten manifestaciones clnicas. En la inspeccin de los genitales durante el examen de la paciente, es importante evaluar que el sangramiento se origine en lesiones ubicadas en los genitales externos, uretra u hemorroides, as como indagar acerca de la posibilidad de abuso sexual. Si despus del tratamiento de la HUD leve contina el sangrado, se recomienda la realizacin de ecografa abdominal para precisar el diagnstico. En pacientes que ya han tenido relaciones sexuales se debe buscar si el tero tiene caractersticas gravdicas, la posibilidad de un aborto en curso, as como la presencia de una masa anexial que permita corroborar la existencia de embarazo o alguna neoplasia benigna o maligna. El examen con espculo podr demostrar un cuello hipermico, sangrante o gravdico; de encontrarse estos hallazgos, se descartara la etiologa disfuncional del sangrado.

Exmenes complementarios bsicos que no pueden faltar: Hemograma completo, Coagulograma completo, Orina, Ultrasonido ginecolgico abdominal, transvaginal o transrectal segn proceda, Ecografa abdominal. Si fuese necesario por los signos identificados en el examen de la paciente, se deben realizar: Dosificacin de FSH, LH, T3, T4, TSH y prolactina si hay sospechas clnicas de otras enfermedades endocrinas concomitantes, Laparoscopia en casos seleccionados por patologa de base, Otros, segn hallazgos de la historia clnica y la exploracin.

74.- Femenino 40 aos, G-3 P-2 A-1, se detecta de anemia ferropnica, de 9.5 g/dl, refiere ciclos menstruales de 31,32 x 8,9 das de duracin, acompaados de cogulos, los cuales aparecieron despus del nacimiento de su segundo hijo hace 12 aos. e.f.: con ligera palides de tegumentos, S/V dentro de los parmetros normales, , genitales con evidencia de sangrado activo, al tacto vaginal bimanual se detecta tero de consistencia firme voluminoso, irregular, aproximadamente de 12 cm. anexos libres. El tratamiento para esta paciente es:

a) b) c) d)

Histerectoma total sin conservar anexos. Histerectoma total conservando anexos. Histerectoma vaginal. Histerectoma radical.

Tratamiento: La ciruga es el tratamiento ms comn del mioma uterino: Miomectomia: Extirpar slo el mioma, los tumores nicos y accesibles, la reseccin histeroscpica de miomas submucosos tambin es posible realizarlo con electrocoagulacin en pacientes con hemorragias (>90%). Histerectoma: La ciruga puede ser abdominal o laparoscpica, la decisin final de la tcnica depender de la eleccin del cirujano, en base al caso individual y a su experiencia, el procedimiento puede ser: -Histerectoma subtotal. -Histerectoma total (remocin cervical). Tcnicamente ms sencillo en su realizacin. La incidencia de carcinoma con origen en el mun cervical es menor al 1% en nuestros das.

Bibliografa: 1. Hanafi m. predictors of leiomyoma recurrence after myomectomy. am coll obstet ginecol 2005; 105: 877-880. 2. Inclan j, mojarra j. miomectoma histeroscpica. abordaje actual para el manejo de los miomas submucosos. reporte de un caso y revisin de la literatura. bol clin hosp. infant edo son 2001;18: 29-34. 3. goldrath mh, husain m. the hysteroscopic management of endometrial leiomyomatosis. j am assoc gynecol laparosc 1997; 4: 263-267. 4. clement pb, scully re. mullerin adenofibroma of the uterus with invasion of myometrium and pelvic veins. int j gynecol pathol 1990; 9: 363-371. 5. ravina jh, herbreteau d, ciraru-vigneron n, bouret jm, houdart e, aymard a, merland jj. arterial embolization to treat uterine myomata. lancet 1995; 346: 671-672. 6. kunhardt-urquiza e, cruz si, fernndez-martnez rl, hernndez-ziga ve: miomatosis de localizacin poco frecuente. ginecol obstet mex 1997; 65: 541-544.

75.- Femenino de 23 aos, acude a su consultorio refiriendo secrecin vaginal ftida. Exploracin Fsica: especuloscopa se observa secrecin gris que rodea la cpula vaginal. En la preparacin en fresco se observan cocobacilos que rodean a las clulas epiteliales. La medida teraputica ms apropiada en esta paciente es?

a) b) c) d)

Clindamicina Clotrimazol Metronidazol Tratamiento para ella y su pareja.

Metronidazol: Tratamiento especfico en vaginosis, la evidencia apoya en primer lugar el uso de: metronidazol por va oral, en dosis de 500mg cada 12 horas durante siete das, con una tasa de curacin de 80 a 90 %. Para el tratamiento local no se ha demostrado una diferencia significativa en la efectividad para curarla vaginosis, al comparar clindamicina en crema a 2 % y metronidazol vaginal, ni entre stos y el tratamiento con metronidazol por va bucal (Ia y III).4,8 En vaginosis tampoco se ha demostrado la utilidad de tratar a la pareja sexual (Ia). Trejo y PJA, Hernndez LB , Carrasco RJR , Ducoing DDLR Gua clnica para el diagnstico, tratamiento y prevencin de cervicovaginitis por bacterias, Trichomonas y Candida Rev Med IMSS 2003; 41 (Supl 1):71-76

76.- A 23-year-old nulligravid female has not menstruated in the past 4 months. Previously, her menstrual cycles were regular. She is otherwise well and denies recent onset of stress, change in exercise routine, headaches, visual field alterations, or galactorrhea.. She has a body mass index of 24, blood pressure of 120/78 mm Hg, and does not appear hirsute. No adnexal masses can be palpated. Laboratory investigations reveal a negative beta human chorionic gonadotropin (-hCG), normal thyroid-stimulating hormone (TSH), and prolactin levels. What is the next best step in the management of this patient? a) b) c) d) Preescribe 7 days of medroxyprgesterona and reevaluate. Measurement of luteinizing hormone. CT scan of the sella turca. Prescribe oral estrogen for 21 days followed by 7 days of medroxyprgesterona and

reevaluate.

La Prueba de respuesta a progestgenos se basa en la observacin de que el tratamiento con progestgeno (acetato de medroxiprogesterona 10 mg por 5 a 6 das) solo induce la menstruacin en las mujeres con concentraciones normales de estrgenos circulantes. Una prueba positiva (hemorragia despus de concluir el tratamiento con progestgenos) seala cifras normales de produccin de estrgenos y una prueba negativa (sin hemorragia por privacin), hipogonadismo franco. Danforth, Tratado de Obstetricia y Ginecologia, 9 Edicin, Ed. Mc Graw Hill Interamericana, Pg 668.

77.- El cncer de tiroides que puede producir un sndrome paraneoplsico y que se asocia a elevaciones de calcitonina es: a) Cncer anaplsico b) Cncer de clulas de Hrttle c) Cncer medular d) Cncer papilar

El cncer medular de tiroides surge de las clulas parafoliculares de la tiroides, que normalmente producen calcitonina. La medicin de calcitonina es importante sobre todo en el seguimiento de los pacientes para detectar enfermedad residual o recidivante. Jimnez RSA, Gmez VE, Bolaos GF. Tiroides. En Flores JF, Cabeza A, Calarco Z (eds): Endocrinologa. 5 ed. Mxico. Mndez Oteo Mxico, 2005: 584-92.

78.- Cual de las medidas siguientes es ms adecuada para combatir la hiperbilirrubinemia (l0mg/dl) de un lactante de 3 semanas, con un desarrollo y crecimiento normales, que recibe lactancia materna?

a) b) c) d)

Fototerapia Exanguineotransfusin Fenobarbital Esperar un par de das y repetir la prueba.

La fototerapia es el empleo de luz visible para el tratamiento de hyperbilirubinemia en el recin nacido (RN). Esta terapia relativamente comn baja el nivel de bilirrubina en el suero por transformacin de la bilirrubina en ismeros solubles en agua que pueden ser eliminados sin la conjugacin en el hgado La fototerapia convierte la bilirrubina que est presente en los capilares superficiales y espacio intersticial a ismeros solubles en agua que son excretables sin pasar por el metabolismo del hgado (Fig. 4). Maisels, un notable experto en bilirrubina, sugiere que la fototerapia se parece mucho a una droga percutanea. Cuando la fototerapia ilumina la piel, una infusin de fotones de energa, como molculas de una medicina, es absorbida por la bilirrubina de la misma manera que una molcula de medicina se une a un receptor. Las molculas de bilirrubina en la piel expuestas a la luz sufren las reacciones fotoqumicas relativamente rpido, configurational isomerization, isomerizacin

estructural, y la forma de fotooxidacin no txica, ismeros excretables. Estos ismeros de bilirrubina tienen formas diferentes del ismero natal, son ms polares, y pueden ser excretados del hgado en la bilis sin sufrir la conjugacin o requerir transporte especial para su excrecin. La eliminacin urinaria y gastrointestinal son ambas importantes en reducir la carga de bilirrubina.

Figura 4. El mecanismo de fototerapia. Cuando las molculas de bilirrubina absorben la luz, 2 reacciones fotoqumicas principales ocurren: el natural 4Z, 15Z-bilirubin se convierte a 4Z, 15E bilirubin (tambin conocido como photobilirrubina) y a lumirrubina. A diferencia de 4Z, 15Z la bilirrubina, photobilirrubina puede ser excretado va heptica sin la conjugacin, pero su clearance es muy lento, y su conversin es reversible. En el intestino (lejos de la luz), photobilirrubina es convertida atrs a bilirubina natal. La lumirrubina no es reversible. Aunque mucho menos lumirrubina que photobilirrubina es formado, lumirrubina es eliminado del suero mucho ms rpidamente, y es probable que la formacin de lumirrubina es principalmente responsable de la disminucin en el suero de la bilirrubina. Las pequeas cantidades de bilirrubina natal tambin son oxidadas a monopyrroles y dipyrroles que pueden ser excretados en la orina. Esto es un proceso lento y slo un contribuidor menor a la eliminacin de bilirrubina durante la fototerapia. Cortesa de diagrama de Mara Puchalski.

El objetivo de la fototerapia es disminuir la bilirrubina srica y prevenir su acumulacin txica en el cerebro, donde puede causar serias complicaciones neurolgicas

permanente conocido como kernicterus. La fototerapia ha reducido enormemente la necesidad de exanguneo transfusin para tratar la hiperbilirrubinemia.

La fototerapia es usada de 2 modos principales: profilctica y teraputicamente. En RN prematuros o aquellos con un conocido proceso hemoltico, a menudo es usado profilacticamente, para prevenir un rpido aumento de la bilirrubina srica. En pretrminos pequeos o RN de trmino, es administrada en dosis teraputicas para reducir niveles de bilirrubina excesivos y evitar el desarrollo de kernicterus. La fotoisomerizacin de bilirrubina comienza casi al instante cuando la piel es expuesta a la luz. A diferencia de la bilirrubina no conjugada, los fotoproductos de estos procesos no son neurotxicos. Por lo tanto, ante una hiperbilirrubinemia severa del RN, es importante comenzar la fototerapia sin retraso.

Referencias:
Stokowski LA. Early recognition of jaundice and kernicterus. Adv Neonatal Care 2002;2:101-114. Maisels MJ. A primer on phototherapy for the jaundiced newborn. Contemp Pediatr. 2005; 22(6): passim. (OR, 38, 40, 44, 47, 48, 53, 54,57). 1. McDonagh AF. Phototherapy: from ancient Egypt to the new millenium. J

Perinatol 2001;21:S7-S12.
Maisels MJ. Phototherapy-traditional and nontraditional. J Perinatol 2001; 21(Suppl 1):S93-S97.

79.- Cuando nos enfrentamos ante un paciente que presenta un cuadro clnico caracterizado por amenorrea, galactorrea y prdida de campo visual el diagnstico ms probable es:

a) Adenoma hipofisario no funcionante. b) Intoxicacin por benzodiacepinas. c) Prolactinoma. d) Meningioma del tubrculo solar.

El prolactinoma es el tumor hipofisario ms frecuente, es de naturaleza benigna y pequeo en 90 % de los casos. El cuadro clnico tpico en la mujer se compone de trastornos menstruales, galactorrea y/o esterilidad; se acompaa de sntomas neurolgicos slo cuando se extiende por arriba de la silla turca. Niveles de prolactina superiores a 100 ng/mL son prcticamente diagnsticos de prolactinoma, siempre y cuando no existan embarazo y/o hipotiroidismo. La primera opcin teraputica del prolactinoma es la farmacolgica con dopaminrgicos, lo que prcticamente ha eliminado la ciruga. Los dopaminrgicos suprimen la sntesis y secrecin de prolactina con la consecuente normalizacin del eje hipotlamo-gonadotrpico. Los dopaminrgicos son efectivos para inducir la ovulacin y favorecen la consecucin de embarazo. Para cualquier dimensin del prolactinoma se usan los dopaminrgicos durante uno a dos aos y generalmente despus de suspenderlos se puede esperar que el tumor se reduzca de tamao y se corrija la hiperprolactinemia. En contraste, en el hombre generalmente se encuentran macroprolactinomas, mayores de 10 milmetros con extensin extraselar acompandose de sntomas neurolgicos; sin embargo, tambin responden favorablemente a los dopaminrgicos.

Gac Md Mx Vol. 140 No. 5, 2004 Referencias: 1. Schlechte JA. Prolactinoma. N Engl J Med 2003;349:2035-2041. 2. Zrate A, Canales ES, Jacobs LS, Soria J, Daughaday WH. Restoration of ovarian function in patients with the amenorrhea-galactorrhea syndrome after long-term therapy with L-Dopa. Fertil Steril 1973;24:340. 3. Tyson JE, Carter JN, Andreassen B, Huth J, Smith B. Nursing mediated prolactin and luteinizing hormone secretion during puerperal lactation. Fertil Steril 1978;30:154. 4. Schlechte JA, Sherman BM, Chapler FK, VanGilder J. Long-term followup of women with surgically treated prolactin-secreting pituitary tumors. J Clin Endocrinol Metab 1986;62:1296-301. 5. Losa M, Mortini P, Barzaghi R, Gioia L, Giovanelli M. Surgical treatment of prolactin-secreting pituitary adenomas: early results and long-term outcome. J Clin Endocrinol Metab 2002;87:3180-3186. 6. Zrate A, Canales ES, Cano C, Pilonieta CJ. Follow-up of patients with prolactinomas after discontinuation of long-term therapy with bromocriptine. Acta Endocrinol 1983;104:139-42. 7. Zrate A, Canales ES, Alger M. The effect of pregnancy and lactation on pituitary prolactin secreting tumors. Acta Endocrinol 1979;92:407-11. 8. Bevan JS, Webster J, Hburke J, Scanlon MF. Dopamine agonists and pituitary tumor shrinkage. Endocr Rev 1992;13:220-240.

80.- Masculino de 3 aos, preescolar. Es atendido en consulta. desde los 2 aos de edad, al llorar presenta cianosis labial y peribucal, cuando se golpea o al regaarlo, no pierde el conocimiento, la cianosis desaparece al ceder el llanto, le ocurre casi a diario, no hay antecedente familiar de enfermedad neurolgica o cardiovascular. Examen fsico peso 13.6 kg., talla 93 cm., resto sin datos patolgicos. En este paciente el diagnstico ms probable es:

a) Epilepsia. b) Espasmo del sollozo. c) Tetralogia de fallot. d) Enfermedad por reflujo gastroesofgico.

Espasmo del sollozo Es la retencin de la respiracin posterior a un evento que disguste tal como una cada, el estar frustrado o enojado, o por estar asustado. -El nio da uno o dos gritos largos y posteriormente retiene su respiracin en expiracin hasta que sus labios se ponen azules. -El nio posteriormente se desmaya (algunos llegan a tener algunos espasmos musculares). despus el nio respira normalmente y permanece completamente alerta en menos de 1 minuto. -El inicio ocurre entre 6 meses y 2 aos. Esto sucede solamente cuando el nio est despierto. Se define al espasmo del sollozo (Breath-holding spells), como un evento caracterizado por crisis recurrentes de apnea transitoria, prdida del conocimiento y cambios en el tono muscular normal. De inicio sbito, no seconsidera secundario a patologa orgnica ni resultado de una manifestacin psiquitrica. Entre el 5 y el 7% de la poblacin infantil sana presenta crisis de espasmos de sollozos, sin embargo, algunos autores coinciden en que esta entidad es mucho ms frecuente. Por lo general dichos eventos inician entre los 6 a 12 meses de edad con un pico de incidencia entre los 12 y los 24 meses de edad. Es poco frecuente que se presente en nios mayores de 6 aos de edad por lo que su aparicin en nios mayores de 4 aos amerita especial atencin. El 25% de los nios que los presentan tiene un familiar directo que lo padeci en la infancia. Se presenta ms frecuentemente en varones. Son causas frecuentes de sncopes y convulsiones anxicas secundarias a isquemia o hipoxia, con la consecuente depresin sbita de la funcin neuronal, en contraste con las convulsiones epilpticas, secundaria a una descarga excesiva de las neuronas. El espasmo del sollozo a sido dividido en el espasmo del sollozo plido y ciantico, dependiendo de la coloracin de la piel durante el evento. La fisiopatologa, en cada caso en particular, es diferente. Entender las caractersticas del espasmo del sollozo y poder diferenciarlo de otras entidades, puede ayudar al pediatra a tranquilizar a los padres.

Bibliografa:

1- Berman, RE., Kliegman, RM., Jenson, HB: Nelson Textbook of Pediatrics. 16th edition, Philadelphia, Pennsylvania, W.B. Saunders Company U.S.A.., 2,000; 1829 2- Anderson JE, Bluestone D: espasmos del sollozo. Contemp Pediatr 2,000;17(1):61-72 3- Macan H, et al: Espasmo del sollozo en 91 nios y respuesta al tratamiento con hierro. Arch Dis Child 1999;81:261-262. 4- Breningstall GN: Breath-holding Spells . Pediatr Neurol 1996;14:91-97 5- DiMario FJ: Breath-holding spell in childhood. Am J Dis Child 1992;146:125-131 6- Lombroso CT, Lei-man P: Breath-holding spell (cyanotic and pallid in fantile syncope). Pediatrics 1967;38:563-581. 7- Gauk EW,Kidd L, Prichard JS: Mechanism of seizures associated with breath-holding spell. N Engl J Med 1963; 268: 1436-1441.

Potrebbero piacerti anche